4 Physiology Supplement Handout Based on Ganong for TOPNOTCH 21 by Jaffar Pineda

June 19, 2018 | Author: Athena Borja | Category: Muscle Contraction, Lipoprotein, Axon, Actin, Immune System
Share Embed Donate


Short Description

physio supple...

Description

TOPNOTCH MEDICAL BOARD PREP PHYSIOLOGY GANONG SUPPLEMENT HANDOUT BY THE TOPNOTCH TEAM For inquiries visit www.topnotchboardprep.com.ph or https://www.facebook.com/topnotchmedicalboardprep/

PHYSIOLOGY SUPPLEMENTAL Instructions: Please Read Ganong’s Review of Medical Physiology  23 Physiology  23rd  or 25 th Ed before using this.

SECTION 1: CELLULAR & MOLECULAR BASIS FOR MEDICAL PHYSIOLOGY (CHAPTER 1) GENERAL PRINCIPLES & ENERGY PRODUCTION IN MEDICAL PHYSIOLOGY ANSWER: B Equilibrium potential can be calculated for K+: 1. The membrane potential of a particular cell is at the EK= 61.5loG [Ko+]/[Ki+] at 37 °C K+equilibrium. K+equilibrium. The intracellular concentration for K+ is at EK= 61.5loG [5.5]/[150] 150 mmol/L and the extracellular concentration concentration for K+ is EK= -90 mV  mV  at 5.5 mmol/L. What is the resting potential? Where A. –70 mV EK = equilibrium potential for K+ B. –90 mV [Ko+] = K+ concentration outside the cell C. +70 mV [Ki+] = K+ concentration concentration inside the cell D. +90 mV Chloride has equilibrium potential of -70 mV, Sodium has equilibrium potential of +60 mV. •



Ganong. Review of Medical Physiology 23rd ed. Chapter 1, page .7 

2. The difference in concentration of H+ in a solution of pH 2.0 compared with one of pH 7.0 is: A. 5-fold B. 1/5 as much C. 105-fold D. 10– 10–5 as much

ANSWER: C “For each pH unit less than 7.0, the [H+] is increased tenfold (10 1); for each pH unit above 7.0, it is decreased tenfold (10 -1).” The difference in pH between 2.0 and 7.0 is 5, therefore an increased of 105 is expected.

3. Transcription refers to: A. the process where an mRNA is used as a template for protein production. B. the process where a DNA sequence is copied into RNA for the purpose of gene expression. C. the process where DNA wraps around histones to form a nucleosome. nucleosome. D. the process of replication of DNA prior to cell division

ANSWER: B

4. The primary structure of a protein refers to: A. the twist, folds, or twist and folds of the amino acid sequence into stabilized structures within the protein (ie, αα -helices and β-sheets). β -sheets). B. the arrangement of subunits to form a functional structure. C. the amino acid sequence of the protein. D. the arrangement of twisted chains and folds within a protein into a stable structure. 5. Fill in the blanks: Glycogen is a storage form of glucose. _______ refers to the process of making glycogen and _______ refers to the process of breakdown of glycogen. A. glycogenolysis, glycogenolysis, glycogenesis B. glycolysis, glycogenolysis C. glycogenesis, glycogenolysis D. glycogenolysis, glycogenolysis, glycolysis





Ganong. Review of Medical Physiology 23rd ed. Chapter 1, page 4.

Transcription is the process where a DNA sequence is copied into RNA for the purpose of gene expression. Translation is the process where an mRNA is used as a template for protein production.  Synthesis is the process of replication of DNA prior to cell division.







Ganong. Review of Medical Physiology 23rd ed. Chapter 1, page 4.

ANSWER: C

Primary structure is the amino acid sequence of the protein.  Secondary structure structure is the twist, folds, or twist and folds of the amino acid sequence into stabilized structures within the protein (ie, α -helices and β-sheets). β-sheets). Tertiary structure is the arrangement of twisted chains and folds within a protein into a stable structure. Quaternary structure is the arrangement of subunits to form a functional structure.

• •





Ganong. Review of Medical Physiology 23rd ed. Chapter 1, page 17.

ANSWER: C •



“For each pH unit less than 7.0, the [H+] is increased tenfold (10 1); for each pH unit above 7.0, it is decreased tenfold (10 -1).” The difference in pH between 2.0 and 7.0 is 5, therefore an increased of 105 is expected. Ganong. Review of Medical Physiology 23rd ed. Chapter 1, page 20

ANSWER:D ANSWER: D •

6. The major lipoprotein source of the cholesterol used in cells is: A. chylomicrons B. intermediate-density intermediate-density lipoproteins (IDL) C. albumin-bound free fatty acids D. low-density lipoproteins lipoproteins (LDL) E. high-density lipoproteins (HDL)





“Low -density -density lipoproteins (LDL) is the major lipoprotein source of the cholesterol used in cells. Chylomicrons is the transport system for ingested exogenous lipids (exogenous pathway). Intermediate-density lipoproteins (IDL) give up phospholipids and, through the action of the plasma enzyme lecithin-cholesterol acyltransferase (LCAT), pick up cholesteryl esters formed from cholesterol in the HDL. High-density lipoproteins (HDL), also transports triglycerides and cholesterol throughout the body.” Ganong. Review of Medical Physiology 23rd ed. Chapter 1 , page 27 .

7. Which of the following produces the most high-energy phosphate compounds? A. Aerobic metabolism of 1 mol of glucose B. Anaerobic metabolism of 1 mol of glucose C. Metabolism of 1 mol of galactose D. Metabolism of 1 mol of amino acid E. Metabolism of 1 mol of long-chain fatty acid

ANSWER:

8. When LDL enters cells by receptor-mediated endocytosis, which of the following does not occur? A. in the formation of cholesterol cholesterol from mevalonic acid B. in the intracellular concentration of cholesteryl esters C. in the transfer of cholesterol from the cell to HDL D.  in the rate of synthesis of LDL receptors E.  in the cholesterol in endosomes

ANSWER:  E ANSWER: E When LDL enters cells by receptor-mediated endocytosis, there will be: 1. Decrease in the formation of cholesterol and in the rate of synthesis of LDL receptors. 2. Increase in the intracellular concentration of cholesteryl esters, transfer of cholesterol from the cell to HDL, and amount of cholesterol cholesterol in endosomes



“Catabolism of 1 mol of a six -carbon fatty acid through the citric acid cycle to CO2 and H2O generates 44 mol of ATP , compared with the 38 mol generated by catabolism of 1 mol of the six-carbon carbohydrate glucose.” Ganong. Review of Medical Physiology 23rd ed. Chapter 1 , page 23.



Ganong. Review of Medical Physiology 23rd ed. Chapter 1 , page 27 .

TOPNOTCH MEDICAL BOARD PREP MED PHYSIOLOGY GANONG SUPPLEMENT HANDOUT BY THE TOPNOTCH TEAM Page 1 of 46 For inquiries visit www.topnotchboardprep.com.ph or https://web.facebook.com/topnotchmedicalboardprep/

TOPNOTCH MEDICAL BOARD PREP PHYSIOLOGY GANONG SUPPLEMENT HANDOUT BY THE TOPNOTCH TEAM For inquiries visit www.topnotchboardprep.com.ph or https://www.facebook.com/topnotchmedicalboardprep/ (CHAPTER 2) OVERVIEW OF CELLULAR PHYSIOLOGY IN MEDICAL PHYSIOLOGY ANSWER: A ANSWER:  A 1. The electrogenic Na, K ATPase plays a critical role in cellular physiology by A. using the energy in ATP to extrude 3 Na+ out of the “Na, K ATPase catalyzes the hydrolysis of ATP to adenosine diphosphate cell in exchange for taking two K+ into the cell. (ADP) and uses the energy to extrude three Na+ from the cell and take B. using the energy in ATP to extrude 3 K+ out of the cell two K+ into the cell for each molecule of ATP hydrolyzed.” in exchange for taking two Na+ into the cell. It is an electrogenic pump which does not produce new ATP. C. using the energy in moving Na+ into the cell or K+ outside the cell to make ATP. Ganong. Review of Medical Physiology 23rd ed. Chapter 2, page 47. D. using the energy in moving Na+ outside of the cell or K+ inside the cell to make ATP. •



2. Cell membranes: A. contain relatively few protein molecules. B. contain many carbohydrate molecules. C. are freely permeable to electrolytes but not to proteins. D. have variable protein and lipid contents depending on their location in the cell. E. have a stable composition throughout the life of the cell.

ANSWER: D

3. Second messengers: A. are substances that interact with first messengers outside cells. B. are substances that bind to first messengers in the cell membrane. C. are hormones secreted by cells in response to stimulation by another hormone. D. mediate the intracellular responses to many different hormones and neurotransmitters. neurotransmitters. E. are not formed in the brain.

ANSWER: D

4. The Golgi complex A. is an organelle that participates in the breakdown of proteins and lipids. B. is an organelle that participates in posttranslational processing of proteins. C. is an organelle that participates in energy production. D. is an organelle that participates in transcription and translation. E. is a subcellular compartment that stores proteins for trafficking to the nucleus. 5. Endocytosis A. includes phagocytosis and pinocytosis, but not clathrin mediated or caveolae-dependent caveolae-dependent uptake of extracellular contents. B. refers to the merging of an intracellular vesicle with the plasma membrane to deliver intracellular contents to the extracellular milieu. C. refers to the invagination of the plasma membrane to uptake extracellular contents into the cell. D. refers to vesicular trafficking between between Golgi stacks. 6. G-protein– G-protein–coupled receptors: A. are intracellular membrane proteins that help regulate movement within the cell. B. are plasma membrane proteins that couple the extracellular binding of primary signaling molecules to exocytosis. C. are plasma membrane proteins that couple the extracellular binding of primary signaling molecules to the activation of heterotrimeric G-proteins. D. are intracellular proteins that couple the binding of primary messenger molecules with transcription.

7. Gap junctions are intercellular connections connections that A. primarily serve to keep cells separated and allow for transport across a tissue barrier. B. serve as a regulated cytoplasmic bridge for sharing of small molecules between cells. C. serve as a barrier to prevent protein movement within the cellular membrane. D. are cellular components for constitutive exocytosis that occurs between adjacent cells.

“Cellular Membranes have variable protein and lipid contents depending on their location in the cell and is  semipermeable, allowing some substances to pass through it and excluding others. There were many proteins embedded in the membrane with abundant  phospholipids.” Cell membrane have variable composition throughout life of the cell.





Ganong. Review of Medical Physiology 23rd ed. Chapter 2, page 32 .



“ Second messengers bring about many short-term changes in cell function by altering enzyme function, triggering exocytosis, and also can lead to the alteration of transcription of various genes.” Ganong. Review of Medical Physiology 23rd ed. Chapter 2, page 51..

ANSWER: B •



• •



Golgi complex is an organelle that participates in posttranslational processing of proteins. Lysosome is an organelle that participates in the breakdown of proteins and lipids. Mitochondrion is an organelle that participates in energy production. Nucleus is an organelle that participates in transcription and translation. Ribosomal Endoplasmic Reticulum is a subcellular compartment that stores proteins for trafficking to the nucleus. Ganong. Review of Medical Physiology 23rd ed. Chapter 2, page 40.

ANSWER: C •



“Endocytosis refers to the invagination of the plasma membrane to uptake extracellular contents into the cell. It includes phagocytosis, pinocytosis, clathrin mediated or caveolae dependent uptake of extracellular contents.” “Exocytosis refers “Exocytosis refers to vesicular trafficking between Golgi stacks. The process refers to the merging of an intracellular vesicle with the plasma membrane to deliver intracellular con tents to the extracellular milieu.” Ganong. Review of Medical Physiology 23rd ed. Chapter 2, page 43 .

ANSWER: C



“G -protein -protein–coupled receptors are plasma membrane proteins that couple the extracellular binding of primary signaling molecules to the activation of heterotrimeric G- proteins.” Ganong. Review of Medical Physiology 23rd ed. Chapter 2, page 54..

ANSWER: B “Tight junctions provide intercellular connections that link cells into a regulated tissue barrier, it also provides barrier to movement of proteins in the cell membrane and thus, are important to cellular polarization. Gap junctions provide contacts between cells that allow for direct passage of small molecules between two cells. Desmosomes and adherens junctions are specialized structures that hold cells together. Hemidesmosomes and focal adhesions attach cells to their basal lamina.” lamina.” •







Ganong. Review of Medical Physiology 23rd ed. Chapter 2 , page 60 ..

TOPNOTCH MEDICAL BOARD PREP MED PHYSIOLOGY GANONG SUPPLEMENT HANDOUT BY THE TOPNOTCH TEAM Page 2 of 46 For inquiries visit www.topnotchboardprep.com.ph or https://web.facebook.com/topnotchmedicalboardprep/

TOPNOTCH MEDICAL BOARD PREP PHYSIOLOGY GANONG SUPPLEMENT HANDOUT BY THE TOPNOTCH TEAM For inquiries visit www.topnotchboardprep.com.ph or https://www.facebook.com/topnotchmedicalboardprep/ (CHAPTER 2) OVERVIEW OF CELLULAR PHYSIOLOGY IN MEDICAL PHYSIOLOGY ANSWER: A ANSWER:  A 1. The electrogenic Na, K ATPase plays a critical role in cellular physiology by A. using the energy in ATP to extrude 3 Na+ out of the “Na, K ATPase catalyzes the hydrolysis of ATP to adenosine diphosphate cell in exchange for taking two K+ into the cell. (ADP) and uses the energy to extrude three Na+ from the cell and take B. using the energy in ATP to extrude 3 K+ out of the cell two K+ into the cell for each molecule of ATP hydrolyzed.” in exchange for taking two Na+ into the cell. It is an electrogenic pump which does not produce new ATP. C. using the energy in moving Na+ into the cell or K+ outside the cell to make ATP. Ganong. Review of Medical Physiology 23rd ed. Chapter 2, page 47. D. using the energy in moving Na+ outside of the cell or K+ inside the cell to make ATP. •



2. Cell membranes: A. contain relatively few protein molecules. B. contain many carbohydrate molecules. C. are freely permeable to electrolytes but not to proteins. D. have variable protein and lipid contents depending on their location in the cell. E. have a stable composition throughout the life of the cell.

ANSWER: D

3. Second messengers: A. are substances that interact with first messengers outside cells. B. are substances that bind to first messengers in the cell membrane. C. are hormones secreted by cells in response to stimulation by another hormone. D. mediate the intracellular responses to many different hormones and neurotransmitters. neurotransmitters. E. are not formed in the brain.

ANSWER: D

4. The Golgi complex A. is an organelle that participates in the breakdown of proteins and lipids. B. is an organelle that participates in posttranslational processing of proteins. C. is an organelle that participates in energy production. D. is an organelle that participates in transcription and translation. E. is a subcellular compartment that stores proteins for trafficking to the nucleus. 5. Endocytosis A. includes phagocytosis and pinocytosis, but not clathrin mediated or caveolae-dependent caveolae-dependent uptake of extracellular contents. B. refers to the merging of an intracellular vesicle with the plasma membrane to deliver intracellular contents to the extracellular milieu. C. refers to the invagination of the plasma membrane to uptake extracellular contents into the cell. D. refers to vesicular trafficking between between Golgi stacks. 6. G-protein– G-protein–coupled receptors: A. are intracellular membrane proteins that help regulate movement within the cell. B. are plasma membrane proteins that couple the extracellular binding of primary signaling molecules to exocytosis. C. are plasma membrane proteins that couple the extracellular binding of primary signaling molecules to the activation of heterotrimeric G-proteins. D. are intracellular proteins that couple the binding of primary messenger molecules with transcription.

7. Gap junctions are intercellular connections connections that A. primarily serve to keep cells separated and allow for transport across a tissue barrier. B. serve as a regulated cytoplasmic bridge for sharing of small molecules between cells. C. serve as a barrier to prevent protein movement within the cellular membrane. D. are cellular components for constitutive exocytosis that occurs between adjacent cells.

“Cellular Membranes have variable protein and lipid contents depending on their location in the cell and is  semipermeable, allowing some substances to pass through it and excluding others. There were many proteins embedded in the membrane with abundant  phospholipids.” Cell membrane have variable composition throughout life of the cell.





Ganong. Review of Medical Physiology 23rd ed. Chapter 2, page 32 .



“ Second messengers bring about many short-term changes in cell function by altering enzyme function, triggering exocytosis, and also can lead to the alteration of transcription of various genes.” Ganong. Review of Medical Physiology 23rd ed. Chapter 2, page 51..

ANSWER: B •



• •



Golgi complex is an organelle that participates in posttranslational processing of proteins. Lysosome is an organelle that participates in the breakdown of proteins and lipids. Mitochondrion is an organelle that participates in energy production. Nucleus is an organelle that participates in transcription and translation. Ribosomal Endoplasmic Reticulum is a subcellular compartment that stores proteins for trafficking to the nucleus. Ganong. Review of Medical Physiology 23rd ed. Chapter 2, page 40.

ANSWER: C •



“Endocytosis refers to the invagination of the plasma membrane to uptake extracellular contents into the cell. It includes phagocytosis, pinocytosis, clathrin mediated or caveolae dependent uptake of extracellular contents.” “Exocytosis refers “Exocytosis refers to vesicular trafficking between Golgi stacks. The process refers to the merging of an intracellular vesicle with the plasma membrane to deliver intracellular con tents to the extracellular milieu.” Ganong. Review of Medical Physiology 23rd ed. Chapter 2, page 43 .

ANSWER: C



“G -protein -protein–coupled receptors are plasma membrane proteins that couple the extracellular binding of primary signaling molecules to the activation of heterotrimeric G- proteins.” Ganong. Review of Medical Physiology 23rd ed. Chapter 2, page 54..

ANSWER: B “Tight junctions provide intercellular connections that link cells into a regulated tissue barrier, it also provides barrier to movement of proteins in the cell membrane and thus, are important to cellular polarization. Gap junctions provide contacts between cells that allow for direct passage of small molecules between two cells. Desmosomes and adherens junctions are specialized structures that hold cells together. Hemidesmosomes and focal adhesions attach cells to their basal lamina.” lamina.” •







Ganong. Review of Medical Physiology 23rd ed. Chapter 2 , page 60 ..

TOPNOTCH MEDICAL BOARD PREP MED PHYSIOLOGY GANONG SUPPLEMENT HANDOUT BY THE TOPNOTCH TEAM Page 2 of 46 For inquiries visit www.topnotchboardprep.com.ph or https://web.facebook.com/topnotchmedicalboardprep/

TOPNOTCH MEDICAL BOARD PREP PHYSIOLOGY GANONG SUPPLEMENT HANDOUT BY THE TOPNOTCH TEAM For inquiries visit www.topnotchboardprep.com.ph or https://www.facebook.com/topnotchmedicalboardprep/ ANSWER:  A ANSWER: A “Filamentous (F) actin provides a structural component for cell 8. F-actin is a component of the cellular cytoskeleton cytoskeleton that: movement. A. provides a structural structural component for cell movement. Globular (G) actin refers to the actin subunits that provide the B. is defined as the “functional” form of actin in the cell. molecular building blocks of the extended actin molecules found in the C. refers to the actin subunits that provide the molecular cell. building blocks blocks of the extended actin actin molecules molecules found i Microtubules provides the molecular architecture for cell to cell the cell. communication . D. provides the molecular architecture for cell to cell Intermediate filaments form a flexible scaffolding for the cell and help communication. it resist external pressure.” •







Ganong. Review of Medical Physiology 23rd ed. Chapter 2, pages 35,36 ..

(CHAPTER 3) IMMUNITY, INFECTION, & INFLAMMATION 1. In an experiment, a scientist treats a group of mice with an antiserum that substantially depletes the number of circulating neutrophils. Compared with untreated control animals, the mice with reduced numbers of neutrophils were found to be significantly more susceptible to death induced by bacterial inoculation. The increased mortality can be ascribed to a relative deficit in which of the following? A. Acquired immunity B. Oxidants C. Platelets D. Granulocyte/macrophage colony stimulating factor (GM-CSF) E. Integrins 2. A 20-year-old college student comes to the student health center in April complaining of runny nose and congestion, itchy eyes, and wheezing. She reports that similar symptoms have occurred at the same time each year, and that she obtains some relief from over-the-counter antihistamine drugs, although they make her too drowsy to study. Her symptoms can most likely be attributed to inappropriate synthesis of which of the following antibodies specific for tree pollen? A. IgA B. IgD C. IgE D. IgG E. IgM

ANSWER: B “Neutrophils mount “Neutrophils mount phagocytic responses that engulf and destroy bacteria. Reduced number of which, will lead to deficiency in oxidation or release of reactive oxygen species.”  Acquired immunity is the ability of lymphocytes to produce antibodies. GM-CSF stimulates production of monocyte and granulocyte, reduced neutrophil will not lead in deficiency of this factors. Platelets are circulating cells that are important mediators of hemostasis. Integrins promotes cellular adhesion.



• •





Ganong. Review of Medical Physiology 23rd ed. Chapter 3 , page 64 .

ANSWER: C



• • • • •

IgE: releases histamine from basophils and mast cells which triggers the patient’s symptoms. IgG: Complement activation IgA: Localized protection in external secretions (tears, intestinal secretions, IgM: Complement activation IgD : Antigen recognition by B cells Ganong. Review of Medical Physiology 23rd ed. Chapter 3 , page 74 .

ANSWER: E 3. If a nasal biopsy were performed on the patient described in Question 2 while symptomatic, histologic examination of the tissue would most likely reveal degranulation of which of the following cell types? A. Dendritic cells B. Lymphocytes C. Neutrophils D. Monocytes E. Mast cells 4. A biotechnology company is working to design a new therapeutic strategy for cancer that involves triggering an enhanced immune response to cellular proteins that are mutated in the disease. Which of the following immune cells or processes will most likely not be required for a successful therapy? A. Cytotoxic T cells B. Antigen presentation in the context of MHC-II C. Proteosomal degradation D. Gene rearrangements rearrangements producing T cell receptors E. The immune synapse



• • •



Mast cell degranulation produces clinical manifestations manifestations of allergy up to and including anaphylaxis. Dendritic cells are antigen presenting cells. Lymphocytes are key elements in the production of immunity. Neutrophils enters the tissues, particularly if triggered to do so by an infection or by inflammatory cytokines. Monocytes are activated into macrophage and migrate in response to chemotactic chemotactic stimuli and engulf and kill bacteria. bacteria. Ganong. Review of Medical Physiology 23rd ed. Chapter , pages 64, 65,71 .

ANSWER: B •



proteins) are coupled primarily to “The class I MHC proteins (MHC-I ( MHC-I proteins) peptide fragments to which the host is not tolerant (eg, those from   proteins) are recognized by cytotoxic T cells. mutant or viral  proteins) The digestion of these proteins occurs in  proteasomes. The class II MHC proteins ) are concerned primarily with peptide proteins (MHC-II (MHC-II proteins) products of extracellular antigens, such as bacteria, that enter the cell endosomes .” by endocytosis and are digested in the late endosomes.” Ganong. Review of Medical Physiology 23rd ed. Chapter 3 , page 71 ..

ANSWER: B 5. The ability of the blood to phagocytose pathogens and mount a respiratory burst is increased by: A. interleukin-2 (IL-2) B. granulocyte colony-stimulating colony-stimulating factor (G-CSF) C. erythropoietin erythropoietin D. interleukin-4 (IL-4) E. interleukin-5 (IL-5)



• •



Granulocyte colony-stimulating factor (G-CSF): promotion of the growth of granulocytes which is responsible for phagocytosis and respiratory burst. Erythropoietin: stimulates production of RBC. sensitization and thus in Interleukin-4 (IL-4): plays a part in mast-cell sensitization allergy and in defense against nematode infections. differentiation of eosinophils Interleukin-5 (IL-5): differentiation Ganong. Review of Medical Physiology 23rd ed. Chapter 3, page 68.

TOPNOTCH MEDICAL BOARD PREP MED PHYSIOLOGY GANONG SUPPLEMENT HANDOUT BY THE TOPNOTCH TEAM Page 3 of 46 For inquiries visit www.topnotchboardprep.com.ph or https://web.facebook.com/topnotchmedicalboardprep/

TOPNOTCH MEDICAL BOARD PREP PHYSIOLOGY GANONG SUPPLEMENT HANDOUT BY THE TOPNOTCH TEAM For inquiries visit www.topnotchboardprep.com.ph or https://www.facebook.com/topnotchmedicalboardprep/ 6. Cells responsible for innate immunity are activated most commonly by: A. glucocorticoids B. pollen C. carbohydrate sequences in bacterial cell walls D. eosinophils E. thrombopoietin

ANSWER: C “The cells that mediate innate immunity include neutrophils, macrophages, and natural killer (NK) cells. All these cells respond to lipid and carbohydrate sequences unique to bacterial cell walls and to other substances characteristic of tumor and transpla nt cells.” Glucocorticoids, Glucocorticoid s, pollen, eosinophils, and thrombopoietin are not responsible for innate immunity. •



Ganong. Review of Medical Physiology 23rd 23rd ed. hapter 3, page 69.

7. A patient suffering from an acute flare in his rheumatoid arthritis undergoes a procedure where fluid is removed from his swollen and inflamed knee joint. Biochemical analysis of the inflammatory cells recovered from the removed fluid would most likely reveal a decrease in which of the following proteins? A. Interleukin-1 B. Tumor necrosis factor-α factor- α C. Nuclear factor-κB factor- κB D. IκBα E. von Willbrand factor

ANSWER: D

8. In normal human blood: A. the eosinophil is the most common type of white blood cell B. there are more lymphocytes than neutrophils C. the iron is mostly in hemoglobin D. there are more white cells than red cells E. there are more platelets than red cells

 C ANSWER: C ANSWER:





“Rheumatoid arthritis is an inflammatory disease in which the nuclear factor-κB, factor-κB, plays a key role. NF-κB  is   is a heterodimer that normally exists in the cytoplasm of cells bound to IκBα, which renders it inactive.” Therefore, IκBα is decreased in rheumatoid arthritis. Ganong. Review of Medical Physiology 23rd ed. Chapter 3 , page 76.



In human blood, iron is mostly in hemoglobin, neutrophil is the most common type of white blood cell, and there are more red blood cells than white blood cells and platelets.

ANSWER: E 9. Lymphocytes: Lymphocytes: A. all originate from the bone marrow after birth. B. are unaffected by hormones C. convert to monocytes in response to antigens D. interact with eosinophils to produce platelets E. are part of the body’s defense against cancer

• •



Lymphocytes are part of the body’s defense against cancer. “After birth, most of lymphocytes are formed in the lymph nodes, thymus, and spleen from precursor cells that originally came from the bone marrow.” Lymphocytes cannot be converted to monocytes in response to antigens and doesn’t interact with eosinophils to prod uce platelets. Ganong. Review of Medical Physiology 23rd 23rd ed. Chapter 3, page 65.

(CHAPTER 4) EXCITABLE TISSUE: NERVE 1. Which of the following statements about glia is true? ANSWER: C A. Microglia arise from macrophages outside of the nervous system and are physiologically and “Protoplasmic astrocytes produce substances that are tropic to embryologically similar to other neural cell types. neurons to help maintain the appropriate concentration of ions and B. Glia do not undergo proliferation. proliferation. neurotransmitters by taking up K+ and the neurotransmitters glutamate C. Protoplasmic astrocytes produce substances that are and GABA.” tropic to neurons to help maintain the appropriate “Microglia arise “Microglia arise from macrophages outside of the nervous system and concentration of ions and neurotransmitters by are physiologically and embryologically embryologically unrelated  to   to other neural cell taking up K+ and the neurotransmitters glutamate glutamate types; they are scavenger cells that resemble tissue macrophages and and GABA. remove debris resulting from injury, infection, and disease.” D. Oligodendrocytes and Schwann cells are involved in “Oligodendrocytes and Schwann cells are involved in myelin myelin formation around axons in the peripheral and formation around axons in the central and peripheral nervous central nervous systems, respectively.  systems, respectively. Glia undergo proliferation.” E. Macroglia are scavenger cells that resemble tissue macrophages and remove debris resulting from Ganong. Review of Medical Physiology 23rd ed. Chapter 4 , page 80 . injury, infection, and disease. 2. Primary erythromelalgia, which may be due to a ANSWER: E peripheral nerve sodium channelopathy, was diagnosed in a 13-year-old girl who was experiencing frequent “The number of Na+ channels per square micrometer of membrane in episodes of red, painful, warm extremities. Which part of myelinated mammalian neurons has been estimated to be 50 –75 in the a neuron has the highest concentration of Na+ channels 350–500 in the initial segment , less than 25 on the  surface cell body , 350– per square micrometer of cell membrane? , 2000– 2000–12,000 at the nodes of Ranvier , and 20– 20–75 at the of the myelin A. dendrites .” axon terminals B. cell body near dendrites C. initial segment Ganong. Review of Medical Medical Physiology 23rd 23rd ed. Chapter 4, page 85. D. axonal membrane under myelin E. node of Ranvier 3. A 45-year-old woman who works in an office had been ANSWER: E experiencing tingling in her index and middle fingers and thumb of her right hand. Recently, her wrist and hand had become weak. Her physician ordered a nerve conduction “In general, the greater the diameter of a given nerve fiber, the greater test to evaluate her for carpal tunnel syndrome. Which one its speed of conduction.” of the following following nerves has the the slowest conduction velocity velocity In decreasing order of diameter and speed: A. Aα fibers Aα fibers > Aβ fibers > Aγ  fibers > B fibers > C fibers B. Aβ fibers C. Aγ fibers Ganong. Review of Medical Physiology 23rd ed. Chapter 4, page 89. D. B fibers E. C fibers •









• •

TOPNOTCH MEDICAL BOARD PREP MED PHYSIOLOGY GANONG SUPPLEMENT HANDOUT BY THE TOPNOTCH TEAM Page 4 of 46 For inquiries visit www.topnotchboardprep.com.ph or https://web.facebook.com/topnotchmedicalboardprep/

TOPNOTCH MEDICAL BOARD PREP PHYSIOLOGY GANONG SUPPLEMENT HANDOUT BY THE TOPNOTCH TEAM For inquiries visit www.topnotchboardprep.com.ph or https://www.facebook.com/topnotchmedicalboardprep/ ANSWER: A 4. Which of the following is not correctly paired? A. Synaptic transmission: Antidromic conduction B. Molecular motors: Dynein and kinesin C. Fast axonal transport: ~400 mm/day D. Slow axonal transport: 0.5–10 mm/day E. Nerve growth factor: Retrograde transport



“In the natural situation, impulses pass in one direction only, ie , from synaptic junctions or receptors along axons to their termination. Such conduction is called orthodromic. Conduction in the opposite direction is called antidromic. Because synapses, unlike axons, permit conduction in one direction only, an antidromic impulse will fail to pass the first synapse they encounter and die out at that point.” Ganong. Review of Medical Physiology 23rd ed. Chapter 4, page 88 .

5. Which of the following ionic changes is correctly matched with a component of the action potential? A. Opening of voltage-gated K+ channels: Afterhyperpolarization B. A decrease in extracellular Ca2+: Repolarization C. Opening of voltage-gated Na+ channels: Depolarization D. Rapid closure of voltage-gated Na+ channels: Resting membrane potential E. Rapid closure of voltage-gated K+ channels: Relative refractory period 6. A man falls into a deep sleep with one arm under his head. This arm is paralyzed when he awakens, but it tingles, and pain sensation in it is still intact. The reason for the loss of motor function without loss of pain sensation is A. A fibers are more susceptible to hypoxia than B fibers. B. A fibers are more sensitive to pressure than C fibers. C. C fibers are more sensitive to pressure than A fibers. D. Motor nerves are more affected by sleep than sensory nerves. E. Sensory nerves are nearer the bone than motor nerves and hence are less affected by pressure. 7. Which of the following statements about nerve growth factor is not true? A. It is made up of three polypeptide subunits. B. It is responsible for the growth and maintenance of adrenergic neurons in the basal forebrain and the striatum. C. It is necessary for the growth and development of the sympathetic nervous system. D. It is picked up by nerves from the organs they innervate. E. It can express both p75 NTR and Trk A receptors. 8. A 20-year-old female student awakens one morning with severe pain and blurry vision in her left eye; the symptoms abate over several days. About 6 months later, on a morning after playing volleyball with friends, she notices weakness but not pain in her right leg; the symptoms intensify while taking a hot shower. Which of the following is most likely to be the case? A. The two episodes described are not likely to be related. B. She may have primary-progressive multiple sclerosis. C. She may have relapsing-remitting multiple sclerosis. D. She may have a lumbar disk rupture. E. She may have Guillain–Barre syndrome.

ANSWER: C Opening of voltage-gated Na+ channels: Depolarization Opening of voltage-gated K+ channels: Repolarization A decrease in extracellular Ca2+: increased excitability  affects Resting Membrane Potential Rapid closure of voltage-gated Na+ channels: Inactivated sate  Relative Refractory Period  Slow   closure of voltage-gated K+ channels:  After-hyperpolarization

• • •





Ganong. Review of Medical Physiology 23rd ed. Chapter 4 , page 85

ANSWER: B “Conversely, pressure on a nerve can cause loss of conduction in largediameter motor , touch, and pressure fibers while  pain sensation remains relatively intact.” •

Ganong. Review of Medical Physiology 23rd ed. Chapter 4, page 89, 90.

ANSWER: B



Nerve Growth Factor is made up of three polypeptide subunits. It can be express both  p75 NTR and Trk  A receptors. It is necessary for the growth and development of the  sympathetic   nervous system; and cholinergic   neurons in the basal forebrain and the striatum. It is picked up by nerves from the organs they innervate. Ganong. Review of Medical Physiology 23rd ed. Chapter 4, page 90

ANSWER: •



“Typical physiological deficits of multiple sclerosis range f rom muscle weakness, fatigue, diminished coordination, slurred speech, blurred or hazy vision, bladder dysfunction, and sensory disturbances. Symptoms are often exasperated by increased body temperature or ambient temperature.” “In the most common form, transient episodes appear suddenly, last a few weeks or months, and then gradually disappear. Subsequent episodes can appear years later, and eventually  full recovery does not occur . Others have a  progressive form of the disease in which there are no periods of remission.” Ganong. Review of Medical Physiology 23rd ed. Chapter 4, page 82.

ANSWER: B 9. The distance from between one stimulating electrode to recording electrode is 4.5 cm. When the axon is If the duration of the latent period and the distance between the stimulated, the latent period is 1.5 ms. What is the stimulating and recording electrodes are known, axonal conduction conduction velocity of the axon? velocity can be calculated. A. 15 m/s Convert the units: 4.5cm = 0.045m ; 1.5ms = 0.0015s B. 30 m/s Conduction Velocity = Distance/Latent period C. 40 m/s Conduction Velocity = 0.045m/0.0015s = 30 m/s D. 67.5 m/s Ganong. Review of Medical Physiology 23rd ed. Chapter , page . E. This cannot be determined from the information given. •

• • •

10. Which part of a neuron has the highest concentration of Na+ channels per square millimeter of cell membrane? A. dendrites B. cell body near dendrites C. initial segment D. axonal membrane under myelin E. none of the above

ANSWER: C •

“The number of Na+ channels per square micrometer of membrane in myelinated mammalian neurons has been estimated to be 50 –75 in the cell body , 350–500 in the initial segment , less than 25 on the  surface of the myelin, 2000–12,000 at the nodes of Ranvier , and 20–75 at the axon terminals.” Ganong. Review of Medical Physiology 23rd ed. Chapter 4, page 85.

TOPNOTCH MEDICAL BOARD PREP MED PHYSIOLOGY GANONG SUPPLEMENT HANDOUT BY THE TOPNOTCH TEAM Page 5 of 46 For inquiries visit www.topnotchboardprep.com.ph or https://web.facebook.com/topnotchmedicalboardprep/

TOPNOTCH MEDICAL BOARD PREP PHYSIOLOGY GANONG SUPPLEMENT HANDOUT BY THE TOPNOTCH TEAM For inquiries visit www.topnotchboardprep.com.ph or https://www.facebook.com/topnotchmedicalboardprep/ 11. Which of the following statements about nerve growth factor is NOT true? A. It is made up of three polypeptide subunits B. It facilitates the process of apoptosis C. It is necessary for the growth and development of the sympathetic nervous system D. It is picked up by nerves from the organs they innervate E. It is present in the brain

ANSWER: B “Nerve Growth Factor is made up of three polypeptide subunits. It is necessary for the growth and development of the  sympathetic  nervous system; and cholinergic   neurons in the basal forebrain and the striatum. There is evidence that the maintenance of neurons by NGF is due to a reduction in apoptosis. It is picked up by nerves from the organs they innervate.”



Ganong. Review of Medical Physiology 23rd ed. Chapter 4, page 90 .

(CHAPTER 5) EXCITABLE TISSUE: MUSCLE 1. The action potential of skeletal muscle: A. has a prolonged plateau phase. B. spreads inward to all parts of the muscle via the T tubules. C. causes the immediate uptake of Ca2+ into the lateral sacs of the sarcoplasmic reticulum. D. is longer than the action potential of cardiac muscle. E. is not essential for contraction. 2. The functions of tropomyosin in skeletal muscle include: A. sliding on actin to produce shortening. B. releasing Ca2+ after initiation of contraction. C. binding to myosin during contraction. D. acting as a “relaxing protein” at rest by covering up the sites where myosin binds to actin. E. generating ATP, which it passes to the contractile mechanism.

ANSWER: B •

Action potential is essential   for contraction which spreads inward to all parts of the muscle via the T- tubules; it triggers the release of Ca2+ from the terminal cisterns, the lateral sacs of the sarcoplasmic reticulum next to the T system; it is  shorter   than the action potential of cardiac muscle (which has prolonged plateau phase). Ganong. Review of Medical Physiology 23rd ed. Chapter 5 , page 100 .

ANSWER: D Tropomyosin: acting as a “relaxing protein” at rest by covering up the sites where myosin binds to actin. Myosin: sliding on actin to produce shortening.  Sarcoplasmic reticulum: releasing Ca2+ after initiation of contraction.  Actin: binding to myosin during contraction. Phosphocreatine: generating ATP, which it passes to the contractile mechanism. •

• •



Ganong. Review of Medical Physiology 23rd ed. Chapter 5; pages 96, 102 .

ANSWER: B 3. The cross-bridges of the sarcomere in skeletal muscle are made up of A. actin. B. myosin. C. troponin. D. tropomyosin. E. myelin.

Tropomyosin: acting as a “relaxing protein” at rest by covering up the sites where myosin binds to actin. Myosin: sliding on actin to produce shortening/ cross-bridging.  Actin: binding to myosin head during contraction. Troponin T binds the troponin components to tropomyosin; Troponin I   inhibits the interaction of myosin with actin; and Troponin C contains the binding sites for the Ca2+ that helps to initiate contraction. Myelin covers the nerve fibers.



• • • • •

Ganong. Review of Medical Physiology 23rd ed. Chapter 2, page 51..

4. The contractile response in skeletal muscle A. starts after the action potential is over. B. does not last as long as the action potential. C. produces more tension when the muscle contracts isometrically than when the muscle contracts isotonically. D. produces more work when the muscle contracts isometrically than when the muscle contracts isotonically. E. decreases in magnitude with repeated stimulation.

ANSWER: C Muscle fiber does not have a refractory period, repeated stimulation before relaxation produces additional activation of the contractile elements, or summation of contractions. Note that because work is the product of force times distance, isotonic contractions do work , whereas isometric contractions do not. When the muscle fiber contracts isometrically , the tension developed is proportional to the number of cross-bridges between the actin and the myosin molecules Contraction starts before the action potential is over and last longer as the action potential. •







Ganong. Review of Medical Physiology 23rd ed. Chapter 5 , page 101 . 5. Gap junctions A. are absent in cardiac muscle. B. are present but of little functional importance in cardiac muscle. C. are present and provide the pathway for rapid spread of excitation from one cardiac muscle fiber to another. D. are absent in smooth muscle. E. connect the sarcotubular system to individual skeletal muscle cells

ANSWER: C •

“Gap junctions are present in cardiac muscles and provide the pathway for rapid spread of excitation from one cardiac muscle fiber to another. It is also present in smooth muscle, which is responsible for its unitary movement; on the other hand, it is absent in skeletal muscle cells. Ganong. Review of Medical Physiology 23rd ed. Chapter 5; pages 106,109 .

TOPNOTCH MEDICAL BOARD PREP MED PHYSIOLOGY GANONG SUPPLEMENT HANDOUT BY THE TOPNOTCH TEAM Page 6 of 46 For inquiries visit www.topnotchboardprep.com.ph or https://web.facebook.com/topnotchmedicalboardprep/

TOPNOTCH MEDICAL BOARD PREP PHYSIOLOGY GANONG SUPPLEMENT HANDOUT BY THE TOPNOTCH TEAM For inquiries visit www.topnotchboardprep.com.ph or https://www.facebook.com/topnotchmedicalboardprep/ (CHAPTER 6) SYNAPTIC & JUNCTIONAL TRANSMISSION 1. Which of the following electrophysiologic events is correctly paired with the change in ionic currents causing the event? A. Fast inhibitory postsynaptic potentials (IPSPs) and closing of Cl– channels B. Fast excitatory postsynaptic potentials (EPSPs) and an increase in Ca2+ conductance C. Endplate potential and an increase in Na+ conductance D. Presynaptic inhibition and closure of voltage-gated K+ channels E. Slow EPSPs and an increase in K+ conductance 2. Which of the following physiologic processes is not correctly paired with a structure? A. Electrical transmission : gap junction B. Negative feedback inhibition : Renshaw cell C. Synaptic vesicle docking and fusion : presynaptic nerve terminal D. Endplate potential : muscarinic cholinergic receptor E. Action potential generation : initial segment

ANSWER: C

Endplate potential : increase in Na+ conductance . Fast IPSPs (inhibitory postsynaptic potentials): opening of Cl – channels Fast EPSPs (excitatory postsynaptic potentials): an increase in Na+ conductance Presynaptic inhibition: opening of voltage-gated K+ channels  Slow EPSPs: decrease in K+ conductance

• •



• •

Ganong. Review of Medical Physiology 23rd ed. Chapter6; page 120, 128.

ANSWER: D •

“The acetylcholine diffuses to the muscle-type nicotinic acetylcholine receptors, which are concentrated at the tops of the junctional folds of the membrane of the motor end plate. Binding of acetylcholine to these receptors increases the Na+ and K+ conductance of the membrane, and the resultant influx of Na+ produces a depolarizing potential, the end plate potential.” Ganong. Review of Medical Physiology 23rd ed. Chapter 6, page 124 ..

ANSWER: E 3. Initiation of an action potential in skeletal muscle A. requires spatial facilitation. B. requires temporal facilitation. C. is inhibited by a high concentration of Ca2+ at the neuromuscular junction. D. requires the release of norepinephrine. E. requires the release of acetylcholine.



Temporal and spatial summation (not facilitation ) are factors which causes excitatory post synaptic potential ( EPSP ) that is different from action potential, because it is not an all-or-none response. Initiation of action potential in skeletal muscle is caused by the release of acetylcholine (not norepinephrine) from the presynaptic vesicles into the neuromuscular junction which eventually binds with nicotinic cholinergic receptor. Ganong. Review of Medical Physiology 23rd ed. Chapter 6, page 120, 124

4. A 35-year-old woman sees her physician to report muscle weakness in the extraocular eye muscles and muscles of the extremities. She states that she feels fine when she gets up in the morning, but the weakness begins soon after she becomes active. The weakness is improved by rest. Sensation appears normal. The physician treats her with an anticholinesterase inhibitor, and she notes immediate return of muscle strength. Her physician diagnoses her with A. Lambert –Eaton syndrome. B. myasthenia gravis. C. multiple sclerosis. D. Parkinson disease. E. muscular dystrophy.

ANSWER: B

5. A 55-year-old woman had an autonomic neuropathy that disrupted the sympathetic nerve supply to the pupillary dilator muscle of her right eye. While having her eyes examined, the ophthalmologist placed phenylephrine in her eyes. The right eye became much more dilated than the left eye. This suggests that A. the sympathetic nerve to the right eye had regenerated. B. the parasympathetic nerve supply to the right eye remained intact and compensated for the loss of the sympathetic nerve. C. phenylephrine blocked the pupillary constrictor muscle of the right eye. D. denervation supersensitivity had developed. E. the left eye also had nerve damage and so was not responding as expected.

ANSWER: D

6. A 47-year-old woman was admitted to the hospital after experiencing nausea and vomiting for about 2 days followed by severe muscle weakness and neurologic symptoms, including ptosis and dysphagia. She indicated she had eaten at a restaurant the evening before the symptoms began. Laboratory tests were positive for Clostridium botulinum . Neurotoxins A. block the reuptake of neurotransmitters into presynaptic terminals. B. such as tetanus toxin bind reversibly to the presynaptic membrane at the neuromuscular junction. C. reach the cell body of the motor neuron by diffusion into the spinal cord. D. exert all of their adverse effects by acting centrally rather than peripherally. E. such as botulinum toxin prevent the release of acetylcholine from motor neurons due to cleavage of either synaptosome-associated proteins or vesicle-associated membrane proteins.

ANSWER: E







Myasthenia gravis is caused by the formation of circulating antibodies to the muscle type of nicotinic acetylcholine receptors this leads to the major clinical feature of the disease –muscle fatigue with sustained or repeated activity . In contrast, Lambert –Eaton Syndrome which is a condition wherein muscle weakness is caused by an autoimmune attack against one of the Ca2+ channels in the nerve at the neuromuscular junction, improves with repetitive stimulation. Among the choices, only myasthenia will have clinical improvement after administration of an anticholinesterase inhibitor. Ganong. Review of Medical Physiology 23rd ed. Chapter 6, page 126.





A good example of denervation supersensitivity  is the response of the denervated iris. Hypersensitivity of the postsynaptic structure to the transmitter previously secreted by the axon endings is a general phenomenon, largely due to the  synthesis or activation of more receptors. Ganong. Review of Medical Physiology 23rd ed. Chapter 6, page 126.



Clinically, tetanus toxin causes spastic paralysis by blocking presynaptic transmitter release in the CNS, and botulism causes flaccid paralysis by blocking the release of acetylcholine at the neuromuscular junction. Ganong. Review of Medical Physiology 23rd ed. Chapter 6, page 119 .

TOPNOTCH MEDICAL BOARD PREP MED PHYSIOLOGY GANONG SUPPLEMENT HANDOUT BY THE TOPNOTCH TEAM Page 7 of 46 For inquiries visit www.topnotchboardprep.com.ph or https://web.facebook.com/topnotchmedicalboardprep/

TOPNOTCH MEDICAL BOARD PREP PHYSIOLOGY GANONG SUPPLEMENT HANDOUT BY THE TOPNOTCH TEAM For inquiries visit www.topnotchboardprep.com.ph or https://www.facebook.com/topnotchmedicalboardprep/ 7. Fast inhibitory postsynaptic potentials (IPSPs): A. are a consequence of decreased Cl- conductance B. occur in skeletal muscle C. can be produced by an increase in Na+ conductance D. can be produced by a decrease in Ca2+ conductance E. interact with other fast and slow potentials to move the membrane potential of the postsynaptic neuron toward or away from the firing level 8. Fast excitatory postsynaptic potentials (EPSPs): A. are a consequence of decreased Cl- conductance B. occur in skeletal muscle C. can be produced by an increase in Na+ conductance D. can be produced by a decrease in Ca2+ conductance E. All of the above

ANSWER: E Fast inhibitory postsynaptic potentials (IPSPs) can be produced by a localized increase in Cl- transport and by opening of K+ channels, with movement of K+ out of the postsynaptic cell, or by closure of Na+ or Ca2+ channels. It occurs in the motor neuron presynaptic ending and interact with other fast and slow potentials to move the membrane potential of the postsynaptic neuron toward or away from the firing level. •

Ganong. Review of Medical Physiology 23rd ed. Chapter 6, page 119, 120.

ANSWER: C •

The EPSP  is produced by depolarization of the postsynaptic cell membrane immediately under the presynaptic ending. The excitatory transmitter opens Na+ or Ca2+ ion channels in the postsynaptic membrane, producing an inward current. Ganong. Review of Medical Physiology 23rd ed. Chapter 6, page 120.

(CHAPTER 7) NEUROTRANSMITTERS & NEUROMODULATORS 1. Which of the following statements about neurotransmitters is true? A. All neurotransmitters are derived from amino acid precursors. B. Small-molecule neurotransmitters include dopamine, histamine, ATP, glycine, enkephalin, and norepinephrine. C. Large-molecule transmitters include ATP, cannabinoids, substance P, and vasopressin. D. Norepinephrine can act as a neurotransmitter in the periphery and a neuromodulator in the CNS. E. Nitrous oxide is a neurotransmitter in the CNS. 2. Which of the following statements is not true? A. Neuronal glutamate is synthesized in glia by the enzymatic conversion from glutamine and then diff uses into the neuronal terminal where it is sequestered into vesicles until released by an influx of Ca2+ into the cytoplasm after an action potential reaches the nerve terminal. B. After release of serotonin into the synaptic cleft, its actions are terminated by reuptake into the presynaptic nerve terminal, an action that can be blocked by tricyclic antidepressants. C. Norepinephrine is the only small-molecule transmitter that is synthesized in synaptic vesicles instead of being transported into the vesicle after its synthesis. D. Each nicotinic cholinergic receptor is made up of five subunits that form a central channel that, when the receptor is activated, permits the passage of Na+ and other cations. E. GABA transaminase converts glutamate to GABA; the vesicular GABA transporter transports both GABA and glycine into synaptic vesicles.

ANSWER: D •



“ Small-molecule transmitters include monoamines (eg, acetylcholine, serotonin, histamine), catecholamines (dopamine, norepinephrine, and epinephrine), and amino acids (eg, glutamate, GABA, glycine). Large-molecule transmitters include a large number of peptides called neuropeptides including substance P, enkephalin, vasopressin.” Norepinephrine can act as a neurotransmitter in the periphery and a neuromodulator in the CNS. Ganong. Review of Medical Physiology 23rd ed. Chapter 7, page 130 .

ANSWER: A

“Glutamate is formed by reductive amination of the Krebs cycle intermediate a-ketoglutarate in the cytoplasm. Glutamate released into the synaptic cleft is taken up by a Na+-dependent glutamate transporter , and in the astrocyte it is converted to glutamine. The  glutamine enters the neuron and is converted to glutamate.”



Ganong. Review of Medical Physiology 23rd ed. Chapter 7, page 140 .

ANSWER: C 3. Which of the following receptors is correctly identified as an ionotropic or a G-protein–coupled receptor (GPCR)? A. Neurokinin receptor: ionotropic B. Nicotinic receptor: GPCR C. GABA A receptor: ionotropic D. NMDA receptor: GPCR E. Glycine: GPCR

• • • • • •

GABA A receptor: ionotropic (chloride) GABA B receptor: metabotropic/GPCR Neurokinin receptor: metabotropic/GPCR Nicotinic receptor: ionotropic (sodium/potassium) NMDA receptor: ionotropic (calcium) Glycine: ionotropic (chloride) Ganong. Review of Medical Physiology 23rd ed. Chap 2 , pg 54; chap 7, pg 141.

ANSWER: D Opioid receptors (δ, μ, κ) differ in physiologic effects, all three are G protein coupled receptors, and all inhibit adenylyl cyclase. Morphine is not an endogenous opioid. •

4. A 27-year-old man was brought to the emergency department with symptoms of opioid intoxication. He was given an intravenous dose of naloxone. Endogenous opioids A. bind to both ionotropic receptors and GPCR. B. include morphine, endorphins, and dynorphins. C. show the following order of affinity for δ receptors: dynorphins > > endorphins. D. show the following order of affinity for μ receptors: dynorphins > endorphins. E. show the following order of affinity for κ receptors: endorphins > > enkephalins.



Ganong. Review of Medical Physiology 23rd ed. Chapter 7 , page 144.

TOPNOTCH MEDICAL BOARD PREP MED PHYSIOLOGY GANONG SUPPLEMENT HANDOUT BY THE TOPNOTCH TEAM Page 8 of 46 For inquiries visit www.topnotchboardprep.com.ph or https://web.facebook.com/topnotchmedicalboardprep/

TOPNOTCH MEDICAL BOARD PREP PHYSIOLOGY GANONG SUPPLEMENT HANDOUT BY THE TOPNOTCH TEAM For inquiries visit www.topnotchboardprep.com.ph or https://www.facebook.com/topnotchmedicalboardprep/ 5. A 38-year-old woman was referred to a psychiatrist after telling her primary care physician that she had difficulty sleeping (awakening at 4 AM frequently for the past few months) and a lack of appetite causing a weight loss of over 20 lb. She also said she no longer enjoyed going out with her friends or doing volunteer service for underprivileged children. What type of drug is her doctor most likely to suggest as an initial step in her therapy? A. A serotonergic receptor antagonist B. An inhibitor of neuronal uptake of serotonin C. An inhibitor of monoamine oxidase D. An amphetamine-like drug E. A drug that causes an increase in both serotonin and dopamine

ANSWER: B

6. A 55-year-old woman had been receiving long-term treatment with phenelzine for her depression. After she consumed Chianti wine, aged cheddar cheese, processed meats, and dried fruits one night at a party, the following symptoms developed: a severe headache, chest pain, rapid heartbeat, enlarged pupils, increased sensitivity to light, and nausea. What is the most likely cause of these symptoms? A. The foods were contaminated with botulinum toxin. B. She had a myocardial infarction. C. She experienced a migraine headache. D. She had an adverse reaction to the mixture of alcohol with her antidepressant. E. She had a hypertensive crisis from eating foods high in tyramine while taking a monoamine oxidase inhibitor for her depression.

ANSWER: E

7. Which of the following is a ligand-gated ion channel? A. VIP receptor B. norepinephrine receptor C. GABA(A) receptor D. GABA(B) receptor E. Metamorphic glutamate receptor

ANSWER: C

8. Which of the following synaptic transmitters is NOT a peptide, polypeptide, or protein? A. substance P B. met-enkephalin C. Beta-endorphin D. serotonin E. dynorphin





The case demonstrated a patient experiencing signs and symptoms of depression. “Focus in treating clinical depression has shifted from norepinephrine to serotonin. It is known that the primary serotonin metabolite 5-HIAA is low in CSF of depressed individuals. Drugs such as fluoxetine (Prozac), which inhibit serotonin reuptake without affecting norepinephrine reuptake, are effective as antidepressants.” Ganong. Review of Medical Physiology 23rd ed. Chapter 7, page 137.

Patients taking MAO inhibitors should avoid tyramine-containing  foods (aged cheese, cured meats, and pickled food). Tyramine’s spectrum of action is similar to that of norepinephrine. In patients treated with MAO inhibitors —particularly inhibitors of the MAO-A isoform—this effect of tyramine may be greatly intensified, leading to marked increases in blood pressure. This occurs because of increased bioavailability of tyramine and increased neuronal stores of catecholamines.



Katzung. Basic and Clinical Pharmaclogy. 13th edition.page 199

• •

GABA A receptor: ionotropic (chloride) GABA B, VIP, norepinephrine, metamorphic glutamate receptor: metabotropic/GPCR Ganong. Review of Medical Physiology 23rd ed. Chapter 7 , page 139, 145, 141.

ANSWER: D “ Small-molecule transmitters include monoamines (eg, acetylcholine,  serotonin, histamine), catecholamines (dopamine, norepinephrine, and epinephrine), and amino acids (eg, glutamate, GABA, glycine). Large-molecule transmitters include a large number of peptides called neuropeptides including substance P, enkephalin, vasopressin.” •

Ganong. Review of Medical Physiology 23rd ed. Chapter 7, page 130 .

9. Activation of which of the following receptors would be expected to decrease anxiety? A. nicotinic cholinergic receptors B. glutamate receptors C. GABA (A) receptors D. glucocorticoid receptors E. alpha-1 adrenergic receptors

ANSWER: C

10. Which of the following receptors is coupled to a heterotrimeric G protein? A. glycine receptor B. GABA (B) receptor C. nicotinic acetylcholine receptor at myoneural junction D. 5-HT3 receptor E. ANP receptor

ANSWER: B GABA B receptor: metabotropic/GPCR GABA A receptor: ionotropic (chloride) Nicotinic receptor: ionotropic (sodium/potassium) Glycine receptor: ionotropic (chloride)  ANP and 5HT3 receptors: ionotropic

11. Which of the following would NOT be expected: A. A drug that increases the entry of arginine into neurons B. A drug that enhances tyrosine hydroxylase activity. C. A drug that enhances dopamine beta-hydroxylase activity D. A drug that inhibits monoamine oxidase E. A drug that inhibits norepinephrine reuptake.



The increase in Cl – conductance produced by GABA A receptors is potentiated by benzodiazepines, drugs that have marked antianxiety activity and are also effective muscle relaxants, anticonvulsants, and sedatives. Ganong. Review of Medical Physiology 23rd ed. Chapter 7, page 142.

• • • • •

Ganong. Review of Medical Physiology 23rd ed. Chap2 , pg 54; chap7, pg 141.

ANSWER: A The following factors increases adrenergic transmission: increase in the entry of tyrosine (not arginine) into neurons; enhancement of tyrosine hydroxylase activity, dopamine beta-hydroxylase activity; inhibition of monoamine oxidase and norepinephrine reuptake. •

Ganong. Review of Medical Physiology 23rd ed. Chapter 7 , page 138.

TOPNOTCH MEDICAL BOARD PREP MED PHYSIOLOGY GANONG SUPPLEMENT HANDOUT BY THE TOPNOTCH TEAM Page 9 of 46 For inquiries visit www.topnotchboardprep.com.ph or https://web.facebook.com/topnotchmedicalboardprep/

TOPNOTCH MEDICAL BOARD PREP PHYSIOLOGY GANONG SUPPLEMENT HANDOUT BY THE TOPNOTCH TEAM For inquiries visit www.topnotchboardprep.com.ph or https://www.facebook.com/topnotchmedicalboardprep/

SECTION 2: CENTRAL AND PERIPHERAL NEUROPHYSIOLOGY (CHAPTER 8) SOMATOSENSORY NEUROTRANSMISSION: TOUCH, PAIN, & TEMPERATURE ANSWER: D 1. A 28-year-old man was seen by a neurologist because he had “Meissner corpuscles are dendrites encapsulated in connective experienced prolonged episodes of tingling and numbness in tissue and respond to changes in texture and slow vibrations.” his right arm. He underwent a neurologic exam to evaluate his “Merkel cells are expanded dendritic endings, and they respond to sensory nervous system. Which of the following receptors is sustained pressure and touch.” correctly paired with the type of stimulus to which it is most “Ruffini corpuscles are enlarged dendritic endings with elongated apt to respond? capsules, and they respond to sustained pressure.” A. Pacinian corpuscle and motion. “Pacinian corpuscles consist of unmyelinated dendritic endings of B. Meissner corpuscle and deep pressure. a sensory nerve fiber, encapsulated by concentric lamellae of C. Merkel cells and warmth. connective tissue that give the organ the appearance of a cocktail D. Ruffini corpuscles and sustained pressure. onion. Theses receptors respond to deep pressure and fast E. Muscle spindle and tension. vibration.” •







Ganong. Review of Medical Physiology 23rd ed. Chapter 8, page 150 .

ANSWER: A 2. Nociceptors A. are activated by strong pressure, severe cold, severe heat, and chemicals. B. are absent in visceral organs. C. are specialized structures located in the skin and joints. D. are innervated by group II afferents. E. are involved in acute but not chronic pain.



Nociceptors are activated by strong pressure, severe cold, severe heat, and chemicals. Visceral organs are also innervated by nociceptor and is triggred by organ distention. They are not specialized structure, they are unmyelinated dendrites of sensory neurons located around hair follicles throughout the glabrous and hairy skin as well as deep tissue. They are innervated by  Aδ fibers and C fibers and are involved in acute and chronic pain. Ganong. Review of Medical Physiology 23rd ed. Chapter 10 , page 167.

3. A generator potential A. always leads to an action potential. B. increases in amplitude as a more intense stimulus is applied. C. is an all-or-none phenomenon. D. is unchanged when a given stimulus is applied repeatedly over time. E. All of the above.

ANSWER: B •

Generator or receptor potential resembles EPSP which increases in amplitude as a more intense stimulus is applied. It is not an allor-none phenomenon and does not always leads to an action potential. Ganong. Review of Medical Physiology 23rd ed. Chapter 8 , page 151 .

ANSWER: A 4. Sensory systems code for the following attributes of a stimulus: A. modality, location, intensity, and duration. B. threshold, receptive field, adaptation, and discrimination. C. touch, taste, hearing, and smell. D. threshold, laterality, sensation, and duration. E. sensitization, discrimination, energy, and projection.





“ All sensory systems code for four elementary attributes of a stimulus: modality, location, intensity, and duration.” “Modality is the type of energy transmitted by the stimulus. Location is the site on the body or space where the stimulus originated. Intensity is signaled by the response amplitude or frequency of action potential generation. Duration refers to the time from start to end of a response in the receptor.” Ganong. Review of Medical Physiology 23rd ed. Chapter 8 , page 152 .

ANSWER: D 5. Which of the f ollowing are correctly paired? A. Neuropathic pain and withdrawal reflex B. First pain and d ull, intense, diffuse, and unpleasant feeling C. Physiologic pain and allodynia D. Second pain and C fibers E. Nociceptive pain and nerve damage

• •

Nociceptive pain is caused by nere damage. “Painful stimulus causes a “bright,” sharp, localized sensation ( fast pain) followed by a dull, intense, diffuse, and unpleasant feeling (slow pain). Pain is frequently classified as physiologic or acute pain an pathologic or chronic pain, which includes inflammatory pain and neuropathic pain.” Ganong. Review of Medical Physiology 23rd ed. Chapter 10 , page 168 .

6. A 32-year-old woman experienced the sudden onset of a severe cramping pain in the abdominal region. She also became nauseated. Visceral pain A. shows relatively rapid adaptation. B. is mediated by B fibers in the dorsal roots of the spinal nerves. C. is poorly localized. D. resembles “fast pain” produced by noxious stimulation of the skin. E. causes relaxation of nearby skeletal muscles.

ANSWER: C

7. A ventrolateral cordotomy is performed that produces relief of pain in the right leg. It is effective because it interrupts the A. left dorsal column. B. left ventrolateral spinothalamic tract. C. right ventrolateral spinothalamic tract. D. right medial lemniscal pathway. E. a direct projection to the primary somatosensory cortex.

ANSWER: B The axons from the neurons responsible for nociception crosses the midline and ascend in the ventrolateral quadrant of the spinal cord, where they form the ventrolateral spinothalamic tract; therefore, ventrolateral cordotomy will result in loss of pain sensation in the contralateral  side.



Visceral pain is is poorly localized, and often referred, causing contraction or spasm of nearby skeletal muscles. It shows slow adaptation mediated by C fibers in the dorsal roots of the spinal nerves. Ganong. Review of Medical Physiology 23rd ed. Chapter 10 , page 169,170 .



Ganong. Review of Medical Physiology 23rd ed. Chapter 11 , page 175.

ANSWER: B 8. Which of the following CNS regions is not correctly paired with a neurotransmitter or a chemical involved in pain modulation? A. Periaqueductal gray matter and morphine B. Nucleus raphe magnus and norepinephrine C. Spinal dorsal horn and enkephalin D. Dorsal root ganglion and opioids E. Spinal dorsal horn and serotonin

• • • • • •

Nucleus raphe magnus and serotonin Locus Coeruleus and norepinephrine Periaqueductal gray matter and morphine Spinal dorsal horn and enkephalin Dorsal root ganglion and opioids Spinal dorsal horn and serotonin

Ganong. Review of Medical Physiology 23rd ed. Chap11 , pg177; Chap15, pg 238

TOPNOTCH MEDICAL BOARD PREP MED PHYSIOLOGY GANONG SUPPLEMENT HANDOUT BY THE TOPNOTCH TEAM Page 10 of 46 For inquiries visit www.topnotchboardprep.com.ph or https://web.facebook.com/topnotchmedicalboardprep/

TOPNOTCH MEDICAL BOARD PREP PHYSIOLOGY GANONG SUPPLEMENT HANDOUT BY THE TOPNOTCH TEAM For inquiries visit www.topnotchboardprep.com.ph or https://www.facebook.com/topnotchmedicalboardprep/ 9. A 47-year-old woman experienced migraine headaches that were not relieved by her current pain medications. Her doctor prescribed one of the newer analgesic agents that exert their effects by targeting synaptic transmission in nociception and peripheral sensory transduction. Which of the following drugs is correctly paired with the type of receptor it acts on to exert its antinociceptive effects? A. Topiramate and Na+ channel B. Ziconotide and NMDA receptors C. Naloxone and opioid receptors D. Lidocaine and TRPVI channels E. Gabapentin and Nav1.8

ANSWER: A

10. A 40-year-old man loses his right hand in a f arm accident. Four years later, he has episodes of severe pain in the missing hand (phantom limb pain). A detailed PET scan study of his cerebral cortex might be expected to show A. expansion of the right hand area in his right primary somatosensory cortex. B. expansion of the right hand area in his left primary somatosensory cortex. C. a metabolically inactive spot where his hand area in his left primary somatosensory cortex would normally be. D. projection of fibers from neighboring sensory areas into the right hand area of his right primary somatosensory cortex. E. projection of fibers from neighboring sensory areas into the right hand area of his left primary somatosensory cortex

ANSWER: E

Topiramate and voltage-gated Na+ channels  Ziconotide and voltage-gated N-type calcium channels Naloxone and opioid receptors (partial antagonist ) Lidocaine and sodium channels Gabapentin and VG-Ca2+ channels

• • • • •

Katzung. Basic and Clinical Pharmaclogy. 13th edition. pgs 329,540,576,730, 741,



“Numerous animal studies point to dramatic reorganization of cortical structures. If a digit is amputated in a monkey, the cortical representation of the neighboring digits spreads into the cortical area that was formerly occupied by the representationof t he amputated digit.” Ganong. Review of Medical Physiology 23rd ed. Chapter 11 page 176.

ANSWER: D •



11. A 50-year-old woman undergoes a neurologic exam that indicates loss of pain and temperature sensitivity, vibratory sense, and proprioception in the left leg. These symptoms could be explained by A. a tumor on the right medial lemniscal pathway in the sacral spinal cord. B. a peripheral neuropathy. C. a tumor on the left medial lemniscal pathway in the sacral spinal cord. D. a tumor affecting the right posterior paracentral gyrus. E. a large tumor in the right lumbar ventrolateral spinal cord.



“The dorsal column–medial lemniscal system carries signals upward to the medulla of the brain and cross to the opposite side in the medulla, they continue through the brain stem to the thalamus by way of the medial lemniscus.” “Conversely, signals in the anterolateral system, immediately after entering the spinal cord, synapse in the dorsal horns of the spinal gray matter, then cross to the opposite side of the cord and ascend through the anterior and lateral white columns of the cord. They terminate at all levels of the lower brain stem and in the thalamus.” “Thalamic sensory relay area, the third -order nerve fibers project mainly to the postcentral gyrus of the cerebral cortex , which is called somatic sensory area I. “

Guyton.Textbook of Medical Physiology 11th ed.Chapter 47 , page 587, 588 .

ANSWER: C 12. The distance by which two touch stimuli must be separated to be perceived as two separate stimuli is greatest on the A. lips B. palm of the hand C. back of the scapula D. dorsum of the hand E. tips of the f ingers



“Two point tactile discrimination on the tips of the fingers, a person can distinguish two separate points even when the needles are as close together as 1 to 2 millimeters. However, on the person’s back , the needles must usually be as far apart as 30 to 70 millimeters before two separate points can be detected. The reason for this difference is the different numbers of specialized tactile receptors in the two areas.”

Guyton.Textbook of Medical Physiology 11th ed.Chapter 47 , page 592, 593.

13. Visceral pain A. shows relatively rapid adaptation B. is mediated by B fibers in the dorsal roots of the spinal nerves C. is poorly localized D. resembles “fast pain” produced by noxious stimulation of the skin E. causes relaxation of nearby skeletal muscles

ANSWER: C •

Visceral pain is is poorly localized, and often referred, causing contraction or spasm of nearby skeletal muscles. It shows slow adaptation mediated by C fibers in the dorsal roots of the spinal nerves. Ganong. Review of Medical Physiology 23rd ed. Chapter 10 , page 169,170 .

ANSWER: D •

14. Thermoreceptors A. are activated only by severe cold or severe heat B. are located on superficial layers of the skin C. are a subtype of nociceptors D. are on dendritic endings of Aδ fibers and C fibers E. All of the above

• •

Nociceptors and thermoreceptors are free nerve endings on unmyelinated or lightly myelinated fibers in hairy and glaborous skin and deep tissues. They are innervated by  Aδ fibers and C fibers. Thermoreceptor is not a subtype of nociceptor and unlike nociceptor which is activated only in severe temperature condition, thermoreceptor threshold for hot and cold temperature is 30 and 24 degrees celsius respectively

Ganong. Review of Medical Physiology 23rd ed. Chapter 10 , page 167, 168 .

TOPNOTCH MEDICAL BOARD PREP MED PHYSIOLOGY GANONG SUPPLEMENT HANDOUT BY THE TOPNOTCH TEAM Page 11 of 46 For inquiries visit www.topnotchboardprep.com.ph or https://web.facebook.com/topnotchmedicalboardprep/

TOPNOTCH MEDICAL BOARD PREP PHYSIOLOGY GANONG SUPPLEMENT HANDOUT BY THE TOPNOTCH TEAM For inquiries visit www.topnotchboardprep.com.ph or https://www.facebook.com/topnotchmedicalboardprep/ (CHAPTER 9) VISION 1. A visual exam in an 80-year-old man shows he has a reduced ability to see objects in the upper and lower quadrants of the left visual fields of both eyes but some vision remains in the central regions of the visual field. The diagnosis is A. central scotoma. B. heteronymous hemianopia with macular sparing. C. lesion of the optic chiasm. D. homonymous hemianopia with macular sparing. E. retinopathy.

ANSWER: D Central scotoma: blind spots in the visual field Heteronymous hemianopia with macular sparing: reduced ability to see objects in the upper and lower quadrants of opposite sides of the visual fields but some vision remains in the central regions of the visual field Lesion of the optic chiasm: bitemporal visual loss. Homonymous hemianopia with macular sparing: reduced ability to see objects in the upper and lower quadrants of the left visual fields of both eyes but some vision remains in the central regions of the visual field Retinopathy: scarring of the retina that can lead to total loss of vision. • •

• •



Ganong. Review of Medical Physiology 23rd ed. Chapter 12 , page 198 .

2. A 45-year-old woman who had never needed to wear glasses experienced difficulty reading a menu in a dimlylit restaurant. She then recalled that as of late she needed to have the newspaper closer to her eyes in order to read it. A friend recommended she purchase reading glasses. Visual accommodation involves A. increased tension on the lens ligaments. B. a decrease in the curvature of the lens. C. relaxation of the sphincter muscle of the iris. D. contraction of the ciliary muscle. E. increased intraocular pressure. 3. A 28-year-old man with severe myopia made an appointment to his ophthalmologist when he began to notice flashing lights and floaters in his visual field. He was diagnosed with a retinal detachment. The retina A. is epithelial tissue that contains photoreceptors. B. lines the anterior one-third of the choroid. C. has an inner nuclear layer that contains bipolar cells, horizontal cells, andsee amacrine cells. D. contains ganglion cells whose axons form the oculomotor nerve. E. contains an optic disk where visual acuity is greatest 4. A 62-year-old white woman experienced a rapid onset of blurry vision along with loss of central vision. A comprehensive eye exam showed that she had wet agerelated macular degeneration. The fovea of the eye A. has the lowest light threshold. B. is the region of highest visual acuity. C. contains only red and green cones. D. contains only rods. E. is situated over the head of the optic nerve.

ANSWER: D

 Accommodation is when the gaze is directed at a near object , the ciliary muscle contracts. This decreases the distance between the edges of the ciliary body and relaxes the lens ligaments, so that the lens springs into a more convex shape.



Ganong. Review of Medical Physiology 23rd ed. Chapter 12, page 189

ANSWER: C •



“The retina extends anteriorly almost to the ciliary body. It is organized in 10 layers and contains the rods and cones, which are the visual receptors, plus four types of neurons: bipolar cells, ganglion cells, horizontal cells, and amacrine cells.” “It contains ganglion cells whose axons form the optic nerve. The area of the optic disk contains no visual receptors over the disk, and consequently this spot is blind (the blind spot ).” Ganong. Review of Medical Physiology 23rd ed. Chapter 12 , page 182..

ANSWER: B •

“Near the posterior pole of the eye is a yellowish pigmented spot, the macula lutea. This marks the location of the  fovea centralis, a thinnedout, rod-free portion of the retina that is present in humans and other primates. In it, the cones are densely packed, and each synapses to a single bipolar cell, which, in turn, synapses on a single ganglion cell, providing a direct pathway to the brain. Consequently, the fovea is the point where visual acuity is greatest .” Ganong. Review of Medical Physiology 23rd ed. Chapter 12 , page 183,184.

ANSWER: E In the extrafoveal portions of the retina, rods predominate (Figure 12-7), and there is a good deal of convergence. •

5. Which of the following parts of the eye has the greatest concentration of rods? A. Ciliary body B. Iris C. Optic disk D. Fovea E. Parafoveal region

Ganong. Review of Medical Physiology 23rd ed. Chapter 12 , page 185, 186 .

6. Which of the following is not correctly paired? A. Rhodopsin: retinal and opsin B. Obstruction of the canal of Schlemm: elevated intraocular pressure C. Myopia: convex lenses D. Astigmatism: nonuniform curvature of the cornea E. Inner segments of rods and cones: synthesis of the photosensitive compounds

ANSWER: C



“Myopia is a defect that can be corrected by glasses with biconcave lenses, which make parallel light rays diverge slightly before they strike the eye.” Ganong. Review of Medical Physiology 23rd ed. Chapter 12, page 188.

TOPNOTCH MEDICAL BOARD PREP MED PHYSIOLOGY GANONG SUPPLEMENT HANDOUT BY THE TOPNOTCH TEAM Page 12 of 46 For inquiries visit www.topnotchboardprep.com.ph or https://web.facebook.com/topnotchmedicalboardprep/

TOPNOTCH MEDICAL BOARD PREP PHYSIOLOGY GANONG SUPPLEMENT HANDOUT BY THE TOPNOTCH TEAM For inquiries visit www.topnotchboardprep.com.ph or https://www.facebook.com/topnotchmedicalboardprep/ ANSWER: D 7. The correct sequence of events involved in phototransduction in rods and cones in response to light is: A. activation of transducin, decreased release of glutamate, structural changes in rhodopsin, closure of Na+ channels, and decrease in intracellular cGMP. B. decreased release of glutamate, activation of transducin, closure of Na+ channels, decrease in intracellular cGMP, and structural changes in rhodopsin. C. structural changes in rhodopsin, decrease in intracellular cGMP, decreased release of glutamate, closure of Na+ channels, and activation of transducin. D. structural changes in rhodopsin, activation of transducin, decrease in intracellular cGMP, closure of Na+ channels, and decreased release of glutamate. E. activation of transducin, structural changes in rhodopsin, closure of Na+ channels, decrease in intracellular cGMP, and decreased release of glutamate. Ganong. Review of Medical Physiology 23rd ed. Chapter 12 , page 192 .

8. A 25-year-old medical student spent a summer volunteering ANSWER: B in the sub-Saharan region of Africa. There he noted a high incidence of people reporting difficulty with night vision due “In view of the importance of vitamin A in the synthesis of to a lack of vitamin A in their diet. Vitamin A is a precursor retinene1/retinal, it is not surprising that a deficiency in this vitamin for the synthesis of produces visual abnormalities. Among these, one of the earliest to A. rods and cones. appear is night blindness (nyctalopia).” B. retinal. C. rod transducin. Ganong. Review of Medical Physiology 23rd ed. Chapter 12 , page 191. D. opsin. E. cone transducin. 9. An 11-year-old boy was having difficulty reading the graphs ANSWER: D that his teacher was showing at the front of classroom. His teacher recommended he be seen by an ophthalmologist. Not only was he asked to look at a Snellen letter chart for visual acuity but he was also asked to identify numbers in “Color blindness are inherited as recessive and X-linked an Ishihara chart. He responded that he merely saw a bunch characteristics. Color blindness is present in males if the X of dots. Abnormal color vision is 20 times more common in chromosome has the abnormal gene. Females show a defect only males than females because most cases are caused by an when both X chromosomes contain the abnormal gene.” abnormal A. dominant gene on the Y chromosome. Ganong. Review of Medical Physiology 23rd ed. Chapter 12 , page 196.. B. recessive gene on the Y chromosome. C. dominant gene on the X chromosome. D. recessive gene on the X chromosome. E. recessive gene on chromosome 22. ANSWER: D Area V3A of visual cortex is responsible for motion and not for color vision. Table: Functions of Visual Projection Areas in the Human Brain 10. Which of the following is not involved in color vision? A. Activation of a pathway that signals differences between S cone responses and the sum of L and M cone responses B. Geniculate layers 3–6 C. P pathway D. Area V3A of visual cortex E. Area V8 of visual cortex •





Ganong. Review of Medical Physiology 23rd ed. Chapter 12, page 196 ..

11. A tumor was diagnosed near the base of the skull in a 56year-old woman, impinging on her optic tract. Which of the following statements about the central visual pathway is correct? A. The fibers from each temporal hemiretina decussate in the optic chiasm, so that the fibers in the optic tracts are those from the temporal half of one retina and the nasal half of the other. B. In the geniculate body, the fibers from the nasal half of one retina and the temporal half of the other synapse on the cells whose axons form the geniculocalcarine tract C. Layers 2 and 3 of the visual cortex contain clusters of cells called globs that contain a high concentration of cytochrome oxidase. D. Complex cells have a preferred orientation of a linear stimulus and, compared to simple cells, are more dependent on the location of the stimulus within the visual field. E. The visual cortex is arranged in horizontal columns that are concerned with orientation.

ANSWER: B



“ The visual pathway is from the rods and cones to bipolar cells to ganglion cells then via the optic tract to the thalamic lateral geniculate body to the occipital lobe of the cerebral cortex. The fibers from each nasal hemiretina decussate in the optic chiasm; the fibers from the nasal half of one retina and the temporal half of the other synapse on the cells whose axons form the  geniculocalcarine tract .” Ganong. Review of Medical Physiology 23rd ed. Chapter , page .

TOPNOTCH MEDICAL BOARD PREP MED PHYSIOLOGY GANONG SUPPLEMENT HANDOUT BY THE TOPNOTCH TEAM Page 13 of 46 For inquiries visit www.topnotchboardprep.com.ph or https://web.facebook.com/topnotchmedicalboardprep/

TOPNOTCH MEDICAL BOARD PREP PHYSIOLOGY GANONG SUPPLEMENT HANDOUT BY THE TOPNOTCH TEAM For inquiries visit www.topnotchboardprep.com.ph or https://www.facebook.com/topnotchmedicalboardprep/ 12. Visual accommodation involves A. Increased tension on the lens ligaments B. a decrease in the curvature of the lens C. relaxation of the sphincter muscle of the iris D. contraction of the ciliary muscle E. increased intraocular pressure 13. The fovea of the eye: A. has the lowest light threshold B. is the region of highest visual acuity C. contains only red and green cones D. contains only rods E. is situated over the head of the optic nerve

ANSWER: D  Accommodation is when the gaze is directed at a near object , the ciliary muscle contracts. This decreases the distance between the edges of the ciliary body and relaxes the lens ligaments, so that the lens springs into a more convex shape. •

Ganong. Review of Medical Physiology 23rd ed. Chapter 12, page 189 .

ANSWER: B “Near the posterior pole of the eye is a yellowish pigmented spot, the macula lutea. This marks the location of the  fovea centralis, a thinned-out, rod-free portion of the retina that is present in humans and other primates. In it, the cones are densely packed, and each synapses to a single bipolar cell, which, in turn, synapses on a single ganglion cell, providing a direct pathway to the brain. Consequently, the fovea is the point where visual acuity is greatest .” •

Ganong. Review of Medical Physiology 23rd ed. Chapter 12 , page 183,184 .

14. Vitamin A is a precursor for the synthesis of: A. Somatostatin B. Retinene1 C. The pigment of the iris D. Scotopsin E. Aqueous humor

ANSWER: B “In view of the importance of vitamin A in the synthesis of retinene1/retinal, it is not surprising that a deficiency in this vitamin produces visual abnormalities. Among these, one of the earliest to appear is night blindness (nyctalopia).” •

Ganong. Review of Medical Physiology 23rd ed. Chapter 12 , page 191.

ANSWER: D Area V3A of visual cortex is responsible for motion and not for color vision. Table: Functions of Visual Projection Areas in the Human Brain •

15. Which of the following is not involved in color vision? A. Activation of a pathway that signals differences between S cone responses and the sum of L and M cone responses B. Geniculate layers 3–6 C. P pathway D. Area V3A of visual cortex E. Area V8 of visual cortex

Ganong. Review of Medical Physiology 23rd ed. Chapter 12, page 196 ..

(CHAPTER 10) HEARING & EQUILIBRIUM 1. A 45-year-old woman visited her clinician after experiencing ANSWER: A sudden onset of vertigo, tinnitus & hearing loss in her left ear, nausea, and vomiting. This was the second episode in the past few months. She was referred to an otolaryngologist to rule out Ménière disease. Which of the following  Sound waves are funneled through the external ear to the external statements correctly describe the functions of the external, auditory meatus and then they pass inward to the tympanic middle, or inner ear? membrane. A. Sound waves are funneled through the external ear to The cochlea of the inner ear contains receptors for hearing, otolith the external auditory meatus and then they pass inward organs contain receptors that respond to head tilt , and the to the tympanic membrane.  semicircular canals contain receptors that respond to rotation. B. The cochlea of the inner ear contains receptors for Contraction of the tensor tympani and stapedius muscles of the hearing, semicircular canals contain receptors that middle ear cause the manubrium of the malleus to be pulled inward respond to head tilt, and the otolith organs contain and the footplate of the  stapes to be pulled outward. receptors that respond to rotation. Sound waves are transformed by the eardrum and auditory ossicles C. Contraction of the tensor tympani and stapedius into movements of the footplate of the stapes. muscles of the middle ear cause the manubrium of the E. The semicircular canals, the utricle, and the  saccule of the inner malleus to be pulled outward and the footplate of the ear are concerned with equilibrium. stapes to be pulled inward. D. Sound waves are transformed by the eardrum & auditor Ganong. Review of Medical Physiology 23rd ed. Chapter 13. page 211-214 ossicles into movements of the footplate of the malleus. E. The semicircular canals, the utricle, and the saccule of the middle ear are concerned with equilibrium. •









2. A 45-year-old man with testicular cancer underwent chemotherapy treatment with cisplatin. He reported several adverse side effects including changes in taste, numbness and tingling in his fingertips, and reduced sound clarity. When the damage to the outer hair cells is greater than the damage to the inner hair cells, A. perception of vertical acceleration is disrupted. B. K+ concentration in endolymph is decreased. C. K+ concentration in perilymph is decreased. D. there is severe hearing loss. E. affected hair cells fail to shorten when exposed to sound

ANSWER: E



“The outer hair cells respond to sound, like the inner hair cells, but depolarization makes them shorten and hyperpolarization makes them lengthen. This action somehow increases the amplitude and clarity of sounds.” Ganong. Review of Medical Physiology 23rd ed. Chapter 13, page 211

TOPNOTCH MEDICAL BOARD PREP MED PHYSIOLOGY GANONG SUPPLEMENT HANDOUT BY THE TOPNOTCH TEAM Page 14 of 46 For inquiries visit www.topnotchboardprep.com.ph or https://web.facebook.com/topnotchmedicalboardprep/

TOPNOTCH MEDICAL BOARD PREP PHYSIOLOGY GANONG SUPPLEMENT HANDOUT BY THE TOPNOTCH TEAM For inquiries visit www.topnotchboardprep.com.ph or https://www.facebook.com/topnotchmedicalboardprep/ 3. Which of the following statements is correct? A. The motor protein for inner hair cells is prestin. B. The auditory ossicles function as a lever system to convert the resonant vibrations of the tympanic membrane into movements of the stapes against the endolymph-filled scala tympani. C. The loudness of a sound is directly correlated with the amplitude of a sound wave, and pitch is inversely correlated with the frequency of the sound wave. D. Conduction of sound waves to the fluid of the inner ear via the tympanic membrane and the auditory ossicles is called bone conduction. E. High-pitched sounds generate waves that reach maximum height near the base of the cochlea; lowpitched sounds generate waves that peak near the apex.

ANSWER: E

4. A 40-year-old man, employed as a road construction worker for nearly 20 years, went to his clinician to report that he recently began to notice difficulty hearing during normal conversations. A Weber test showed that sound from a vibrating tuning fork was localized to the right ear. A Schwabach test showed that bone conduction was below normal. A Rinne test showed that both air and bone conduction were abnormal, but air conduction lasted longer than bone conduction. The diagnosis was A. sensorial deafness in both ears. B. conduction deafness in the right ear. C. sensorial deafness in the right ear. D. conduction deafness in the left ear. E. sensorineural deafness in the left ear.

ANSWER: E



• •





High-pitched sounds generate waves that reach maximum height near the base of the cochlea ; low-pitched sounds generate waves that peak near the apex. The motor protein for outer hair cells is  prestin. The auditory ossicles function as a lever system to convert the resonant vibrations of the tympanic membrane into movements of the stapes against the endolymph-filled scala media. The loudness of a sound is directly correlated with the amplitude of a sound wave, and  pitch is directly correlated with the  frequency  of the sound wave. Bone conduction, is the transmission of vibrations of the bones of the skull to the fluid of the inner ear Ganong. Review of Medical Physiology 23rd ed. Chapter 13 , page 210, 211.

Ganong. Review of Medical Physiology 23rd ed. Chapter 13, page 214.

ANSWER: B

5. What would the diagnosis be if a patient had the following test results? Weber test showed that sound from a vibrating tuning fork was louder than normal; Schwabach test showed that bone conduction was better than normal; and Rinne test showed that air conduction did not outlast bone conduction. A. Sensorial deafness in both ears B. Conduction deafness in both ears C. Normal hearing D. Both sensorial and conduction deafness E. A possible tumor on the eighth cranial nerve Ganong. Review of Medical Physiology 23rd ed. Chapter 13, page 214.

6. The auditory pathway A. and vestibular pathway contain a synapse in the cerebellum. B. and vestibular pathway project to the same regions of the cerebral cortex. C. is composed of afferent fibers of the eighth cranial nerve, the dorsal and ventral cochlear nuclei, the superior colliculi, the lateral geniculate body, and the auditory cortex. D. is composed of afferent fibers of the eighth cranial nerve, the dorsal and ventral cochlear nuclei, the inferior colliculi, the medial geniculate body, and the auditory cortex. E. is not subject to plasticity like the visual pathways. 7. A healthy male medical student volunteered to undergo evaluation of the function of his vestibular system for a class demonstration. The direction of his nystagmus is expected to be vertical when he is rotated A. after warm water is put in one of his ears. B. with his head tipped backward. C. after cold water is put in both of his ears. D. with his head tipped sideways. E. with his head tipped forward.

ANSWER: D



“The activity within the auditory pathway passes from the eighth cranial nerve afferent fibers to the dorsal and ventral cochlear nuclei to the inferior colliculi  to the thalamic medial geniculate body and then to the auditory cortex .” Ganong. Review of Medical Physiology 23rd ed. Chapter 13, page 216

ANSWER: D



Nystagmus is frequently horizontal  (ie, the eyes move in the horizontal plane), but it can also be vertical  (when the head is tipped sidewise during rotation) or rotatory  (when the head is tipped forward). Ganong. Review of Medical Physiology 23rd ed. Chapter 13, page 215.

TOPNOTCH MEDICAL BOARD PREP MED PHYSIOLOGY GANONG SUPPLEMENT HANDOUT BY THE TOPNOTCH TEAM Page 15 of 46 For inquiries visit www.topnotchboardprep.com.ph or https://web.facebook.com/topnotchmedicalboardprep/

TOPNOTCH MEDICAL BOARD PREP PHYSIOLOGY GANONG SUPPLEMENT HANDOUT BY THE TOPNOTCH TEAM For inquiries visit www.topnotchboardprep.com.ph or https://www.facebook.com/topnotchmedicalboardprep/ ANSWER: E

8. In the utricle, tip links in hair cells are involved in A. formation of perilymph. B. depolarization of the stria vascularis. C. movements of the basement membrane. D. perception of sound. E. regulation of distortion-activated ion channels.

Schematic representation of the role of tip links in the responses of hair cells. When a stereocilium is pushed toward a taller stereocilium, the tip line is stretched and opens an ion channel in its t aller neighbour. The channel next is presumably moved down the taller stereocilium by a molecular motor, so the tension on the tip link is released. When the hairs return to the resting position, the motor moves back up the stereocilium. Ganong. Review of Medical Physiology 23rd ed. Chapter 13, page 208

9. Postrotatory nystagmus is caused by continued movement of A. aqueous humor over the ciliary body in the eye. B. cerebrospinal fluid over the parts of the brainstem that contain the vestibular nuclei. C. endolymph in the semicircular canals, with consequent bending of the cupula and stimulation of hair cells. D. endolymph toward the helicotrema. E. perilymph over hair cells that have their processes embedded in the tectorial membrane 10. A patient enters the hospital for evaluation of deafness. He is found to also have an elevated plasma renin, although his blood pressure is 118/75 mm Hg. Mutation of what single gene may explain these findings? A. The gene for barttin B. The gene for Na+ channel C. The gene for renin D. The gene for cystic fibrosis transmembrane conductance regulator E. The gene for tyrosine hydroxylase

ANSWER: C



“The direction of the quick component during rotation is the same as that of the rotation, but the  postrotatory nystagmus that occurs owing to displacement of the cupula when rotation is stopped is in the opposite direction.” Ganong. Review of Medical Physiology 23rd ed. Chapter 13, page 215.

ANSWER: A •

“Bartter syndrome is due to defective transport in the thick ascending limb. It is characterized by chronic Na+ loss in the urine, with resultant hypovolemia causing stimulation of rennin and aldosterone secretion. The condition can be caused by loss-of-function mutations in the gene for any of four key proteins: the Na –K–2Cl cotransporter, the ROMK K+ channel,the ClC–Kb Cl– channel, or barttin.”

Ganong. Review of Medical Physiology 23rd ed. Chap 13, pg 214 ;Chap 38, pg 654.

(CHAPTER 11) SMELL & TASTE 1. A young boy was diagnosed with congenital anosmia, a rare disorder in which an individual is born without the ability to smell. Odorant receptors are A. located in the olfactory bulb. B. located on dendrites of mitral and tufted cells. C. located on neurons that project directly to the olfactory cortex. D. located on neurons in the olfactory epithelium that project to mitral cells and from there directly to the olfactory cortex. E. located on sustentacular cells that project to the olfactory bulb.

ANSWER: D



“Olfactory sensory neuron expresses only one of the 1000 different odorant receptors. Each neuron projects to one or two glomeruli which provides a distinct two-dimensional map in the olfactory bulb that is unique to the odorant. The mitral cells with their glomeruli project to different parts of the olfactory cortex.” Ganong. Review of Medical Physiology 23rd ed. Chapter 14, page 222..

ANSWER: D 2. A 37-year-old female was diagnosed with multiple sclerosis. One of the potential consequences of this disorder is diminished taste sensitivity. Taste receptors A. for sweet, sour, bitter, salt, and umami are spatially separated on the surface of the tongue. B. are synonymous with taste buds. C. are a type of chemoreceptor. D. are innervated by afferents in the facial, trigeminal, and glossopharyngeal nerves. E. All of the above.

3. Which of the following does not increase the ability to discriminate many different odors? A. Many different receptors B. Pattern of olfactory receptors activated by a given odorant C. Projection of different mitral cell axons to different parts of the brain D. High β-arrestin content in olfactory neurons E. Sniffing







Taste receptors , not synonymous with taste buds, are a type of chemoreceptor . It used to be thought that the surface of the tongue had special areas, but it is now clear that all tastants are sensed from all parts of the tongue and adjacent structures. The sensory nerve fibers from the anterior two-thirds of the tongue ( facial nerve), and those from the posterior third of the ( glossopharyngeal nerve). The fibers from areas other than the tongue (vagus nerve) Ganong. Review of Medical Physiology 23rd ed. Chapter 14, page 223-225 .

ANSWER: D “Termination of the activity of transducin in photoreception is also accelerated by its binding of β -arrestin.” therefore β -arrestin is for vision and not for olfaction. The portion of the nasal cavity containing the olfactory receptors is poorly ventilated in humans, amount of air reaching this region is greatly increased by  sniffing. Increased in number of different receptors, pattern of olfactory receptors activated by a given odorant, and projection of different mitral cell axons to different parts of the brain increases the ability to di scriminate many different odors •





Ganong. Review of Medical Physiology 23rd ed. Chap 12, page 191; Chap 14, page 223.

TOPNOTCH MEDICAL BOARD PREP MED PHYSIOLOGY GANONG SUPPLEMENT HANDOUT BY THE TOPNOTCH TEAM Page 16 of 46 For inquiries visit www.topnotchboardprep.com.ph or https://web.facebook.com/topnotchmedicalboardprep/

TOPNOTCH MEDICAL BOARD PREP PHYSIOLOGY GANONG SUPPLEMENT HANDOUT BY THE TOPNOTCH TEAM For inquiries visit www.topnotchboardprep.com.ph or https://www.facebook.com/topnotchmedicalboardprep/ 4. As a result of an automobile accident, a 10-year-old boy suffered damage to the brain including the periamygdaloid, piriform, and entorhinal cortices. Which of the following sensory deficits is he most likely to experience? A. Visual disturbance B. Hyperosmia C. Auditory problems D. Taste and odor abnormalities E. No major sensory deficits

ANSWER: D •

“The pathway to the amygdala is probably involved with the emotional responses to olfactory stimuli, and the pathway to the entorhinal cortex   is concerned with olfactory memories.” Damage along this pathway would result to odor and taste abnormalities. Ganong. Review of Medical Physiology 23rd ed. Chapter 14, page 221.

5. A 9-year-old boy had frequent episodes of uncontrollable nose bleeds. ANSWER: C At the advice of his clinician, he underwent surgery to correct a Conscious discrimination of odor depends on the neocortex problem in his nasal septum. A few days after the surgery, he told his (orbitofrontal and frontal cortices). mother he could not smell the cinnamon rolls she was baking in the Although there are millions of olfactory  sensory neurons, oven. Which of the following is true about olfactory transmission? each expresses only one of the 1000 different odorant A. An olfactory sensory neuron expresses a wide range of odorant receptors. receptors. The olfactory glomeruli demonstrate lateral inhibition B. Lateral inhibition within the olfactory glomeruli reduces the mediated by periglomerular cells and granule cells. This ability to distinguish between different types of odorant sharpens and focuses olfactory signals. receptors. Olfaction is physiologically related to gustation because of C. Conscious discrimination of odors is dependent on the pathway its use of chemoreceptors , and thru chemoreceptive to the orbitofrontal cortex. sensory interaction, they are able to perceive complex flavors. D. Olfaction is closely related to gustation because odorant and gustatory receptors use the same central pathways. Ganong. Review of Medical Physiology 23rd ed. Chapter 14, pages E. All of the above.  219,221,222. •







6. A 31-year-old female is a smoker who has had poor oral hygiene for most of her life. In the past few years she has noticed a reduced sensitivity to the flavors in various foods which she used to enjoy eating. Which of the following is not true about gustatory sensation? A. The sensory nerve fibers from the taste buds on the anterior two-thirds of the tongue travel in the chorda tympani branch of the facial nerve. B. The sensory nerve fibers from the taste buds on the posterior third of the tongue travel in the petrosal branch of the glossopharyngeal nerve. C. The pathway from taste buds on the left side of the tongue is transmitted ipsilaterally to the cerebral cortex. D. Sustentacular cells in the taste buds serve as stem cells to permit growth of new taste buds. E. The pathway from taste receptors includes synapses in the nucleus of the tractus solitarius in the brainstem and ventral posterior medial nucleus in the thalamus.

ANSWER: D



The human olfactory epithelium contains 10 to 20 million bipolar olfactory sensory neurons interspersed with glia-like supporting (sustentacular cells) and basal stem cells which permit growth of new taste buds. Ganong. Review of Medical Physiology 23rd ed. Chapter 14, page 219.

7. A 20-year-old woman was diagnosed with Bell palsy (damage to facial ANSWER: E nerve). Which of the following symptoms is she likely to exhibit? “The onset of Bell’s palsy   is fairly abrupt, with maximal A. Loss of sense of taste weakness being attained by 48 h as a general rule. Pain B. Facial twitching behind the ear may precede the paralysis for a day or two. C. Droopy eyelid Taste sensationmay be l ost unilaterally, and hyperacusis D. Ipsilateral facial paralysis may be present.” E. All of the above Harrison’s Principles of Internal Medicine 19th ed. Chap 445 , page 2648. •

(CHAPTER 12) REFLEX & VOLUNTARY CONTROL OF POSTURE & MOVEMENT 1. When dynamic γ-motor neurons are activated at the same time as α motor neurons to muscle, A. prompt inhibition of discharge in spindle Ia afferents takes place. B. clonus is likely to occur. C. the muscle will not contract. D. the number of impulses in spindle Ia afferents is smaller than when α discharge alone is increased. E. the number of impulses in spindle Ia afferents is greater than when α discharge alone is increased.

ANSWER: E •



“Activation of dynamic γ -motor neurons increases the dynamic sensitivity of the group Ia endings.” “Because of this “ α–γ  coactivation,” intrafusal and extrafusal fibers shorten together, and spindle afferent activity can occur throughout the period of muscle contraction.” Ganong. Review of Medical Physiology 23rd ed. Chapt 9, page 158, 160.

ANSWER: C

2. The inverse stretch reflex A. occurs when Ia spindle afferents are inhibited. B. is a monosynaptic reflex initiated by activation of the Golgi tendon organ. C. is a disynaptic reflex with a single interneuron inserted between the afferent and efferent limbs. D. is a polysynaptic reflex with many interneurons inserted between the afferent and efferent limbs. E. uses type II afferent fibers from the Golgi tendon organ.

Ganong. Review of Medical Physiology 23rd ed. Chap , page .

TOPNOTCH MEDICAL BOARD PREP MED PHYSIOLOGY GANONG SUPPLEMENT HANDOUT BY THE TOPNOTCH TEAM Page 17 of 46 For inquiries visit www.topnotchboardprep.com.ph or https://web.facebook.com/topnotchmedicalboardprep/

TOPNOTCH MEDICAL BOARD PREP PHYSIOLOGY GANONG SUPPLEMENT HANDOUT BY THE TOPNOTCH TEAM For inquiries visit www.topnotchboardprep.com.ph or https://www.facebook.com/topnotchmedicalboardprep/ ANSWER: E •

3. Withdrawal reflexes are not A. initiated by nociceptive stimuli. B. prepotent. C. prolonged if the stimulus is strong. D. an example of a flexor reflex. E. accompanied by the same response on both sides of the body.







“Withdrawal reflexes are  prepotent , they preempt the spinal pathways from any other reflex activity taking place at the moment.” “The withdrawal reflex is a typical polysynaptic reflex that occurs in response to a usually  painful  stimulation.” “A weak stimulus causes one quick flexion movement; a strong stimulus causes prolonged  flexion and sometimes a series of flexion movements.” “When a strong stimulus is applied to a limb, the response includes not only flexion and withdrawal of that limb but also extension of the opposite limb. This crossed extensor response is properly part of the withdrawal reflex.” Ganong. Review of Medical Physiology 23rd ed. Chapter 9, page 163-164.

4. While exercising, a 42-year-old woman developed sudden onset of tingling in her right leg and an inability to control movement in that limb. A neurologic exam showed a hyperactive knee jerk reflex and a positive Babinski sign. Which of the following is not characteristic of a reflex? A. Reflexes can be modified by impulses from various parts of the CNS B. Reflexes may involve simultaneous contraction of some muscles and relaxation of others C. Reflexes are chronically suppressed after spinal cord transection D. Reflexes involve transmission across at least one synapse E. Reflexes often occur without conscious perception

ANSWER: C





“When the spinal cord is suddenly transected all cord functions, including the cord reflexes, immediately become depressed, a reaction called  spinal shock.”  “After a few hours to a few weeks, the spinal neurons gradually regain their excitability, they increase their own natural degree of excitability to make up at least partially for the loss.” Textbook of Medical Physiology 11th ed. Guyton Chapter 54, page 684 .

ANSWER: E •

5. Increased neural activity before a skilled voluntary movement is first seen in the A. spinal motor neurons. B. precentral motor cortex. C. midbrain. D. cerebellum. E. cortical association areas.

“Commands for voluntary movement originate in cortical association areas. The cortex, basal ganglia, and cerebellum work cooperatively to plan movements. Movement executed by the cortex is relayed via the corticospinal tracts and corticobulbar tracts to motor neurons. The cerebellum provides feedback to adjust and smooth movement.”

Ganong. Review of Medical Physiology 23rd ed. Chapter 16, page 242

6. A 58-year-old woman was brought to the emergency ANSWER: B department of her local hospital because of a sudden change of consciousness. All f our limbs were extended, suggestive of decerebrate rigidity. A brain CT showed a rostral pontine hemorrhage. Which of the following describes components of the central pathway responsible for control of posture? A. The tectospinal pathway terminates on neurons in the The medial brain stem pathways, which work in concert with the dorsolateral area of the spinal ventral horn that innervate ventral corticospinal tract, are the  pontine and medullary limb muscles. reticulospinal , vestibulospinal , and tectospinal tracts. These B. The medullary reticulospinal pathway terminates on pathways descend in the ipsilateral ventral columns of the spinal neurons in the ventromedial area of the spinal ventral cord and terminate predominantly on interneurons and long horn that innervate axial and proximal muscles. propriospinal neurons in the ventromedial part of the ventral horn C. The pontine reticulospinal pathway terminates on to control axial and proximal muscles. neurons in the dorsomedial area of the spinal ventral horn that innervate limb muscles. Ganong. Review of Medical Physiology 23rd ed. Chapter 16, page 246. D. The medial vestibular pathway terminates on neurons in the dorsomedial area of the spinal ventral horn that innervate axial and proximal muscles. E. The lateral vestibular pathway terminates on neurons in the dorsolateral area of the spinal ventral horn that innervate axial and proximal muscles. 7. A 38-year-old woman with a metastatic brain tumor was ANSWER: C brought to the emergency department of her local hospital because of irregular breathing and progressive loss of consciousness. She also showed signs of decerebrate posturing. Which of the following is not true about decerebrate rigidity? “In midcollicular decerebrate cats, section of dorsal roots to a limb A. It involves hyperactivity in extensor muscles of all 4 limbs immediately eliminates the hyperactivity of extensor muscles. This B. The excitatory input from the reticulospinal pathway suggests That decerebrate rigidity is spasticity due to facilitation activates γ-motor neurons, which indirectly activate α of the myotatic stretch reflex.” motor neurons. Ganong. Review of Medical Physiology 23rd ed. Chapter 16, page 247 . C. It is actually a type of spasticity due to inhibition of the stretch reflex. D. It resembles what ensues after uncal herniation. E. Lower extremities are extended with toes pointed inward •



TOPNOTCH MEDICAL BOARD PREP MED PHYSIOLOGY GANONG SUPPLEMENT HANDOUT BY THE TOPNOTCH TEAM Page 18 of 46 For inquiries visit www.topnotchboardprep.com.ph or https://web.facebook.com/topnotchmedicalboardprep/

TOPNOTCH MEDICAL BOARD PREP PHYSIOLOGY GANONG SUPPLEMENT HANDOUT BY THE TOPNOTCH TEAM For inquiries visit www.topnotchboardprep.com.ph or https://www.facebook.com/topnotchmedicalboardprep/ ANSWER: A The subthalamic nucleus releases glutamate to excite the  globus pallidus, internal segment. The substantia nigra pars reticulata releases GABA to inhibit  the thalamus. The substantia nigra pars compacta releases dopamine to inhibit  the striatum. The striatum releases acetylcholine to inhibit  the substantia nigra pars reticulata. E. The globus pallidus, external segment releases GABA to inhibit  the subthalamic nucleus . •

8. Which of the following describes a connection between components of the basal ganglia? A. The subthalamic nucleus releases glutamate to excite the globus pallidus, internal segment. B. The substantia nigra pars reticulata releases dopamine to inhibit the striatum. C. The substantia nigra pars compacta releases dopamine to excite the globus pallidus, external segment. D. The striatum releases acetylcholine to excite the substantia nigra pars reticulata. E. The globus pallidus, external segment releases glutamate to excite the striatum.









Ganong. Review of Medical Physiology 23rd ed. Chapter 16 , page 251 , 257

9. A 60-year-old man with Parkinson disease, which was diagnosed 15 years ago, has been taking carbidopa and levodopa (Sinemet); until recently, he has been able to continue to work and help with routine jobs around the house. Now his tremor and rigidity interfere with these activities. His clinician has suggested that he undergo deep brain stimulation therapy. The therapeutic effect of L-dopa in patients with Parkinson disease eventually wears off because A. antibodies to dopamine receptors develop. B. inhibitory pathways grow into the basal ganglia from the frontal lobe. C. there is an increase in circulating α-synuclein. D. the normal action of nerve growth factor (NGF) is disrupted E. the dopaminergic neurons in the substantia nigra continue to degenerate.

ANSWER: E



Administration of L-dopa (levodopa), unlike dopamine, crosses the blood– brain barrier and helps repair the dopamine deficiency. However, the degeneration of these neurons continues and in 5 to 7 years the beneficial effects of L-dopa disappear. Ganong. Review of Medical Physiology 23rd ed. Chapter 16, page 254.

ANSWER: C Granule cells release glutamate to excite basket cells and  stellate cells. Basket cells release GABA to inhibit Purkinje cells. Climbing and mossy fiber inputs exert a strong excitatory  effect on Purkinje cells. The axons of Purkinje cells are the sole output of the cerebellar cortex, and they release GABA to inhibit  the deep cerebellar  nuclei . Golgi cells are excited  by mossy  fiber collaterals. •

• •

10. An 8-year-old girl was brought to her pediatrician because her parents noted frequent episodes of gait unsteadiness and speech difficulties. Her mother was concerned because of a family history of Friedreich ataxia. Which of the following is a correct description of connections involving cerebellar neurons? A. Basket cells release glutamate to activate Purkinje cells. B. Climbing fiber inputs exert a strong excitatory effect on Purkinje cells, and mossy fiber inputs exert a strong inhibitory effect on Purkinje cells. C. Granule cells release glutamate to excite basket cells and stellate cells. D. The axons of Purkinje cells are the sole output of the cerebellar cortex, and they release glutamate to excite the deep cerebellar nuclei. E. Golgi cells are inhibited by mossy fiber collaterals.





Ganong. Review of Medical Physiology 23rd ed. Chapter 16 , page 256-257 .

11. After falling down a flight of stairs, a young woman is f ound to have partial loss of voluntary movement on the right side of her body and loss of pain and temperature sensation on the left side below the midthoracic region. It is probable that she has a lesion A. transecting the left half of the spinal cord in the lumbar region. B. transecting the left half of the spinal cord in the upper thoracic region. C. transecting sensory and motor pathways on the right side of the pons. D. transecting the right half of the spinal cord in the upper thoracic region. E. transecting the dorsal half of the spinal cord in the upper thoracic region.

ANSWER: D



There is transection of the right half of the spinal cord in the upper thoracic region resulting in Brown– Séquard Syndrome, described as “the loss of the  spinothalamic tract  leads to contralateral loss of pain and temperature sensation beginning one or two segments below the lesion. Damage to the corticospinal tract  produces weakness and spasticity in certain muscle groups on the same side of the body.” Ganong. Review of Medical Physiology 23rd ed. Chapter 11 , page 177 .

TOPNOTCH MEDICAL BOARD PREP MED PHYSIOLOGY GANONG SUPPLEMENT HANDOUT BY THE TOPNOTCH TEAM Page 19 of 46 For inquiries visit www.topnotchboardprep.com.ph or https://web.facebook.com/topnotchmedicalboardprep/

TOPNOTCH MEDICAL BOARD PREP PHYSIOLOGY GANONG SUPPLEMENT HANDOUT BY THE TOPNOTCH TEAM For inquiries visit www.topnotchboardprep.com.ph or https://www.facebook.com/topnotchmedicalboardprep/ 12. At the age of 30, a male postal worker reported weakness in his right leg. Within a year the weakness had spread to his entire right side. A neurologic examination revealed f laccid paralysis, muscular atrophy, fasciculations, hypotonia, and hyporeflexia of muscles in the right arm and leg. Sensory and cognitive function tests were normal. Which of the following diagnosis is likely? A. A large tumor in the left primary motor cortex B. A cerebral infarct in the region of the corona radiate C. A vestibulocerebellar tumor D. Damage to the basal ganglia E. Amyotrophic lateral sclerosis

ANSWER: E •



Patient’s neurologic examination decribes a lower motor neuron problem; therefore the cerebral cortex, basal ganglia, and vestibulocerebellar can be eliminated because they are within the CNS  ( upper motor neurons ). Lower motor neurons (i.e. α -motor neurons.) are those whose axons terminate on skeletal muscles. An example of a disease that leads to lower motor neuron damage is amyotrophic lateral sclerosis (ALS) which is a selective, progressive degeneration of α -motor neurons. Ganong. Review of Medical Physiology 23rd ed. Chapter 16 , page 244.

ANSWER: C Basal ganglia is involved in the planning and programming of movement or, more broadly, in the processes by which an abstract thought is converted into voluntary action. Thalamus: sensory integration Ventral hippocampus: short-term memory Precentral motor cortex : planning voluntary movement Hypothalamus: neuroendocrine control Pontine Reticular Formation: slow-wave sleep •

13. A primary function of the basal ganglia is: A. sensory integration B. short-term memory C. planning voluntary movement D. neuroendocrine control E. slow-wave sleep

• • • • •

Ganong. Review of Medical Physiology 23rd ed. Chapter 16 , page 250.

14. Patients with transected spinal cords frequently have a negative nitrogen balance because: A. they develop hypercalcemia, and this causes dissolution of the protein in bone B. they are paralyzed below the level of the transection C. they lack afferent input that normally maintains growth hormone secretion D. they have difficulty voiding, and this causes nitrogen to accumulate in the urine in the bladder E. their corticotropin response to stress is reduced 15. Which of the following diseases is NOT known to be caused by overexpression of trinucleotide repeat? A. Alzheimer disease B. Fragile X syndrome C. Spinocerebellar ataxia, type 3 D. Huntington disease E. Friedreich ataxia

ANSWER: B



“Due to immobilization, SCI patients develop a negative nitrogen balance and catabolize large amounts of body protein.” Ganong. Review of Medical Physiology 23rd ed. Chapter 16 , page 250.

ANSWER: A  Alzheimer’s disease appears to be a metabolic neurodegenerative disease and not caused by trinucleotide repeats.



Guyton.Textbook of Medical Physiology 11th ed.Chapter 59 , page 746.

(CHAPTER 13) AUTONOMIC NERVOUS SYSTEM 1. Hypertension developed in a 26-year-old man after he began taking amphetamine to boost his energy and to suppress his appetite. Which of the following drugs would be expected to mimic the effects of increased sympathetic discharge on blood vessels? A. Phenylephrine B. Trimethaphan C. Atropine D. Reserpine E. Albuterol

2. A 35-year-old woman in whom multiple system atrophy was diagnosed had symptoms indicative of failure of sympathetic nerve activity. Which of the following statements about the sympathetic nervous system is correct? A. All postganglionic sympathetic nerves release norepinephrine from their terminals. B. Cell bodies of preganglionic sympathetic neurons are located in the intermediolateral column of the thoracic and sacral spinal cord. C. The sympathetic nervous system is required for survival. D. Acetylcholine is released from all sympathetic preganglionic nerve terminals. E. The sympathetic nervous system adjusts pupillary diameter by relaxing the pupillary constrictor muscle.

ANSWER: A Phenylephrine: Alpha1-selective agonist which causes increased vascular resistance secondary to its action in the vascular smooth muscle Trimethaphan: ganglion blocker, treatment for malignant HPN  Atropine: blocks muscarinic receptors and inhibits parasympathomimetic effects. Reserpine: Drugs that deplete the adrenergic nerve terminal of its norepinephrine stores  Albuterol: Beta-adrenoceptor agonists, caused bronchodilation. •

• •





Katzung. Pharmacology Examination & Board Review 10th Edition.Chapter 9,  page 74,75,79; Chapter 11 page 102; Chapter 20, page 184

ANSWER: D  Acetylcholine is released from all sympathetic preganglionic nerve terminals. Not all postganglionic sympathetic nerves releases norepinephrine; exemptions are neurons that innervate sweat  glands and blood vessels in some skeletal muscles , both of which releases acetylcholine. Cell bodies of  preganglionic  sympathetic  neurons are located in the intermediolateral   column of the thoracic and lumbar spinal cord. Although survival is possible without an ANS , the ability to adapt to environmental stressors is severely compromised The sympathetic   nervous system adjusts  pupillary diameter  by contracting the pupillary constrictor muscle.











Ganong. Review of Medical Physiology 23rd ed. Chapter 17 , page 266 .

TOPNOTCH MEDICAL BOARD PREP MED PHYSIOLOGY GANONG SUPPLEMENT HANDOUT BY THE TOPNOTCH TEAM Page 20 of 46 For inquiries visit www.topnotchboardprep.com.ph or https://web.facebook.com/topnotchmedicalboardprep/

TOPNOTCH MEDICAL BOARD PREP PHYSIOLOGY GANONG SUPPLEMENT HANDOUT BY THE TOPNOTCH TEAM For inquiries visit www.topnotchboardprep.com.ph or https://www.facebook.com/topnotchmedicalboardprep/ 3. A 45-year-old man had a meal containing wild mushrooms that he picked in a field earlier in the day. Within a few hours after eating, nausea, vomiting, diarrhea, urinary urgency, vasodilation, sweating, and salivation developed. Which of the following statements about the parasympathetic nervous system is correct? A. Postganglionic parasympathetic nerves release acetylcholine to activate muscarinic receptors on sweat glands B. Parasympathetic nerve activity affects only smooth muscles and glands. C. Parasympathetic nerve activity causes contraction of smooth muscles of the gastrointestinal wall and relaxation of the gastrointestinal sphincter. D. Parasympathetic nerve activity causes contraction of the radial muscle of the eye to allow accommodation for near vision. E. An increase in parasympathetic activity causes an increase in heart rate. 4. Which of the following is correctly paired? A. Sinoatrial node: Nicotinic cholinergic receptors B. Autonomic ganglia: Muscarinic cholinergic receptors C. Pilomotor smooth muscle: β2 -adrenergic receptors D. Vasculature of some skeletal muscles: Muscarinic cholinergic receptors E. Sweat glands: α2-adrenergic receptors

ANSWER: C



“In contrast, release of acetylcholine onto smooth muscle of some organs leads to contraction (eg, walls of the gastrointestinal tract) while release onto other organs leads to relaxation (eg, sphincters in the gastrointestinal tract).” Ganong. Review of Medical Physiology 23rd ed. Chapter 17, page 266 .

ANSWER: D Vasculature of  some skeletal muscles: Muscarinic cholinergic receptors  Sinoatrial node: β1-adrenergic receptors  Autonomic  ganglia: Nicotinic cholinergic receptors Pilomotor  smooth muscle: α1-adrenergic receptors  Sweat  glands: α1-adrenergic receptors •

• • • •

Ganong. Review of Medical Physiology 23rd ed. Chapter 7, page 135 . Chapter 17, page 270..

ANSWER: C Pathways that control autonomic responses. Direct  projections (solid lines) to autonomic preganglionic neurons include the hypothalamic paraventricular nucleus, parabrachial nucleus, nucleus of the solitary tract, ventrolateral medulla, and medullary raphe (not shown). Indirect  projections (dashed lines) include the cerebral cortex, amygdala, and periaqueductal grey matter. •

5. A 57-year-old man had severe hypertension that was found to result from a tumor compressing on the surface of the medulla. Which one of the following statements about pathways involved in the control of sympathetic nerve activity is correct? A. Preganglionic sympathetic nerves receive inhibitory input from the rostral ventrolateral medulla. B. The major source of excitatory input to preganglionic sympathetic nerves is the paraventricular nucleus of the hypothalamus. C. The activity of sympathetic preganglionic neurons can be affected by the activity of neurons in the amygdala. D. Unlike the activity in δ-motor neurons, sympathetic preganglionic neurons are not under any significant reflex control. E. Under resting conditions, the sympathetic nervous system is not active; it is active only during stress giving rise to the term “flight or fight” response.

Ganong. Review of Medical Physiology 23rd ed. Chapter 17, page 271

6. Diabetic autonomic neuropathy was diagnosed a few years ago in a 53-year-old woman with diabetes. She recently noted abdominal distension and a feeling of being full after eating only a small portion of food, suggesting the neuropathy had extended to her enteric nervous system to cause gastroparesis. Which of the following statements about the enteric nervous system is correct? A. The enteric nervous system is a subdivision of the parasympathetic nervous system for control of gastrointestinal function. B. The myenteric plexus is a group of motor neurons located within circular layer of muscle in a portion of the gastrointestinal tract. C. The submucosal plexus is a group of sensory neurons located between the circular muscle and the luminal mucosa of the gastrointestinal tract. D. Neurons comprising the enteric nervous system are located only in the stomach and intestine. E. The enteric nervous system can function independent of the autonomic innervation to the gastrointestinal tract. 7. Which of the following drugs would NOT be expected to increase sympathetic discharge or mimic the effects of increased sympathetic discharge? A. Prazosin B. Neostigmine C. Amphetamine D. Isoproterenol E. Methoxamine

ANSWER: C/E







The enteric nervous system is not a subdivision of the parasympathetic nervous system; in fact, it can function independent  of the autonomic innervation to the gastrointestinal tract. the myenteric plexus (Auerbach’s plexus), between the outer longitudinal and middle circular muscle layers, and the submucous plexus (Meissner’s plexus), between the middle circular layer and the mucosa. Neurons comprising the enteric nervous system are located only in the intestine. Ganong. Review of Medical Physiology 23rd ed. Chapter 26, page 448 .

ANSWER: A •



Prazosin is an alpha1-selective blocker prototype used in hypertension and benign prostatic hyperplasia . All other drugs are symphatomimetics. Katzung. Pharmacology Examination & Board Review 10th Edition. Chapter 11 page 102

TOPNOTCH MEDICAL BOARD PREP MED PHYSIOLOGY GANONG SUPPLEMENT HANDOUT BY THE TOPNOTCH TEAM Page 21 of 46 For inquiries visit www.topnotchboardprep.com.ph or https://web.facebook.com/topnotchmedicalboardprep/

TOPNOTCH MEDICAL BOARD PREP PHYSIOLOGY GANONG SUPPLEMENT HANDOUT BY THE TOPNOTCH TEAM For inquiries visit www.topnotchboardprep.com.ph or https://www.facebook.com/topnotchmedicalboardprep/ 8. Sympathetic nerve activity: A. is essential for survival B. causes contraction of some smooth muscles and relaxation of others C. causes relaxation of the radial muscles of the eye to dilate the pupil D. relaxes smooth muscle of the gastrointestinal wall and gastrointestinal sphincter E. all of the above

ANSWER: A

9. Parasympathetic nerve activity: A. is essential for survival B. affects only smooth muscles and glands C. causes contraction of the radial muscle of the eye to allow accommodation for near vision D. contracts smooth muscle of the gastrointestinal wall and relaxation of gastrointestinal sphincter E. all of the above

ANSWER: D

 Sympathetic   nerve activity causes contraction of some smooth muscles (i.e. sphincters) and relaxation of others (i.e. walls GI tract)



Ganong. Review of Medical Physiology 23rd ed. Chapter 17 , page 268.



“Parasympathetic   action favors digestion and absorption of food by increasing the activity of the intestinal musculature, increasing gastric secretion, and relaxing the pyloric sphincter.”

Ganong. Review of Medical Physiology 23rd ed. Chapter 17, page 266.

(CHAPTER 14) ELECTRICAL ACTIVITY OF THE BRAIN, SLEEP–WAKE STATES, & CIRCADIAN RHYTHMS 1. In a healthy, alert adult sitting with their eyes closed, the dominant EEG rhythm observed with electrodes over the occipital lobes is A. delta (0.5–4 Hz). B. theta (4–7 Hz). C. alpha (8–13 Hz). D. beta (18–30 Hz). E. fast, irregular low-voltage activity.

ANSWER: C alpha (8 –13 Hz): awake but at rest with the mind wandering and the eyes closed. delta (0.5–4 Hz) : stage 3 NREM sleep theta (4–7 Hz) : early stage of slowwave sleep beta (18 –30 Hz) : aroused, alert state  fast, irregular low-voltage activity  : motor activity in response to the stimulus •

• • • •

Ganong. Review of Medical Physiology 23rd ed. Chap 15, pg 233, 234.

ANSWER: D

2. A 35-year-old man spent the evening in a sleep clinic to determine whether he had obstructive sleep apnea. The tests showed that NREM sleep accounted for over 30% of his total sleep time. Which of the following pattern of changes in central neurotransmitters or neuromodulators are associated with the transition from NREM to wakefulness? A. Decrease in norepinephrine, increase in serotonin, increase in acetylcholine, decrease in histamine, and decrease in GABA. B. Decrease in norepinephrine, increase in serotonin, increase in acetylcholine, decrease in histamine, and increase in GABA. C. Decrease in norepinephrine, decrease in serotonin, increase in acetylcholine, increase in histamine, and increase in GABA. D. Increase in norepinephrine, increase in serotonin, decrease in acetylcholine, increase in histamine, and decrease in GABA. E. Increase in norepinephrine, decrease in serotonin, decrease in acetylcholine, increase in histamine, and decrease in GABA.

Ganong. Review of Medical Physiology 23rd ed. Chapter 15, page 239

3. A gamma rhythm (30–80 Hz) A. is characteristic of seizure activity. B. is seen in an individual who is awake but not focused. C. may be a mechanism to bind together sensory information into a single percept and action. D. is independent of thalamocortical loops. E. is generated in the hippocampus. 4. For the past several months, a 67-year-old woman experienced difficulty initiating and/or maintaining sleep several times a week. A friend suggested that she take melatonin to regulate her sleep –wake cycle. Melatonin secretion would probably not be increased by A. stimulation of the superior cervical ganglia. B. intravenous infusion of tryptophan. C. intravenous infusion of epinephrine. D. stimulation of the optic nerve. E. induction of pineal hydroxyindole-O-methyltransferase.

ANSWER: C •

“The gamma rhythm has been suggested as a mechanism to “bind” together diverse sensory information into a single percept and action .”

Ganong. Review of Medical Physiology 23rd ed. Chapter 15, page 236 .

ANSWER: D



Direct light stimulation of the retina would send signals to the optic nerve that would lead to decrease activity of the pineal gland and decrease in secretion of melatonin. Ganong. Review of Medical Physiology 23rd ed. Chapter , page .

TOPNOTCH MEDICAL BOARD PREP MED PHYSIOLOGY GANONG SUPPLEMENT HANDOUT BY THE TOPNOTCH TEAM Page 22 of 46 For inquiries visit www.topnotchboardprep.com.ph or https://web.facebook.com/topnotchmedicalboardprep/

TOPNOTCH MEDICAL BOARD PREP PHYSIOLOGY GANONG SUPPLEMENT HANDOUT BY THE TOPNOTCH TEAM For inquiries visit www.topnotchboardprep.com.ph or https://www.facebook.com/topnotchmedicalboardprep/ 5. Childhood absence epilepsy was diagnosed in a 10-year-old boy. His EEG showed a bilateral synchronous, symmetric 3-Hz spike-and-wave discharge. Absence seizures A. are a form of nonconvulsive generalized seizures accompanied by momentary loss of consciousness. B. are a form of complex partial seizures accompanied by momentary loss of consciousness. C. are a form of nonconvulsive generalized seizures without a loss of consciousness. D. are a form of simple partial seizures without a loss of consciousness. E. are a form of convulsive generalized seizures accompanied by momentary loss of consciousness.

ANSWER: A

6. A 57-year-old professor at a medical school experienced numerous episodes of a sudden loss of muscle tone and an irresistible urge to sleep in the middle of the afternoon. The diagnosis was narcolepsy, which: A. is characterized by a sudden onset of NREM sleep. B. has a familial incidence associated with a class II antigen of the major histocompatibility complex. C. may be due to the presence of an excessive number of orexinproducing neurons in the hypothalamus. D. is often effectively treated with dopamine receptor agonists. E. is the most common cause of daytime sleepiness.

ANSWER:B

7. Narcolepsy is triggered by abnormalities in the: A. skeletal muscle B. medulla oblongata C. hypothalamus D. olfactory bulb E. neocortex

ANSWER: C



“ Absence seizures (formerly called petit mal seizures) are one of the forms of nonconvulsive generalized seizures characterized by a momentary loss of consciousness. They are associated with 3/s doublets, each consisting of a typical spike and rounded wave, and lasting about 10 s.” Ganong. Review of Medical Physiology 23rd ed. Chap 15, page 233..







“Narcolepsy  is characterized by a sudden onset of REM sleep, unlike normal sleep which begins with NREM, slowwave sleep.” “Narcolepsy has a familial incidence strongly associated with a class II antigen of the major histocompatibility complex on chromosome 6.” “Brains from humans with narcolepsy often contain fewer hypocretin (orexin)-producing neurons in the hypothalamus.” Ganong. Review of Medical Physiology 23rd ed. Chap15 , page 237.



“Brains from humans with narcolepsy   often contain fewer hypocretin (orexin)-producing neurons in the hypothalamus.” Ganong. Review of Medical Physiology 23rd ed. Chap15 , page 237.

(CHAPTER 15) LEARNING, MEMORY, LANGUAGE, & SPEECH ANSWER: C 1. A 27-year-old man suffered a traumatic brain injury as a result of a motorcycle accident. He was unconscious and was rushed to the emergency department of the local hospital. A CT scan was performed and appropriate interventions were taken. About 6 months later he still had memory deficits. Which of the following is correctly paired to show the relationship between a brain area and a type of memory? A. Hippocampus and implicit memory B. Neocortex and associative learning C. Medial temporal lobe and declarative memory D. Angular gyrus and procedural memory E. Striatum and priming Ganong. Review of Medical Physiology 23rd ed. Chap 19, page 290.

2. The optic chiasm and corpus callosum are sectioned in a dog, and with the right eye covered, the animal is trained to bark when it sees a red square. The right eye is then uncovered and the left eye covered. The animal will now A. fail to respond to the red square because the square does not produce impulses that reach the right occipital cortex. B. fail to respond to the red square because the animal has bitemporal hemianopia. C. fail to respond to the red square if the posterior commissure is also sectioned. D. respond to the red square only after retraining. E. respond promptly to the red square in spite of the lack of input to the left occipital cortex.

ANSWER: E

3. A 32-year-old man had medial temporal lobe epilepsy for over 10 years. This caused bilateral loss of hippocampal function. As a result, this individual might be expected to experience a A. disappearance of remote memories. B. loss of working memory. C. loss of the ability to encode events of the recent past into longterm memory. D. loss of the ability to recall faces and forms but not the ability to recall printed or spoken words. E. production of inappropriate emotional responses when recalling events of the recent past.

ANSWER: C







This is an example of Intercortical Transfer of Memory “If a cat or monkey is conditioned to respond to a visual stimulus with one eye covered and then tested with the blindfold transferred to the other eye, it performs the conditioned response.” “This is true even if the optic chiasm has been cut, making the visual input from each eye go only to the ipsilateral cortex.” “Partial callosal section experiments indicate that the memory transfer occurs in the anterior commissure of the corpus callosum.” Ganong. Review of Medical Physiology 23rd ed. Chapter 15, page 239

Hippocampus: ability to encode events of the recent past into long-term memory. Neocortical areas: remote memories. Prefrontal cortex : working memory. Inferior temporal lobe: ability to recall faces and forms but not the ability to recall printed or spoken words.  Amygdala: production of inappropriate emotional responses when recalling events of the recent past.



• • •



Ganong. Review of Medical Physiology 23rd ed. Chap19, pg 294, 298, 398 .

TOPNOTCH MEDICAL BOARD PREP MED PHYSIOLOGY GANONG SUPPLEMENT HANDOUT BY THE TOPNOTCH TEAM Page 23 of 46 For inquiries visit www.topnotchboardprep.com.ph or https://web.facebook.com/topnotchmedicalboardprep/

TOPNOTCH MEDICAL BOARD PREP PHYSIOLOGY GANONG SUPPLEMENT HANDOUT BY THE TOPNOTCH TEAM For inquiries visit www.topnotchboardprep.com.ph or https://www.facebook.com/topnotchmedicalboardprep/ 4. A 70-year-old woman fell down a flight of stairs, hitting her head on the concrete sidewalk. The trauma caused a severe intracranial hemorrhage. The symptoms she might experience are dependent on the area of the brain most affected. Which of the following is incorrectly paired? A. Damage to the parietal lobe of the representational hemisphere : Unilateral inattention and neglect B. Loss of cholinergic neurons in the nucleus basalis of Meynert and related areas of the forebrain : Loss of recent memory C. Damage to the mammillary bodies : Loss of recent memory D. Damage to the angular gyrus in the categorical hemisphere : Nonfluent aphasia E. Damage to Broca area in the categorical hemisphere : Slow speech 5. The representational hemisphere is better than the categorical hemisphere at A. language functions. B. recognition of objects by their form. C. understanding printed words. D. understanding spoken words. E. mathematical calculations. 6. A 67-year-old woman suffered a stroke that damaged the posterior end of the superior temporal gyrus. A lesion of Wernicke area in the categorical hemisphere causes her to: A. lose her short-term memory. B. experience nonfluent aphasia in which she speaks in a slow, halting voice. C. experience déjà vu. D. talk rapidly but make little sense, which is characteristic of fluent aphasia. E. lose the ability to recognize faces, w/c is called prosopagnosia

ANSWER: A/ D Damage to the angular  gyrus in the categorical hemisphere :  Anomic aphasia Damage to Broca area in the categorical hemisphere : Slow speech or Nonfluent Aphasia. Inferior parietal lobule: unilateral inattention and neglect  Damage to the  parietal lobe of the representational  hemisphere : Asteriognosis and Agnosia. Loss of cholinergic neurons in the nucleus basalis of Meynert  and related areas of the forebrain : Loss of recent memory Damage to the mammillary bodies : Loss of recent memory





• •





Ganong. Review of Medical Physiology 23rd ed. Chapter , pg 293, 296, 298

ANSWER: B •

“Lesions in the categorical hemisphere  produce language disorders, whereas extensive lesions in the representational  hemisphere do not. Instead, lesions in the representational hemisphere produce astereognosis—the inability to identify objects by feeling them.” Ganong. Review of Medical Physiology 23rd ed. Chapter 19, page 298.

ANSWER: D



Lesion in Wernicke’s area: the speech itself is normal and sometimes the patients talk excessively. However, what they say is full of jargon and neologisms that make little sense. Ganong. Review of Medical Physiology 23rd ed. Chapter 19, page 297 ..

7. Which of the following is most likely involved in production of LTP? ANSWER: D A. NO release, activation of NMDA receptors, and membrane hyperpolarization “Long-term potentiation (LTP) is a rapidly developing B. Decreased Ca2+ in presynaptic or postsynaptic neurons, persistent enhancement of the postsynaptic potential activation of NMDA receptors, and membrane depolarization response to presynaptic stimulation after a brief period of C. Activation of NMDA receptors, NO-induced reduction in rapidly repeated stimulation of the presynaptic neuron.” glutamate release in a presynaptic neuron, and membrane Increased Ca2+ in presynaptic or postsynaptic neurons, depolarization activation of NMDA receptors, and membrane D. Increased Ca2+ in presynaptic or postsynaptic neurons, depolarization. activation of NMDA receptors, and membrane depolarization E. NO-induced increase in glutamate release in a presynaptic Ganong. Review of Medical Physiology 23rd ed. Chapter 19 , page 291. neuron, activation of non-NMDA receptors, membrane hyperpolarization •



ANSWER: B 8. A 79-year-old woman has been experiencing difficulty finding her way back home after her morning walks. Her husband has also noted that she takes much longer to do routine chores around the home and often appears to be confused. He is hoping that this is The cytopathologic hallmarks of Alzheimer disease are just due to “old age” but fears it may be a sign of Alzheimer intracellular neurofibrillary tangles, made up in part of disease. Which of the following is the definitive sign of this dse? hyperphosphorylated form of the tau protein that normally A. Loss of short-term memory binds to microtubules, and extracellular senile plaques, B. The presence of intracellular neurofibrillary tangles and which have a core of β-amyloid peptides (AΒ ) surrounded extracellular neuritic plaques with a core of β -amyloid peptides by altered nerve fibers and reactive glial cells. C. A mutation in genes for amyloid precursor protein (APP) on chromosome 21 Ganong. Review of Medical Physiology 23rd ed. Chapter 19 , page 294 . D. Rapid reversal of symptoms with the use of acetylcholinesterase inhibitors E. A loss of cholinergic neurons in the nucleus basalis of Meynert •

9. The representational hemisphere: A. is the right cerebral hemisphere in most right-handed individuals B. is the left cerebral hemisphere in most left-handed individuals C. includes the part of the brain concerned with language functions D. is the site of lesions in most patients with aphasia E. is morphologically identical to the opposite nonrepresentational hemisphere

ANSWER: A •

In 96% of right-handed individuals, who constitute 91% of the human population, the left hemisphere is the dominant  or categorical hemisphere; and the right hemisphere is the non-dominant  or the representational  hemisphere, Ganong. Review of Medical Physiology 23rd ed. Chapter 19 , page 295.

TOPNOTCH MEDICAL BOARD PREP MED PHYSIOLOGY GANONG SUPPLEMENT HANDOUT BY THE TOPNOTCH TEAM Page 24 of 46 For inquiries visit www.topnotchboardprep.com.ph or https://web.facebook.com/topnotchmedicalboardprep/

TOPNOTCH MEDICAL BOARD PREP PHYSIOLOGY GANONG SUPPLEMENT HANDOUT BY THE TOPNOTCH TEAM For inquiries visit www.topnotchboardprep.com.ph or https://www.facebook.com/topnotchmedicalboardprep/

SECTION 3: ENDOCRINE AND REPRODUCTIVE PHYSIOLOGY (CHAPTER 17) HYPOTHALAMIC REGULATION OF HORMONAL FUNCTIONS ANSWER: B •

1. Thirst is stimulated by A. increases in plasma osmolality and volume. B. an increase in plasma osmolality and a decrease in plasma volume. C. a decrease in plasma osmolality and an increase in plasma volume. D. decreases in p lasma osmolality and volume. E. injection of vasopressin into the hypothalamus.

When the effective osmotic pressure of the plasma rises, vasopressin secretion is increased and the thirst  mechanism is stimulated; water is retained in the body, diluting the hypertonic plasma; and water intake is increased

Ganong. Review of Medical Physiology 23rd ed. Chapter 39 , page 665

2. When an individual is naked in a room in which the air temperature is 21°C (69.8°F) and the humidity 80%, the greatest amount of heat is lost from the body by A. elevated metabolism. B. respiration. C. urination. D. vaporization of sweat. E. radiation and conduction.

ANSWER: E

In questions 3–8, select A if the item is associated with (a) below, B if the item is associated with (b) below, C if the item is associated with both (a) and (b), and D if the item is associated with neither (a) nor (b). (a) V1A vasopressin receptors (b) V2 vasopressin receptors 3. Activation of Gs 4. Vasoconstriction 5. Increase in intracellular inositol triphosphate 6. Movement of aquaporin 7. Proteinuria 8. Milk ejection

ANSWER: 3. B, 4. A, 5. A, 6. B, 7. D, 8. D V1A and V1B receptors act through phosphatidylinositol hydrolysis to increase the intracellular Ca2+ concentration. The V2 receptors act through Gs to increase cyclic adenosine 3',5'-monophosphate (cAMP) levels. V2 receptors and involves the insertion of proteins called water channels/aquaporins,  into the apical (luminal) membranes of the principal cells of the collecting ducts. V1A receptors mediate the vasoconstrictor  effect of vasopressin,



When an individual is in a cold environment, heat is lost by conduction to the surrounding air and by radiation to cool objects in the vicinity.

Ganong. Review of Medical Physiology 23rd ed. Chapter 18, page 284







Ganong. Review of Medical Physiology 23rd ed. Chap39 , page 666..

(CHAPTER 18) THE PITUITARY GLAND ANSWER: E The hormones of the  posterior lobe (PL) are released into the general circulation from the endings of supraoptic and paraventricular neurons, whereas hypophysiotropic  hormones of the anterior  pituitary  are secreted into the portal hypophysial circulation , within the median eminence from the endings of arcuate and other hypothalamic neurons. •

1. A neuroscientist is studying communication between the hypothalamus and pituitary in a rat model. She interrupts blood flow through the median eminence and then measures circulating levels of pituitary hormones following appropriate physiologic stimulation. Secretion of which of the following hormones will be unaffected by the experimental manipulation? A. Growth hormone B. Prolactin C. Thyroid-stimulating hormone D. Follicle-stimulating hormone E. Vasopressin

Ganong. Review of Medical Physiology 23rd ed. Chap 18 , page 274.

2. Which of the following p ituitary hormones is an opioid peptide? A. α-Melanocyte-stimulating hormone (α-MSH) B. β-MSH C. ACTH D. Growth hormone E. β-Endorphin

ANSWER: E

3. During childbirth, a woman suffers a serious hemorrhage and goes into shock. After she recovers, she displays symptoms of hypopituitarism. Which of the following will not be expected in this patient? A. Cachexia B. Infertility C. Pallor D. Low basal metabolic rate E. Intolerance to stress

ANSWER: E



Proopiomelanocortin (POMC) is hydrolyzed to corticotropin-like intermediate-lobe peptide (CLIP),  γLPH, and appreciable quantities of β –endorphin, which is an opioid peptide Ganong. Review of Medical Physiology 23rd ed. Chap22 , page 352.



Thus, patient with hypopituitarism is that of a lethargic person (from lack of thyroid hormones) who is gaining weight (not cachexia) (because of lack of fat mobilization by growth, adrenocorticotropic, adrenocortical, and thyroid hormones) and has lost all sexual functions.

Ganong. Review of Medical Physiology 23rd ed. Chapter 75 , page 926.

TOPNOTCH MEDICAL BOARD PREP MED PHYSIOLOGY GANONG SUPPLEMENT HANDOUT BY THE TOPNOTCH TEAM Page 25 of 46 For inquiries visit www.topnotchboardprep.com.ph or https://web.facebook.com/topnotchmedicalboardprep/

TOPNOTCH MEDICAL BOARD PREP PHYSIOLOGY GANONG SUPPLEMENT HANDOUT BY THE TOPNOTCH TEAM For inquiries visit www.topnotchboardprep.com.ph or https://www.facebook.com/topnotchmedicalboardprep/ 4. A scientist finds that infusion of growth hormone into the median eminence of the hypothalamus in experimental animals inhibits the secretion of growth hormone and concludes that this proves that growth hormone feeds back to inhibit GHRH secretion. Do you accept this conclusion? A. No, because growth hormone does not cross the blood-brain barrier B. No, because the infused growth hormone could be stimulating dopamine secretion. C. No, because substances placed in the median eminence could be transported to the anterior pituitary. D. Yes, because systemically administered growth hormone inhibits growth hormone secretion. E. Yes, because growth hormone binds GHRH, inactivating it.

ANSWER: C

Growth hormone secretion is under feedback control, like the secretion of other anterior pituitary hormones. It acts on the hypothalamus to antagonize GHRH release. Growth hormone also increases circulating IGF-I, and IGF-I  in turn exerts a direct inhibitory action on growth hormone secretion from the pituitary. It also stimulates  somatostatin secretion. GH  in systemic circulation is the one responsible for the feedback, and not the one in the portal system.





Ganong. Review of Medical Physiology 23rd ed. Chapter 24, page 384

5. The growth hormone receptor A. activates Gs. B. requires dimerization to exert its effects. C. must be internalized to exert its effects. D. resembles the IGF-I receptor. E. resembles the ACTH receptor.

ANSWER: B Growth hormone receptor: o Activates PLC  o Requires dimerization to exert its effects. o Is not internalized   to exert its effects, only the  JAK  STAT migrates in the nucleus. o resembles the prolactin receptor.

6. Which of the following hormones exerts the least effect on growth? A. growth hormone B. testosterone C. T4 D. insulin E. Vasopressin

ANSWER: E Growth overall is a complex phenomenon that is affected not only by growth hormone and somatomedins, but also, as would be predicted by the previous discussion, by t hyroid hormones, androgens, estrogens, glucocorticoids,  and insulin .



Ganong. Review of Medical Physiology 23rd ed. Chapter 24, page 381, 382; chapter 25, page 401 •

Ganong. Review of Medical Physiology 23rd ed. Chapter24 , page 384.

(CHAPTER 19) THE THYROID GLAND 1. A 40-year-old woman comes to her primary care clinician complaining of nervousness and an unexplained weight loss of 20 pounds over the past 3 months despite her impression that she is eating all the time. On physical examination, her eyes are found to be protruding, her skin is moist and warm, and her fingers have a slight tremor. Compared to a healthy individual, a biopsy of her thyroid gland would most likely reveal which of the following: A. Decreased numbers of reabsorption lacunae B. Decreased evidence of endocytosis C. A decrease in the cross-sectional area occupied by colloid D. Increased levels of NIS in the basolateral membrane of thyrocytes E. Decreased evidence of lysosomal activity 2. Which of the following is not essential for normal biosynthesis of thyroid hormones? A. Iodine B. Ferritin C. Thyroglobulin D. Protein synthesis E. TSH

ANSWER: C



“Thyroid gland is increased to two to three times normal size, with tremendous hyperplasia and infolding of the follicular cell lining into the follicles, so that the number of cells is increased greatly. Also, each cell increases its rate of secretion severalfold.” This leads to a decrease in the cross-sectional area occupied by colloid

Guyton.Textbook of Medical Physiology 11th ed. Chapter76, page 940.

ANSWER: B Iodide (not ferritin) undergoes a process referred to as organification. First, it is oxidized to iodine, and then incorporated into the carbon 3 position of tyrosine residues that are part of the thyroglobulin, which is a glycoprotein. TSH, there are increases in iodide binding; synthesis of T3, T4, and iodotyrosines; secretion of thyroglobulin into the colloid. •



Ganong. Review of Medical Physiology 23rd ed. Chap20, page 304.

3. Increasing intracellular I– due to the action of NIS is an example of A. Endocytosis B. Passive diffusion C. Na+ and K+ cotransport D. Primary active transport E. Secondary active transport 4. The metabolic rate is least affected by an increase in the plasma level of A. TSH B. TRH C. TBG D. Free T4 E. Free T3 5. In which of the following conditions is it most likely that the TSH response to TRH will be reduced? A. Hypothyroidism due to tissue resistance to thyroid hormone B. Hypothyroidism due to disease destroying the thyroid gland C. Hyperthyroidism due to circulating antithyroid antibodies with TSH activity D. Hyperthyroidism due to diffuse hyperplasia of thyrotropes of the anterior pituitary E. Iodine deficiency

ANSWER: E The basolateral membranes of thyrocytes facing the capillaries contain a Na+/I – symporter that transports two Na+ ions and one I ion into the cell with each cycle, against the The process involved is  secondary active transport   electrochemical gradient for I –. •

Ganong. Review of Medical Physiology 23rd ed. Chapter , page .

ANSWER: C •

Patients with elevated or decreased concentrations of binding proteins, particularly TBG , are typically neither hyper-nor hypothyroid; that is, they are euthyroid.

Ganong. Review of Medical Physiology 23rd ed. Chapter 20, page 306

ANSWER: C

TRH  directly affects the anterior pituitary gland cells to increase their output of TSH. However, high level of thyroid hormone secretion caused by thyroid stimulating immunoglobulin in turn suppresses anterior pituitary formation of TSH.



Guyton.Textbook of Medical Physiology 11th ed.Chapter 76, pages 938, 940

TOPNOTCH MEDICAL BOARD PREP MED PHYSIOLOGY GANONG SUPPLEMENT HANDOUT BY THE TOPNOTCH TEAM Page 26 of 46 For inquiries visit www.topnotchboardprep.com.ph or https://web.facebook.com/topnotchmedicalboardprep/

TOPNOTCH MEDICAL BOARD PREP PHYSIOLOGY GANONG SUPPLEMENT HANDOUT BY THE TOPNOTCH TEAM For inquiries visit www.topnotchboardprep.com.ph or https://www.facebook.com/topnotchmedicalboardprep/ 6. Hypothyroidism due to disease of the thyroid gland is associated with increased plasma levels of: A. Cholesterol B. Albumin C. RT3 D. Iodide E. TBG 7. A young woman has puffy skin and a hoarse voice. Her plasma TSH concentration is low but increases markedly when she is given TRH. She probably has: A. hyperthyroidism due to a thyroid tumor. B. hypothyroidism due to a primary abnormality in the thyroid gland. C. hypothyroidism due to a primary abnormality in the pituitary gland. D. hypothyroidism due to a primary abnormality in the hypothalamus. E. hyperthyroidism due to a primary abnormality in the hypothalamus 8. The enzyme primarily responsible for the conversion of T4 to T3 in the periphery is: A. D1 thyroid deiodinase B. D2 thyroid deiodinase C. D3 thyroid deiodinase D. Thyroid peroxidase E. None of the above

ANSWER: A •

Thyroid hormones lower circulating cholesterol levels. Therefore in hypothyroidism, levels of cholesterol is expected to increase. Ganong. Review of Medical Physiology 23rd ed. Chap20 , page 313.

ANSWER: D •



Synthesis and secretion of thyroid hormones is stimulated by thyroid-stimulating hormone (TSH) from the pituitary, which in turn is released in response to thyrotropin-releasing hormone (TRH) from the hypothalamus. Problems in the hypothalamus would result in low levels of TRH and therefore exogenous administration of TRH  would increase the level of TSH. Ganong. Review of Medical Physiology 23rd ed. Chap20 , page 308..

ANSWER: A •





D1 deiodinase (liver, kidneys, thyroid, and pituitary) primarily responsible for monitoring the formation of T3 from T4 in the periphery. D2 deiodinase (brain, pituitary, and brown fat) produces a supply of T3 to neurons. D3 deiodinase (brain and in reproductive tissues). Ganong. Review of Medical Physiology 23rd ed. Chap20 , page 306 ..

ANSWER: Cyclic Adenosine Monophosphate mediates the stimulatory effect of TSH: 1. Increased proteolysis of the thyroglobulin 2. Increased activity of the iodide pump 3. Increased iodination of tyrosine 4. Increased size and increased secretory activity of the thyroid cells 5. Increased number of thyroid cells •

9. Which of the following would be least affected by injections of TSH? A. Thyroidal uptake of iodine B. Synthesis of thyroglobulin C. Cyclic adenosine monophosphate (cAMP) in thyroid cells D. Cyclic guanosine monophosphate (cGMP) in thyroid cells E. Size of the thyroid

Guyton.Textbook of Medical Physiology 11th ed. Chap 76 , page 938..

10. Thyroid hormone receptors bind to DNA in which of the ff forms? A. A heterodimer with the prolactin receptor B. A heterodimer with the growth hormone receptor C. A heterodimer with the retinoid X receptor D. A heterodimer with the insulin receptor E. A heterodimer with the progesterone receptor

ANSWER: C •

Thyroid receptors bind to DNA as monomers, homodimers, and heterodimers with other nuclear receptors, particularly the retinoid X receptor. Ganong. Review of Medical Physiology 23rd ed. Chap 20, page 308

(CHAPTER 20) THE ADRENAL MEDULLA & ADRENAL CORTEX 1. Which of the following is produced only by large amounts of glucocorticoids? A. Normal responsiveness of fat depots to norepinephrine B. Maintenance of normal vascular reactivity C. Increased excretion of a water load D. Inhibition of the inflammatory response E. Inhibition of ACTH secretion

ANSWER: “Glucocorticoids inhibit the inflammatory response to tissue injury these effects require high levels of circulating glucocorticoids and cannot be produced by administering steroids without producing the other manifestations of glucocorticoid excess.” •

Ganong. Review of Medical Physiology 23rd ed. Chapter 22 , page 351

ANSWER: B 2. Which of the following are incorrectly paired? A. Gluconeogenesis : Cortisol B. Free fatty acid mobilization : Dehydroepiandrosterone C. Muscle glycogenolysis : Epinephrine D. Kaliuresis : Aldosterone E. Hepatic glycogenesis : Insulin 3. Which of the following hormones has the shortest plasma half-life? A. Corticosterone B. Renin C. Dehydroepiandrosterone D. Aldosterone E. Norepinephrine 4. Mole for mole, which of the following has the greatest effect on Na+ excretion? A. Progesterone B. Cortisol C. Vasopressin D. Aldosterone E. Dehydroepiandrosterone

• • • • •

Gluconeogenesis : Cortisol Puberty: Dehydroepiandrosterone Muscle glycogenolysis : Epinephrine Kaliuresis : Aldosterone Hepatic glycogenesis : Insulin

Ganong. Review of Medical Physiology 23rd ed. Chapter 22 , page 348

ANSWER: E Norepinephrine :2 minutes Renin: 15 minutes Aldosterone: 20 minutes Corticosterone: 60-90 minutes Dehydroepiandrosterone: 7-22 hours • • • • •

Ganong. Review of Medical Physiology 23rd ed. Chap77, page 947.

ANSWER: D •

“Changes in Na+ excretion occur too rapidly to be due solely to changes in aldosterone secretion. Na+ excretion is decreased within a few minutes, and this rapid change in Na+ excretion occurs in adrenalectomized subjects.” Ganong. Review of Medical Physiology 23rd ed. Chap 39, page 670.

TOPNOTCH MEDICAL BOARD PREP MED PHYSIOLOGY GANONG SUPPLEMENT HANDOUT BY THE TOPNOTCH TEAM Page 27 of 46 For inquiries visit www.topnotchboardprep.com.ph or https://web.facebook.com/topnotchmedicalboardprep/

TOPNOTCH MEDICAL BOARD PREP PHYSIOLOGY GANONG SUPPLEMENT HANDOUT BY THE TOPNOTCH TEAM For inquiries visit www.topnotchboardprep.com.ph or https://www.facebook.com/topnotchmedicalboardprep/ 5. Mole for mole, which of the following has the greatest effect on plasma osmolality? A. Progesterone B. Cortisol C. Vasopressin D. Aldosterone E. Dehydroepiandrosterone

ANSWER: 20 The injection of extremely minute quantities of  ADH —as small as 2 nanograms—can cause decreased excretion of water by the kidneys (antidiuresis) but allows continuing excretion of electrolytes , thus decreasing the plasma osmolarity back toward normal. •

Guyton.Textbook of Medical Physiology 11th ed. Chapter 58, page 733. Chapter 75 , page 928.

ANSWER: D 6. The secretion of which of the following would be least affected by a decrease in extracellular fluid volume? A. CRH B. Arginine vasopressin C. Dehydroepiandrosterone D. Estrogens E. Aldosterone



In cases when there is decrease in extracellular fluid volume (e.i. hemorrhage) aside from activating the renin –angiotensin-aldosterone system and vasopressin, the physiologic stress also increases release of CRH which stimulates ACTH which in turn will also affect the output of aldosterone as well as that of glucocorticoids and dehydroepiandrosterone Ganong. Review of Medical Physiology 23rd ed. Chap22 , page 356..

ANSWER: D 7. A young man presents with a blood pressure of 175/110 mm Hg. He is found to have a high circulating aldosterone but a low circulating cortisol. Glucocorticoid treatment lowers his circulating aldosterone and lowers his blood pressure to 140/85 mm Hg. He probably has an abnormal: A. 17α-hydroxylase. B. 21β-hydroxylase. C. 3β-hydroxysteroid dehydrogenase. D. aldosterone synthase. E. cholesterol desmolase.







The patient had glucocorticoid-remediable aldosteronism (GRA) which is an autosomal dominant disorder in which the increase in aldosterone secretion produced by ACTH is no longer transient. The hypersecretion of aldosterone and the accompanying hypertension are remedied when ACTH secretion is suppressed by administering glucocorticoids. The genes encoding aldosterone synthase and 11βhydroxylase are 95% identical and are close together on chromosome 8.

Ganong. Review of Medical Physiology 23rd ed. Chapter 22, page 356

8. A 32-year-old woman presents with a blood pressure of 155/96 mm Hg. In response to questioning, she admits that she loves licorice and eats some at least three times a week. She probably has a low level of A. type 2 11β-hydroxysteroid dehydrogenase activity. B. ACTH. C. 11β-hydroxylase activity. D. glucuronyl transferase. E. norepinephrine.

ANSWER: A Patient had apparent mineralocorticoid excess (AME) secondaryto prolonged ingestion of licorice which contains glycyrrhetinic acid, that inhibits 11βhydroxysteroid dehydrogenase type 2. If 11β -hydroxysteroid dehydrogenase type 2 is inhibited or absent, cortisol has marked mineralocorticoid effects. •



Ganong. Review of Medical Physiology 23rd ed. Chapter 22 , page 355

ANSWER: It increases transport of ENaCs from the cytoplasm to the cell membrane. Aldosterone binds to a cytoplasmic receptor, and the receptor-hormone complex moves to the nucleus. Evidence is accumulating that aldosterone also binds to the cell membrane and increases the activity of membrane Na+–K+ exchangers. Among the genes activated by aldosterone is the gene for serum- and glucocorticoid regulated kinase (sgk) , a serine-threonine protein kinase. Aldosterone doesn’t bind to glucocorticoid receptors. •

9. In its action in cells, aldosterone A. increases transport of ENaCs from the cytoplasm to the cell membrane. B. does not act on the cell membrane. C. binds to a receptor excluded from the nucleus. D. may activate a heat shock protein. E. also binds to glucocorticoid receptors.









Ganong. Review of Medical Physiology 23rd ed. Chapter 22 , page 355

(CHAPTER 21) HORMONAL CONTROL OF CALCIUM, & PHOSPHATE METABOLISM & THE PHYSIOLOGY OF BONE 1. A patient with parathyroid deficiency 10 days after inadvertent damage to the parathyroid glands during thyroid surgery would probably have: A. low plasma phosphate and Ca2+ levels and tetany. B. low plasma phosphate and Ca2+ levels and tetanus. C. a low plasma Ca2+ level, increased muscular excitability, and spasm of the muscles of the upper extremity (Trousseau sign). D. high plasma phosphate and Ca2+ levels and bone demineralization. E. increased muscular excitability, a high plasma Ca2+ level, and bone demineralization. 2. In an experiment, a rat is infused with a small volume of a calcium chloride solution, or sodium chloride as a control. Compared to the control condition, which of the following would result from the calcium load? A. Bone demineralization B. Increased formation of 1,25-dihydroxycholecalciferol C. Decreased secretion of calcitonin D. Decreased blood coagulability E. Increased formation of 24,25-dihydroxycholecalciferol

ANSWER: C



After parathyroidectomy : o Decline in the plasma calcium level. o Neuromuscular hyperexcitability  appear. o Hypocalcemic tetany.(chvostek’s and trousseau’s). o Plasma phosphate levels usually rise.

Ganong. Review of Medical Physiology 23rd ed. Chapter 23 , page 368

ANSWER: E •

When the plasma Ca2+ level is high, little 1,25dihydroxycholecalciferol is produced and the kidneys produce the relatively inactive metabolite 24,25dihydroxycholecalciferol instead.

Ganong. Review of Medical Physiology 23rd ed. Chapter 23 , page 366

TOPNOTCH MEDICAL BOARD PREP MED PHYSIOLOGY GANONG SUPPLEMENT HANDOUT BY THE TOPNOTCH TEAM Page 28 of 46 For inquiries visit www.topnotchboardprep.com.ph or https://web.facebook.com/topnotchmedicalboardprep/

TOPNOTCH MEDICAL BOARD PREP PHYSIOLOGY GANONG SUPPLEMENT HANDOUT BY THE TOPNOTCH TEAM For inquiries visit www.topnotchboardprep.com.ph or https://www.facebook.com/topnotchmedicalboardprep/ ANSWER: D Kidneys: converts 25-Hydroxycholecalciferol into 1,25-Dihydroxycholecalciferol (active form) using 3. Which of the following is not involved in regulating plasma Ca 2+ levels? 1α-Hydroxylase and 24,25-Dihydroxycholecalciferol A. Kidneys (inactive form) using 24-Hydroxylase. B. Skin  Skin: sunlight activates 7-Dehydrocholesterol into C. Liver Vitamin D3 (cholecalciferol) D. Lungs Liver : converts Vitamin D3 (cholecalciferol) to 25E. Intestine Hydroxycholecalciferol with the use of 25-Hydroxylase Intestine: increases absorption of calcium in response to increased vitamin D level •







Ganong. Review of Medical Physiology 23rd ed. Chapter 23, page 366

4. 1,25-Dihydroxycholecalciferol affects intestinal Ca2+ absorption through a mechanism that: A. includes alterations in the activity of genes. B. activates adenylyl cyclase. C. decreases cell turnover. D. changes gastric acid secretion. E. involves degradation of apical calcium channels.

ANSWER: A

5. Which of the following would you expect to find in a patient whose diet has been low in calcium for 2 months? A. Increased formation of 24,25-dihydroxycholecalciferol B. Decreased amounts of calcium-binding protein in intestinal epithelial cells C. Increased parathyroid hormone secretion D. A high plasma calcitonin concentration E. Increased plasma phosphate

ANSWER: C

6. A mouse is engineered to lack a transcription factor necessary for the normal development of osteoclasts. Compared to normal littermate mice, which of the following would be reduced in the knock-out animals? A. Phosphate deposition in trabecular bone B. Hydroxyapatite levels in bone C. Osteoblast proliferation D. Secretion of acid proteases E. Bone collagen

ANSWER: D Osteoclast forms acid proteases using two types of substances: (1) proteolytic enzymes, released from the lysosomes of the osteoclasts, and (2) several acids, including citric acid and lactic acid, released from the mitochondria and secretory vesicles. The enzymes digest or dissolve the organic matrix of the bone, and the acids cause solution of the bone salts.

7. The skeleton of a normal male college student would be expected to display which of the following features, relative to that of his 7-yearold brother? A. Merging of cortical bone and trabecular bone. B. Differentiation of osteoclasts and osteoblasts. C. An extended amount of proliferating cartilage that contributes to bone elongation. D. A meeting of the lacunae with the trabecular bone. E. Epiphyses that are united with the bone shaft.

ANSWER: E

8. At epiphysial closure A. cortical bone and trabecular bone merge B. osteoclasts and osteoblasts undergo differentiation C. there is an extended amount of prolifering cartilage that contributes to bone elongation D. lacunae meet the trabecular bone E. ephyses unit with the shaft to end normal linear bone growth

ANSWER: E



1,25 dihydroxycholecalciferol stimulates the expression of a number of gene products involved in calcium transport (Ca2+– ATPase) and handling via its receptor (calbindin) which acts as a transcriptional regulator in its ligand-bound form.

Ganong. Review of Medical Physiology 23rd ed. Chapter 23 , page 365

• •

• • •

Increased formation of 1,25 dihydroxycholecalciferol  Increased amounts of calcium-binding protein in intestinal epithelial cells Increased parathyroid hormone secretion A low plasma calcitonin concentration Decreased plasma phosphate

Ganong. Review of Medical Physiology 23rd ed. Chapter 23 , page 368 •



Ganong. Review of Medical Physiology 23rd ed. Chapter 79 , page 982



The patient age group (college) approximates the time of epiphyseal closure in men. Linear bone growth can occur as long as the epiphyses are separated from the shaft of the bone, but such  growth ceases after the epiphyses unite with the shaft (epiphysial closure).

Ganong. Review of Medical Physiology 23rd ed. Chapter 23, page 371



Linear bone growth can occur as long as the epiphyses are separated from the shaft of the bone, but such  growth ceases after the epiphyses unite with the shaft (epiphysial closure).

Ganong. Review of Medical Physiology 23rd ed. Chapter 23, page 371

(CHAPTER 22) REPRODUCTIVE DEVELOPMENT & FUNCTION OF THE FEMALE REPRODUCTIVE SYSTEM ANSWER: 1. If a young woman has high plasma levels of T3, cortisol, and renin activity but her blood pressure is only slightly elevated and she has no symptoms or signs of thyrotoxicosis or Cushing syndrome, the most likely explanation is that A. she has been treated with TSH and ACTH. B. she has been treated with T3 and cortisol. C. she is in the third trimester of pregnancy. D. she has an adrenocortical tumor. E. she has been subjected to chronic stress. 2. In humans, fertilization usually occurs in the: A. vagina. B. cervix C. uterine cavity. D. uterine tubes. E. abdominal cavity.

The cytotrophoblast of the human chorion contains  prorenin The rate of adrenocortical secretion of the  glucocorticoids is moderately increased throughout pregnancy The increased thyroxine (as well as T3) production is caused at least partly by a thyrotropic effect of human chorionic gonadotropin.







Ganong. Review of Medical Physiology 23rd ed. Chapter 25, page 425 Guyton.Textbook of Medical Physiology 11th ed.Chap82 , page 1034.

ANSWER: D •

In humans, fertilization of the ovum by the sperm usually occurs in the ampulla of the uterine tube .

Ganong. Review of Medical Physiology 23rd ed. Chapter 25, page 423

TOPNOTCH MEDICAL BOARD PREP MED PHYSIOLOGY GANONG SUPPLEMENT HANDOUT BY THE TOPNOTCH TEAM Page 29 of 46 For inquiries visit www.topnotchboardprep.com.ph or https://web.facebook.com/topnotchmedicalboardprep/

TOPNOTCH MEDICAL BOARD PREP PHYSIOLOGY GANONG SUPPLEMENT HANDOUT BY THE TOPNOTCH TEAM For inquiries visit www.topnotchboardprep.com.ph or https://www.facebook.com/topnotchmedicalboardprep/ ANSWER: C Relaxin is a polypeptide hormone that is produced in the corpus luteum, uterus, placenta, and mammary glands in women and in the prostate gland in men. During pregnancy , it relaxes the pubic symphysis and other pelvic joints and  softens and dilates the uterine cervix. In men, it is found in  semen, where it may help maintain sperm motility and aid in sperm penetration of the ovum •

3. Which of the following is not a steroid? A. 17α-hydroxyprogesterone B. Estrone C. Relaxin D. Pregnenolone E. Etiocholanolone





Ganong. Review of Medical Physiology 23rd ed. Chapter 25 , page 420

ANSWER: A  fetal  ACTH  DHEAS  estrogen This makes the uterus : o more excitable (increases the number of gap junctions between myometrial cells) o production of more prostaglandins, which in turn cause uterine contractions In early labor, the oxytocin concentration in maternal plasma is not elevated from the prelabor value. •

4. Which of the following probably triggers the onset of labor? A. ACTH in the fetus B. ACTH in the mother C. Prostaglandins D. Oxytocin E. Placental renin





Ganong. Review of Medical Physiology 23rd ed. Chapter 25, page 425

5. Full development and function of the seminiferous tubules require: A. somatostatin B. LH C. Oxytocin D. FSH E. androgens and FSH

ANSWER: E Factors that affects spermatogenesis in the seminiferous tubules: o FSH   acts on the Sertoli cells to facilitate the last stages of spermatid   maturation. o Androgen  affects maturation from spermatids to  spermatozoa depends on androgen acting on the Sertoli cells. •

Ganong. Review of Medical Physiology 23rd ed. Chapter 25 , page 404

ANSWER: A  fetal  ACTH  DHEAS  estrogen This makes the uterus : o more excitable (increases the number of gap junctions between myometrial cells) o production of more prostaglandins, which in turn cause uterine contractions In early labor, the oxytocin concentration in maternal plasma is not elevated from the prelabor value. •

6. Which of the following probably triggers the onset of labor? A. ACTH in the fetus B. ACTH in the mother C. Prostaglandins D. Oxytocin E. Placental renin





Ganong. Review of Medical Physiology 23rd ed. Chapter 25, page 425

(CHAPTER 23) FUNCTION OF THE MALE REPRODUCTIVE SYSTEM 1. Full development and function of the seminiferous tubules require A. somatostatin. B. LH. C. oxytocin. D. FSH. E. androgens and FSH.

ANSWER: E Factors that affects spermatogenesis in the seminiferous tubules: o FSH   acts on the Sertoli cells to facilitate the last stages of spermatid   maturation. o Androgen  affects maturation from spermatids to  spermatozoa depends on androgen acting on the Sertoli cells. •

Ganong. Review of Medical Physiology 23rd ed. Chapter 25 , page 404

2. In human males, testosterone is produced mainly by the: A. Leydig cells. B. Sertoli cells. C. seminiferous tubules. D. epididymis. E. vas deferens. 3. Nitric oxide synthase contributes to erection by: ’ A. raising cAMP levels that relax smooth muscles and increase blood flow. B. blocking phosphodiesterases to increase cGMP levels that release smooth muscle and increase blood flow. C. activating soluble guanylyl cyclases to increase cGMP levels that relax smooth muscle and increase blood flow. D. raising intracellular Ca2+ concentrations that relax smooth muscles and increase blood flow.

ANSWER: A •

Testosterone is synthesized from cholesterol in the Leydig cells and is also formed from androstenedione secreted by the adrenal cortex.

Ganong. Review of Medical Physiology 23rd ed. Chapter 25, page 406

ANSWER: C



Nitric Oxide Synthase nitric oxide production  activates guanylyl cyclase  increased production of cyclic GMP  potent vasodilator. Ganong. Review of Medical Physiology 23rd ed. Chap25 , page 405.

ANSWER: B •

4. Testosterone is p roduced A. in the testes after reduction of dihydrotestosterone. B. in Leydig cells from cholesterol and pregnenolone precursors. C. by LH in Leydig cells. D. as a precursor for several membrane lipids.





Testosterone is not produced by LH, it is produced by pregnenolone precursor in the Leydig cells, it is also not due to reduction of dihydrotestosterone: LH   stimulates  Leydig cells   converts cholesterol to pregnenolone DHEAS Testosterone. Dihydrotestosterone (DHT) is formed by the reduction of testosterone with 5α-reductase. DHT amplifies the action of testosterone in target tissues. Ganong. Review of Medical Physiology 23rd ed. Chap25 , page 406.

TOPNOTCH MEDICAL BOARD PREP MED PHYSIOLOGY GANONG SUPPLEMENT HANDOUT BY THE TOPNOTCH TEAM Page 30 of 46 For inquiries visit www.topnotchboardprep.com.ph or https://web.facebook.com/topnotchmedicalboardprep/

TOPNOTCH MEDICAL BOARD PREP PHYSIOLOGY GANONG SUPPLEMENT HANDOUT BY THE TOPNOTCH TEAM For inquiries visit www.topnotchboardprep.com.ph or https://www.facebook.com/topnotchmedicalboardprep/

(CHAPTER 24) ENDOCRINE FUNCTIONS OF THE PANCREAS & REGULATION OF CARBOHYDRATE METABOLISM ANSWER: Pancreatic endocrine and exocrine cells structure: o A cells: glucagon o B cells: insulin o D cells: somatostatin o F cells: pancreatic polypeptide o Pancreatic exocrine cells: chymotrypsinogen Gastrin is a hormone that is released by G cells in the antrum of the stomach both in response to a specific neurotransmitter released from enteric nerve endings. •

1. Which of the following are incorrectly paired? A. B cells: insulin B. D cells: somatostatin C. A cells: glucagons D. Pancreatic exocrine cells: chymotrypsinogen E. F cells: gastrin



Ganong. Review of Medical Physiology 23rd ed. Chap21, pg316. Chap26, pg43

ANSWER: D Insulin increases protein synthesis Progesterone does not  have anabolic effects. Hypoglycemia triggers increased secretion of at least four counter-regulatory hormones: o Epinephrine: glycogenolysis in skeletal muscle o Glucagon: gluconeogenesis o Cortisol  : plasma glucose level o Growth hormone: plasma glucose level **The end result is increased blood glucose level •

2. Which of the following are incorrectly paired? A. Epinephrine: increased glycogenolysis in skeletal muscle B. Insulin: increased protein synthesis C. Glucagon: increased gluconeogenesis D. Progesterone: increased plasma glucose level E. Growth hormone: increased plasma glucose level

• •



Ganong. Review of Medical Physiology 23rd ed. Chapter 21, page . 325

3. Which of the following would be least likely to be seen 14 days after a rat is injected with a drug that kills all of its pancreatic B cells? A. A rise in the plasma H+ concentration B. A rise in the plasma glucagon concentration C. A fall in the plasma HCO3 – concentration D. A fall in the plasma amino acid concentration E. A rise in plasma osmolality

ANSWER: D B cells are responsible pof the production of insulin. o Destroyed b cells  low insulin less protein syntesis  elevated amino acids in the plasma . Since the body cannot utilized glucose, it would release counterregulatory hormones : glucagon,cortisol,epinephrine,growth hormone. Increased H+ and decreased HCO3- is caused by Ketoacidosis secondary to accumulation of acetoacetic acid. Hyperglycemia increased plama osmolarity. •







Ganong. Review of Medical Physiology 23rd ed. Chapter 21, page .323.

4. When the plasma glucose concentration falls to low levels, a number of different hormones help combat the hypoglycemia. After intravenous administration of a large dose of insulin, the return of a low blood sugar level to normal is delayed in A. adrenal medullary insufficiency. B. glucagon deficiency. C. combined adrenal medullary insufficiency and glucagon deficiency. D. thyrotoxicosis. E. acromegaly 5. Insulin increases the entry of glucose into: A. all tissues. B. renal tubular cells. C. the mucosa of the small intestine. D. most neurons in the cerebral cortex. E. skeletal muscle.

ANSWER: C

6. Glucagon increases glycogenolysis in liver cells but ACTH does not because: A. cortisol increases the plasma glucose level. B. liver cells have an adenylyl cyclase different from that in adrenocortical cells. C. ACTH cannot enter the nucleus of liver cells. D. the membranes of liver cells contain receptors different from those in adrenocortical cells. E. liver cells contain a protein that inhibits the action of ACTH.

ANSWER: Membranes of liver cells contain receptors different from those in adrenocortical cells. Glucagon acts on phospholipase C on hepatic cellsincrease cytoplasmic Ca2+   glycogenolysis.  ACTH   binds to high-affinity receptors on the plasma membraneof adrenocortical cells. This activates adenylyl cyclase via Gs.

7. A meal rich in proteins containing the amino acids that stimulate insulin secretion but low in carbohydrates does not cause hypoglycemia because A. the meal causes a compensatory increase in T4 secretion. B. cortisol in the circulation prevents glucose from entering muscle. C. glucagon secretion is also stimulated by the meal. D. the amino acids in the meal are promptly converted to glucose E. insulin does not bind to insulin receptors if the plasma concentration of amino acids is elevated.

ANSWER: C

“The keys to counter-regulation appear to be epinephrine and  glucagon: if the plasma concentration of either increases, the decline in the plasma glucose level is reversed; but if both fail to increase, there is little if any compensatory rise in the plasma glucose level. The actions of the other hormones are supplementary.”



Ganong. Review of Medical Physiology 23rd ed. Chapter 21 , page .326.

ANSWER: E •

GLUT 4 is an insulin-stimulated glucose transporter that promotes uptake of  glucose to skeletal and cardiac muscle, and adipose tissue. Ganong. Review of Medical Physiology 23rd ed. Chapter 21, page .320.







Ganong. Review of Medical Physiology 23rd ed. Chapter 21 , page . 321.

The increase in  glucagon secretion following a protein meal is also valuable, since the amino acids stimulate insulin  secretion and the secreted glucagon prevents the development of hypoglycemia while the insulin promotes storage of the absorbed carbohydrates and lipids.



Ganong. Review of Medical Physiology 23rd ed. Chapter 21, page . 330

TOPNOTCH MEDICAL BOARD PREP MED PHYSIOLOGY GANONG SUPPLEMENT HANDOUT BY THE TOPNOTCH TEAM Page 31 of 46 For inquiries visit www.topnotchboardprep.com.ph or https://web.facebook.com/topnotchmedicalboardprep/

TOPNOTCH MEDICAL BOARD PREP PHYSIOLOGY GANONG SUPPLEMENT HANDOUT BY THE TOPNOTCH TEAM For inquiries visit www.topnotchboardprep.com.ph or https://www.facebook.com/topnotchmedicalboardprep/

SECTION 4: GASTROINTESTINAL PHYSIOLOGY (CHAPTER 25) OVERVIEW OF GASTROINTESTINAL FUNCTION & REGULATION ANSWER: C

1. Water is absorbed in the jejunum, ileum, and colon and excreted in the feces. Arrange these in order of the amount of water absorbed or excreted from greatest to smallest. A. Colon, jejunum, ileum, feces B. Feces, colon, ileum, jejunum C. Jejunum, ileum, colon, feces D. Colon, ileum, jejunum, feces E. Feces, jejunum, ileum, colon

Ganong. Review of Medical Physiology 23rd ed. Chapter 26 , page .441

ANSWER: 2. Following a natural disaster in Haiti, there is an outbreak of cholera among displaced persons living in a tent encampment. The affected individuals display severe diarrheal symptoms Cholera toxin  increase cAMP opens chloride channels because of which of the following changes in intestinal transport? ’ chloride ions secretion  activate a sodium pump  secretion A. Increased Na+–K+ cotransport in the small intestine of sodium ions  NaCl  formed in the crypts  extreme B. Increased K+ secretion into the colon osmosis of water from the blood into the intestinal lumen. C. Reduced K+ absorption in the crypts of Lieberk ühn D. Increased Na+ absorption in the small intestine Guyton.Textbook of Medical Physiology 11th ed.Chapter 65, page 815 E. Increased Cl− secretion into the intestinal lumen •

3. A 50-year-old man comes to see his clinician complaining of severe epigastric pain, frequent heartburn, and unexplained weight loss of 20 pounds over a 6-month period. He claims to have obtained no relief from over-the-counter H2 antihistamine drugs. He is referred to a gastroenterologist, and upper endoscopy reveals erosions and ulcerations in the proximal duodenum and an increased output of gastric acid in the fasting state. The patient is most likely to have a tumor secreting which of the following hormones? A. Secretin B. Somatostatin C. Motilin D. Gastrin E. Cholecystokinin 4. Which of the following has the highest pH? A. Gastric juice B. Colonic luminal contents C. Pancreatic juice D. Saliva E. Contents of the intestinal crypts 5. A 60-year-old woman undergoes total pancreatectomy because of the presence of a tumor. Which of the following outcomes would not be expected after she recovers from the operation? A. Steatorrhea B. Hyperglycemia C. Metabolic acidosis D. Weight gain E. Decreased absorption of amino acids 6. A patient with a tumor secreting abnormal amounts of gastrin (gastrinoma) would be most likely to exhibit which of the ff? A. decreased chief cell exocytosis B. duodenal ulceration C. increased gastric pH in the period between meals D. a reduced incidence of gastroesophageal reflux disease E. protein malabsoprtion

ANSWER: D



Zollinger–Ellison syndrome is a syndrome is seen in patients with gastrinomas. Gastrin causes prolonged hypersecretion of acid, and severe ulcers are produced. Ganong. Review of Medical Physiology 23rd ed. Chapter 26, page 433 .

ANSWER: C • • • • •

Gastric juice  pH 1-3 MOST ACIDIC Colonic luminal contents  pH 5-7  Pancreatic juice  pH 8 MOST BASIC Saliva pH 6.5-7  Contents of the intestinal crypts  pH 5-7  Ganong. Review of Medical Physiology 23rd ed. Chapter 26, page 438 .

ANSWER: D Pancreatectomy  loss of pancreatic secretions steatorrhea and protein malabsorption. Pancreatectomy  loss of insulin secretion high  glucagon/insulin ratio hyperglycemia Pancreatectomy  loss of insulin secretion decreased intracellular uptake of glucose and increase protein and fat catabolism weight loss. •





Ganong. Review of Medical Physiology 23rd ed. Chapter 21, page 324.

ANSWER: C •

Zollinger–Ellison syndrome is a syndrome is seen in patients with gastrinomas. Gastrin causes prolonged hypersecretion of acid, and severe ulcers are produced. Ganong. Review of Medical Physiology 23rd ed. Chapter 26, page 433 ..

TOPNOTCH MEDICAL BOARD PREP MED PHYSIOLOGY GANONG SUPPLEMENT HANDOUT BY THE TOPNOTCH TEAM Page 32 of 46 For inquiries visit www.topnotchboardprep.com.ph or https://web.facebook.com/topnotchmedicalboardprep/

TOPNOTCH MEDICAL BOARD PREP PHYSIOLOGY GANONG SUPPLEMENT HANDOUT BY THE TOPNOTCH TEAM For inquiries visit www.topnotchboardprep.com.ph or https://www.facebook.com/topnotchmedicalboardprep/

(CHAPTER 26) DIGESTION, ABSORPTION, & NUTRITIONAL PRINCIPLES 1. Maximum absorption of short-chain fatty acids produced by bacteria occurs in the A. stomach. B. duodenum. C. jejunum. D. ileum. E. colon. 2. A premenopausal woman who is physically active seeks advice from her primary care clinician regarding measures she can take to ensure adequate availability of dietary calcium to ensure bone health later in life. Which of the following dietary components should enhance calcium uptake? A. Protein B. Oxalates C. Iron D. Vitamin D E. Sodium 3. A decrease in which of the following would be expected in a child exhibiting a congenital absence of enterokinase? A. Incidence of pancreatitis B. Glucose absorption C. Bile acid reabsorption D. Gastric pH E. Protein assimilation

ANSWER:

 Short-chain fatty acids are absorbed by specific transporters present in colonic epithelial cells. SCFAs make a significant contribution to the total caloric intake and exert a trophic effect on the colonic epithelial cells, combat inflammation, and maintain acid–base equilibrium.



Ganong. Review of Medical Physiology 23rd ed. Chapter 27 , page 458

ANSWER: D



Role of vitamin D derivative: Ca2+ absorption is adjusted to body needs; absorption is increased in the presence of Ca2+ deficiency and decreased in the presence of Ca2+ excess. Ganong. Review of Medical Physiology 23rd ed. Chapter 27 , page 458 .

ANSWER: E The protein assimilation machinery, which rests heavily on the proteases in pancreatic juice, is arranged such that these enzymes are not activated until they reach their substrates in the small intestinal lumen. This is accomplished by the restricted localization of an activating enzyme, enterokinase. •

Ganong. Review of Medical Physiology 23rd ed. Chapter 27 , page 466.

ANSWER: A Hartnup disease is a congenital defect in the transport of neutral amino acids does not lead to nutritional deficiencies of these amino acids because peptide transport ( PepT1) compensates. •

4. In Hartnup disease (a defect in the transport of neutral amino acids), patients do not become deficient in these amino acids due to the activity of: A. PepT1. B. brush border peptidases. C. Na+, K+ ATPase. D. cystic fibrosis transmembrane conductance regulator (CFTR). E. trypsin.

Ganong. Review of Medical Physiology 23rd ed. Chapter 27 , page 456..

5. A newborn baby is brought to the pediatrician suffering from severe diarrhea that worsens with meals. The symptoms diminish when nutrients are delivered intravenously. The child most likely has a mutation in which of the following intestinal transporters? A. Na+, K+ ATPase B. NHE3 C. SGLT-1 D. H+, K+ ATPase E. NKCC1 6. Calcium absorption is increased by: A. hypercalcemia B. oxalates in the diet C. iron overload D. 1,25- dihydroxycholecalciferol E. increased Na+ absorption

ANSWER: C •

“When the sodium-dependent glucose transporter (SGLT-1) is congenitally defective, the resulting glucose/galactose malabsorption causes severe diarrhea that is often fatal if glucose and galactose are not promptly removed from the diet.” Ganong. Review of Medical Physiology 23rd ed. Chapter 27 , page 453 .

ANSWER: D •

Role of 1,25- dihydroxycholecalciferol : Ca2+ absorption is adjusted to body needs; absorption is increased in the presence of Ca2+ deficiency and decreased in the presence of Ca2+ excess. Ganong. Review of Medical Physiology 23rd ed. Chapter 27 , page 458 .

(CHAPTER 27) GASTROINTESTINAL MOTILITY 1. In infants, defecation often follows a meal. The cause of colonic contractions in this situation is A. histamine. B. increased circulating levels of CCK. C. the gastrocolic reflex. D. increased circulating levels of somatostatin. E. the enterogastric reflex.

ANSWER: C Distention of the stomach by food initiates contractions of the rectum and, frequently, a desire to defecate. The response is called the gastrocolic reflex and may be amplified by an action of gastrin on the colon. Because of the response, defecation after meals is the rule in children. In adults, habit and cultural factors play a large role in determining when defecation occurs. •



Ganong. Review of Medical Physiology 23rd ed. Chapter , page .

2. The symptoms of the dumping syndrome (discomfort after meals in patients with intestinal short circuits such as anastomosis of the jejunum to the stomach) are caused in part by A. increased blood pressure. B. increased secretion of glucagon. C. increased secretion of CCK. D. hypoglycemia. E. hyperglycemia.

ANSWER: D •

“Dumping syndrome,” is a distressing syndrome that develops in patients in whom portions of the stomach have been removed or the jejunum has been anastomosed. Symptoms of weakness, dizziness, and sweating after meals is due to the hypoglycemia about 2 h after meals. Ganong. Review of Medical Physiology 23rd ed. Chapter 28 , page 474 .

TOPNOTCH MEDICAL BOARD PREP MED PHYSIOLOGY GANONG SUPPLEMENT HANDOUT BY THE TOPNOTCH TEAM Page 33 of 46 For inquiries visit www.topnotchboardprep.com.ph or https://web.facebook.com/topnotchmedicalboardprep/

TOPNOTCH MEDICAL BOARD PREP PHYSIOLOGY GANONG SUPPLEMENT HANDOUT BY THE TOPNOTCH TEAM For inquiries visit www.topnotchboardprep.com.ph or https://www.facebook.com/topnotchmedicalboardprep/ 3. Gastric pressures seldom rise above the levels that breach the lower esophageal sphincter, even when the stomach is filled with a meal, due to which of the following processes? A. Peristalsis B. Gastroileal reflex C. Segmentation D. Stimulation of the vomiting center E. Receptive relaxation

ANSWER: •

The stomach accommodates the meal by a process of receptive relaxation. When food enters the stomach, the  fundus and upper portion of the body   relax, this permits an increase in volume without a significant increase in pressure.

Ganong. Review of Medical Physiology 23rd ed. Chapter 28 , page 473

ANSWER: A Motilin released cyclically and stimulates interdigestive myoelectric complexes from stomach and small intestine every 90 minutes in a fasted  person. NO intestinal smooth muscle relxation. CCK strongly contracts the gallbladder , expelling bile into the small intestine where the bile in turn plays important roles in emulsifying fatty substances.  Somatostatin inhibits intestinal secretions.  Secretin promote pancreatic secretion of bicarbonate which in turn helps to neutralize the acid in the small intestine. •

4. The migrating motor complex is triggered by which of the following? A. Motilin B. NO C. CCK D. Somatostatin E. Secretin

• •

• •

Guyton.Textbook of Medical Physiology 11th ed.Chap62 , page 776

5. A patient is referred to a gastroenterologist because of persistent difficulties with swallowing. Endoscopic examination reveals that the lower esophageal sphincter fails to fully open as the bolus reaches it, and a diagnosis of achalasia is made. During the examination, or in biopsies taken from the sphincter region, a decrease would be expected in which of the following? A. Esophageal peristalsis B. Expression of neuronal NO synthase C. Acetylcholine receptors D. Substance P release E. Contraction of the crural diaphragm

ANSWER: B



In achalasia, the myenteric plexus of the esophagus is deficient at the LES in this condition and the release of NO and VIP is defective, therefore there will be no anterograde relaxation or relaxation ahead of the stimulus. Ganong. Review of Medical Physiology 23rd ed. Chapter 28, page 470, 473.

(CHAPTER 28) TRANSPORT & METABOLIC FUNCTIONS OF THE LIVER 1. A patient suffering from severe ulcerative colitis undergoes a ANSWER: E total colectomy with formation of a stoma. After a full recovery Colectomy  antibiotic bowel preparation + resection of the from surgery, and compared to his condition prior to surgery, entire colon eradicate intestinal bacteria and colonic which of the following would be expected to be decreased? bacteria  deconjugation of bilirubin  absorption of A. Ability to absorb lipids uribilinogen urobilinogen excretion in the kidneys. B. Ability to clot the blood Lipids, fat soluble vitamin(vitamin K) reabsorption is not C. Circulating levels of conjugated bile acids altered since the patient had intact intestinal mucosa D. Urinary urea Ganong. Review of Medical Physiology 23rd ed. Chapter 29, page . 483. E. Urinary urobilinogen ANSWER: E 2. A surgeon is studying new methods of liver transplantation. She Estrogen levels are expected to rise since estrogen is converted to performs a complete hepatectomy in an experimental animal.  glucuronide and sulfate conjugates. All these compounds, along with Before the donor liver is grafted, a rise in the blood level of which other metabolites, are excreted in the urine. of the following would be expected? The other choices are expected to fall because: A. Glucose o Glucose: no liver glycogen source B. Fibrinogen o Fibrinogen: no synthesis C. 25-Hydroxycholecalciferol o 25-Hydroxycholecalciferol  : absence of liver enzyme 24hydroxylase. D. Conjugated bilirubin o Conjugated bilirubin: absence of liver UDP-glucoronidase E. Estrogens Ganong. Review of Medical Physiology 23rd ed. Chapter 25 , page 412.. •







3. Which of the following cell types protects against sepsis secondary to translocation of intestinal bacteria? A. Hepatic stellate cell B. Cholangiocyte C. Kupffer cell D. Hepatocyte E. Gallbladder epithelial cell

ANSWER: C • • •

• •

Hepatic stellate cell : formation of fibrosis Cholangiocyte: absorptive and secretory function. Kupffer cell : prevents bacterial translocation by its phagocytic activity Hepatocyte: synthesis of various compounds, & detoxification. Gallbladder  epithelial cell: concentrates bile Ganong. Review of Medical Physiology 23rd ed. Chapter 29 , page . 486.

4. P450s (CYPs) are highly expressed in hepatocytes. In which of the following do they not play an important role? A. Bile acid formation B. Carcinogenesis C. Steroid hormone formation D. Detoxification of drugs E. Glycogen synthesis

ANSWER: E

5. A 40-year-old woman comes to her primary care clinician complaining of severe, episodic abdominal pain that is particularly intense after she ingests a fatty meal. An imaging procedure reveals that her gallbladder is acutely dilated, and a diagnosis of cholelithiasis is made. A gallstone lodged in which location will also increase her risk of pancreatitis?

ANSWER: E

A. Left hepatic duct B. Right hepatic duct C. Cystic duct D. Common bile duct E. Sphincter of Oddi



Glycogen synthesis is the function of  glycogen synthase and not the CYP450. Ganong. Review of Medical Physiology 23rd ed. Chapter 1 , page 21..

The most common cause of pancreatitis is drinking excess alcohol, and the second most common cause is blockage of the  papilla of Vater by a gallstone; the two together account for more than 90 per cent of all cases.



Guyton.Textbook of Medical Physiology 11th ed.Chapter 66, page 821.

TOPNOTCH MEDICAL BOARD PREP MED PHYSIOLOGY GANONG SUPPLEMENT HANDOUT BY THE TOPNOTCH TEAM Page 34 of 46 For inquiries visit www.topnotchboardprep.com.ph or https://web.facebook.com/topnotchmedicalboardprep/

TOPNOTCH MEDICAL BOARD PREP PHYSIOLOGY GANONG SUPPLEMENT HANDOUT BY THE TOPNOTCH TEAM For inquiries visit www.topnotchboardprep.com.ph or https://www.facebook.com/topnotchmedicalboardprep/ ANSWER: B The concentration of  sodium ions rises with a concomitant loss of chloride and bicarbonate as the bile is concentrated in the gallbladder. Other substances which are highly concentrated in the gallbladder bile: o Calcium ions o Bile salts o Cholesterol o Lecithin •

6. Compared to hepatic bile, gallbladder bile contains a reduced concentration of which of the following? A. Bile acids B. Chloride ions C. Protons D. Glucose E. Calcium ions



Ganong. Review of Medical Physiology 23rd ed., Chapter 29 page .486 Guyton.Textbook of Medical Physiology 11th ed.Chapter 64, page 587, 803.

ANSWER: E Colectomy  antibiotic bowel preparation + resection of the entire colon eradicate intestinal bacteria and colonic bacteria  decreased deconjugation of bilirubin decreased absorption of uribilinogen decreased urobilinogen excretion in the kidneys.  Anemia due to decreased B12 and malnutrition because of protein malabsorption is not a consequence because both are reabsorb in the  small intestines.  Ammonia is handled by the liver, and levels will increase in cirrhosis. •

7. Removal of the entire colon would be expected to cause: A. death B. megaloblastic anemia C. severe malnutrition D. a decrease in the blood level of ammonia in patients with cirrhosis of the liver E. decreased urinary urobilinogen





Ganong. Review of Medical Physiology 23rd ed. Chapter 29, page . 483

SECTION 5: CARDIOVASCULAR PHYSIOLOGY (CHAPTER 29) ORIGIN OF THE HEARTBEAT & THE ELECTRICAL ACTIVITY OF THE HEART 1. Which part of the ECG (eg, Figure 29 –5) corresponds to ventricular repolarization? A. The P wave B. The QRS duration C. The T wave D. The U wave E. The PR interval 2. Which of the following normally has a slowly depolarizing “prepotential”? A. Sinoatrial node B. Atrial muscle cells C. Bundle of His D. Purkinje fibers E. Ventricular muscle cells 3. In second-degree heart block A. the ventricular rate is lower than the atrial rate. B. the ventricular ECG complexes are distorted. C. there is a high incidence of ventricular tachycardia. D. stroke volume is decreased. E. cardiac output is increased. 4. Currents caused by opening of which of the following channels contribute to the repolarization phase of the action potential of ventricular muscle fibers? A. Na+ channels B. Cl− channels C. Ca2+ channels D. K+ channels E. HCO3− channels 5. In complete heart block A. fainting may occur because the atria are unable to pump blood into the ventricles. B. ventricular fibrillation is common. C. the atrial rate is lower than the ventricular rate. D. fainting may occur because of prolonged periods during which the ventricles fail to contract.

ANSWER: C The P wave: atrial depolarization The QRS duration: ventricular depolarization The T wave: ventricular repolarization The U wave: inconstant finding, prominent in hypokalemia. The PR interval : time between atrial depolarization and conduction through AV node. • • • • •

Ganong. Review of Medical Physiology 23rd ed. Chapter 30, page 492 ..

ANSWER: A •

“Pacemaker  or prepotentials are normally prominent only in the SA and AV nodes, wherein there is no sharp, rapid depolarizing spike before the plateau ass there is in other parts of the conduction system and the atrial and ventricular fibers.” Ganong. Review of Medical Physiology 23rd ed. Chapter 30 , page 491 .

ANSWER: A •

In second-degree heart block , not all atrial impulses are conducted to the ventricles. For example, a ventricular beat may follow every second or every third atrial beat (2:1 block, 3:1 block, etc). This would lead to ventricular rates which is lower than the atrial. Ganong. Review of Medical Physiology 23rd ed. Chapter 30, page 497 .

ANSWER: D Initial depolarization: Na + influx through rapidly opening Na +channels (the Rapid repolarization: inactivation of Na+ channels Plateau phase: Ca2+ influx through more slowly opening Ca2+ channels . Repolarization: net K+ efflux through multiple types of K+ channels •

• •



Ganong. Review of Medical Physiology 23rd ed. Chapter 30 , page 491 ..

ANSWER: D •

Individuals with complete heart block, ventricles beat at a low rate independently of the atria, there may also be periods of asystole lasting a minute or more. The resultant cerebral ischemia causes dizziness and fainting (Stokes– Adams syndrome) Ganong. Review of Medical Physiology 23rd ed. Chapter 30 , page 497 .

TOPNOTCH MEDICAL BOARD PREP MED PHYSIOLOGY GANONG SUPPLEMENT HANDOUT BY THE TOPNOTCH TEAM Page 35 of 46 For inquiries visit www.topnotchboardprep.com.ph or https://web.facebook.com/topnotchmedicalboardprep/

TOPNOTCH MEDICAL BOARD PREP PHYSIOLOGY GANONG SUPPLEMENT HANDOUT BY THE TOPNOTCH TEAM For inquiries visit www.topnotchboardprep.com.ph or https://www.facebook.com/topnotchmedicalboardprep/

(CHAPTER 30) THE HEART AS A PUMP 1. The second heart sound is caused by A. closure of the aortic and pulmonary valves. B. vibrations in the ventricular wall during systole. C. ventricular filling. D. closure of the mitral and tricuspid valves. E. retrograde flow in the vena cava.

ANSWER: A

2. The fourth heart sound is caused by A. closure of the aortic and pulmonary valves. B. vibrations in the ventricular wall during systole. C. ventricular filling. D. closure of the mitral and tricuspid valves. E. retrograde flow in the vena cava.

ANSWER: A

3. The dicrotic notch on the aortic pressure curve is caused by A. closure of the mitral valve. B. closure of the tricuspid valve. C. closure of the aortic valve. D. closure of the pulmonary valve. E. rapid filling of the left ventricle.

ANSWER: C

 S2: closure of the aortic and pulmonary valves.  S4: vibrations in the ventricular wall during systole.  S3: ventricular filling.  S1: closure of the mitral and tricuspid valves.

• • • •

Ganong. Review of Medical Physiology 23rd ed. Chapter 31 , page 513..

 S3: ventricular filling.  S2: closure of the aortic and pulmonary valves.  S4: vibrations in the ventricular wall during systole.  S1: closure of the mitral and tricuspid valves.

• • • •

Ganong. Review of Medical Physiology 23rd ed. Chapter 31 , page 513.



“The dicrotic notch, a small oscillation on the falling phase of the pulse wave caused by vibrations set up when the aortic valve snaps shut.” Ganong. Review of Medical Physiology 23rd ed. Chapter 31, page 512..

ANSWER: C 4. During exercise, a man consumes 1.8 L of oxygen per minute. His arterial O2 content is 190 mL/L, and the O2 content of his mixed venous blood is 134 mL/L. His cardiac output is approximately A. 3.2 L/min. B. 16 L/min. C. 32 L/min. D. 54 L/min. E. 160 mL/min. Ganong. Review of Medical Physiology 23rd ed. Chapter 31 , page 513 .

5. The work performed by the left ventricle is substantially greater than that performed by the right ventricle, because in the left ventricle A. the contraction is slower. B. the wall is thicker. C. the stroke volume is greater. D. the preload is greater. E. the afterload is greater.

ANSWER: E

6. Starling’s law of the heart  A. does not operate in the failing heart. B. does not operate during exercise. C. explains the increase in heart rate produced by exercise. D. explains the increase in cardiac output that occurs when venous return is increased. E. explains the increase in cardiac output when the sympathetic nerves supplying the heart are stimulated.

ANSWER: D

In terms of Preload (volume): right heart > left heart In terms of Afterload (pressure): left heart > right heart Pressure work produces a greater increase in O2 consumption than Volume work .  Afterload   causes a greater increase in cardiac O2 consumption than does an increase in Preload .

• • •



Ganong. Review of Medical Physiology 23rd ed. Chapter 31 , page 519 .



Increase venous return  cardiac muscle stretched to optimal length optimal overlap of actin and myosin crossbridges increase in force of contraction of cardiac muscle  increase cardiac output. Guyton.Textbook of Medical Physiology 11th ed.Chapter 9 , page 112..

(CHAPTER 31) BLOOD AS A CIRCULATORY FLUID & THE DYNAMICS OF BLOOD & LYMPH FLOW ANSWER: C 1. Which of the following has the highest total cross-sectional area in the body? A. Arteries B. Arterioles C. Capillaries D. Venules E. Veins Guyton.Textbook of Medical Physiology 11th ed.Chapter 14, page 162

ANSWER: B 2. Lymph flow from the f oot is A. increased when an individual rises from the supine to the standing position. B. increased by massaging the foot. C. increased when capillary permeability is decreased. D. decreased when the valves of the leg veins are incompetent. E. decreased by exercise.



Flow in the collecting lymphatics INCREASED by : o movements of skeletal muscle (by massaging/contraction ) as with foot massage or exercise. o negative intrathoracic pressure during inspiration o suction effect of high velocity flow of blood in the veins in which the lymphatics terminate. o Increase capillary permeability Ganong. Review of Medical Physiology 23rd ed. Chapter 31 , page 513..

TOPNOTCH MEDICAL BOARD PREP MED PHYSIOLOGY GANONG SUPPLEMENT HANDOUT BY THE TOPNOTCH TEAM Page 36 of 46 For inquiries visit www.topnotchboardprep.com.ph or https://web.facebook.com/topnotchmedicalboardprep/

TOPNOTCH MEDICAL BOARD PREP PHYSIOLOGY GANONG SUPPLEMENT HANDOUT BY THE TOPNOTCH TEAM For inquiries visit www.topnotchboardprep.com.ph or https://www.facebook.com/topnotchmedicalboardprep/ ANSWER: D Fluid movement = k[(Pc – Pi) – (πc – πi)] where o k = capillary filtration coefficient o Pc = capillary hydrostatic pressure 35-14= 11mmHg o Pi = interstitial hydrostatic pressure 0 o πc = capillary colloid osmotic pressure 25mmHg o πi = interstitial colloid osmotic pressure 1mmHg •

3. The pressure in a capillary in skeletal muscle is 35 mm Hg at the arteriolar end and 14 mm Hg at the venular end. The interstitial pressure is 0 mm Hg. The colloid osmotic pressure is 25 mm Hg in the capillary and 1 mm Hg in the interstitium. The net force producing fluid movement across the capillary wall at its arteriolar end is A. 3 mm Hg out of the capillary. B. 3 mm Hg into the capillary. C. 10 mm Hg out of the capillary. D. 11 mm Hg out of the capillary. E. 11 mm Hg into the capillary



Ganong. Review of Medical Physiology 23rd ed. Chapter 32 , page . 458

ANSWER: B The average velocity  of fluid movement at any point in a system of tubes in parallel is inversely  proportional  to the total cross-sectional area. Referring to the illustration below, veins had greater velocity than venules because it has smaller cross sectional area. •



4. The velocity of blood flow A. is higher in the capillaries than the arterioles. B. is higher in the veins than in the venules. C. is higher in the veins than the arteries. D. falls to zero in the descending aorta during diastole. E. is reduced in a constricted area of a blood vessel.

Ganong. Review of Medical Physiology 23rd ed. Chapter 32, page 541.

ANSWER: E  flow   varies DIRECTLY  and resistance INVERSELY  with the fourth power of the radius . •

5. When the radius of the resistance vessels is increased, which of the following is increased? A. Systolic blood pressure B. Diastolic blood pressure C. Viscosity of the blood D. Hematocrit E. Capillary blood flow



• • • • •

in which:

F is the rate of blood flow, DP is the pressure difference r  is the radius of the vessel l  is length of the vessel h is viscosity of the blood. Ganong. Review of Medical Physiology 23rd ed. Chapter , page .

6. A 30-year-old patient comes to her primary care clinician complaining of headaches and vertigo. A blood test reveals a hematocrit of 55%, and a diagnosis of polycythemia is made. Which of the following would also be increased? A. Mean blood pressure B. Radius of the resistance vessels C. Radius of the capacitance vessels D. Central venous pressure E. Capillary blood flow

ANSWER: A Increased hematocrit  increased viscosity  Increase resistance increase mean blood pressure •



• • • • •

in which:

R is resistance , DP is the pressure  difference r  is the radius of the vessel l  is length of the vessel h is viscosity  of the blood. Ganong. Review of Medical Physiology 23rd ed. Chapter 32 , page 542 .

7. A pharmacologist discovers a drug that stimulates the production of VEGF receptors. He is excited because the drug might be of value in the treatment of A. coronary artery disease. B. cancer. C. emphysema. D. diabetes insipidus. E. dysmenorrhea.

ANSWER: A

8. Why is the dilator response to injected acetylcholine changed to a constrictor response when the endothelium is damaged? A. More Na+ is generated. B. More bradykinin is generated. C. The damage lowers the pH of the remaining layers of the artery. D. The damage augments the production of endothelin by the endothelium. E. The damage interferes with the production of NO by the Endothelium

ANSWER: E





VEGF agonist responsible for vasculogenesis or development of new blood supply that will help bypass the blocked arteries as seen in coronary artery disease. VEGF antagonists and other angiogenesis inhibitors have now entered clinical practice as adjunctive therapies for many malignancies Ganong. Review of Medical Physiology 23rd ed. Chapter 32, page . 539





Acetylcholine act directly on vascular smooth muscle would produce much greater constriction if their effects were not restricted by simultaneous release of NO from the intact blood vessels. Damage endothelium NO is not released  absence of relaxation of vascular smooth muscle  predominance of vasoconstriction effect of acetylcholine. Ganong. Review of Medical Physiology 23rd ed. Chapter 33 , page 564 .

TOPNOTCH MEDICAL BOARD PREP MED PHYSIOLOGY GANONG SUPPLEMENT HANDOUT BY THE TOPNOTCH TEAM Page 37 of 46 For inquiries visit www.topnotchboardprep.com.ph or https://web.facebook.com/topnotchmedicalboardprep/

TOPNOTCH MEDICAL BOARD PREP PHYSIOLOGY GANONG SUPPLEMENT HANDOUT BY THE TOPNOTCH TEAM For inquiries visit www.topnotchboardprep.com.ph or https://www.facebook.com/topnotchmedicalboardprep/

(CHAPTER 32) CARDIOVASCULAR REGULATORY MECHANISMS 1. When a pheochromocytoma (tumor of the adrenal medulla) suddenly discharges a large amount of epinephrine into the circulation, the patient's heart rate would be expected to A. increase because the increase in blood pressure stimulates the carotid and aortic baroreceptors. B. increase because epinephrine has a direct chronotropic effect on the heart. C. increase because of increased tonic parasympathetic discharge to the heart. D. decrease because the increase in blood pressure stimulates the carotid and aortic chemoreceptors. E. decrease because of increased tonic parasympathetic discharge to the heart.

ANSWER: B

2. Orthostatic hypotension due to a malfunction in the baroreceptor reflex was diagnosed in a 65-year-old man who had been experiencing frequent episodes of syncope as he got out of bed in the mornings. Activation of the baroreceptor reflex A. is primarily involved in short-term regulation of systemic blood pressure. B. leads to an increase in heart rate because of inhibition of the vagal cardiac motor neurons. C. inhibits neurons in the CVLM. D. excites neurons in the RVLM. E. occurs only under situations in which blood pressure is markedly elevated.

ANSWER: A

3. A 45-year-old woman had a blood pressure of 155/95 mm Hg when she was at her clinician's office for a physical. It was her first time to see this clinician and her first physical in over 10 years. The clinician suggested that she begin monitoring her blood pressure at home. Sympathetic nerve activity would be expected to increase A. if glutamate receptors were activated in the NTS. B. if GABA receptors were activated in the RVLM. C. if glutamate receptors were activated in the CVLM. D. during stress. E. when one transitions from an erect to a supine posture.

ANSWER: D Glutamate: stimulatory GABA: inhibitory Glutamate receptors were activated in the NTS  glutamate receptors were activated in the CVLM  GABA receptors were activated in the RVLM  decrease sympathetic output . Sympathetic signals are increased during sexual excitement, anger, and stress thru descending tracts to the vasomotor area from the cerebral cortex (particularly the limbic cortex ).

4. Which of the following neurotransmitters are correctly matched with an autonomic pathway? A. GABA is released by NTS neurons projecting to the RVLM. B. Glutamate is released by CVLM neurons projecting to the IML C. GABA is released by NTS neurons projecting to the nucleus ambiguus. D. GABA is released by CVLM neurons projecting to the RVLM. E. Glutamate is released by CVLM neurons projecting to the NTS

ANSWER: D

5. A 53-year-old woman with chronic lung disease was experiencing difficulty breathing. Her arterial PO2 and PCO2 were 50 mm Hg and 60 mm Hg, respectively. Which one of the following statements about chemoreceptors is correct? A. Peripheral chemoreceptors are very sensitive to small increases in arterial PCO2. B. Activation of arterial chemoreceptors leads to a fall in arterial pressure. C. Peripheral chemoreceptors are located in the NTS. D. Central chemoreceptors can be activated by an increase in intracranial pressure that compromises blood flow in the medulla. E. Central chemoreceptors are activated by increases in tissue pH

ANSWER: D

6. Why is the dilator response to injected acetylcholine change to a constrictor response when the endothelium is damaged? A. More Na+ is generated B. More bradykinin is generated C. The damage lowers the pH of the remaining layers of the artery D. The damage augments the production of endothelin by the endothelium E. The damage interferes with the production of NO by the endothelium

ANSWER: E





Pheochromocytoma large amount of epinephrine released  direct chronotropic effect  increase in heart rate. Low blood pressure stimulates the baroreceptor  to increase the heart rate. Ganong. Review of Medical Physiology 23rd ed. Chapter 31, page 514 .



Baroreceptors are very important in short-term control of arterial pressure. Activation of the reflex allows for rapid adjustments in blood pressure in response to abrupt changes in blood volume, cardiac output, or peripheral resistance. Ganong. Review of Medical Physiology 23rd ed. Chapter 33, page 561 .

• • •



Ganong. Review of Medical Physiology 23rd ed. Chapter 33 , page 557.

• • •

Glutamate: stimulatory GABA: inhibitory Glutamate receptors were activated in the NTS  glutamate receptors were activated in the CVLM  GABA receptors were activated in the RVLM  decrease sympathetic output . Ganong. Review of Medical Physiology 23rd ed. Chapter 33 , page 557.



Increase intracranial pressuredecreased medullary blood flow decreased oxygen and excess buildup of carbon dioxide and hydrogen ions  central chemoreceptors become stimulated. Guyton.Textbook of Medical Physiology 11th ed.Chapter 18 , page 212 .





Acetylcholine act directly on vascular smooth muscle would produce much greater constriction if their effects were not restricted by simultaneous release of NO from the intact blood vessels. Damage endothelium NO is not released  absence of relaxation of vascular smooth muscle  predominance of vasoconstriction effect of acetylcholine. Ganong. Review of Medical Physiology 23rd ed. Chapter 33 , page 564 .

TOPNOTCH MEDICAL BOARD PREP MED PHYSIOLOGY GANONG SUPPLEMENT HANDOUT BY THE TOPNOTCH TEAM Page 38 of 46 For inquiries visit www.topnotchboardprep.com.ph or https://web.facebook.com/topnotchmedicalboardprep/

TOPNOTCH MEDICAL BOARD PREP PHYSIOLOGY GANONG SUPPLEMENT HANDOUT BY THE TOPNOTCH TEAM For inquiries visit www.topnotchboardprep.com.ph or https://www.facebook.com/topnotchmedicalboardprep/

(CHAPTER 33) CIRCULATION THROUGH SPECIAL REGIONS 1. Blood in which of the following vessels normally has the lowest Po2? A. Maternal artery B. Maternal uterine vein C. Maternal femoral vein D. Umbilical artery E. Umbilical vein

ANSWER: • • •

Maternal artery : 98% oxygen saturation Umbilical vein: 80% oxygen saturation Umbilical artery: 60% oxygen saturation Ganong. Review of Medical Physiology 23rd ed. Chapter 34, page .582.

ANSWER: A

2. The pressure differential between the heart and the aorta is least in the A. left ventricle during systole. B. left ventricle during diastole. C. right ventricle during systole. D. right ventricle during diastole. E. left atrium during systole.

Ganong. Review of Medical Physiology 23rd ed. Chapter 34 , page 579 ..

3. Injection of tissue plasminogen activator (t-PA) would probably be most beneficial: A. after at least 1 year of uncomplicated recovery following occlusion of a coronary artery. B. after at least 2 months of rest and recuperation following occlusion of a coronary artery. C. during the second week after occlusion of a coronary artery. D. during the second day after occlusion of a coronary artery. E. during the second hour after occlusion of a coronary artery. 4. Which of the following organs has the greatest blood flow per 100 g of tissue? A. Brain B. Heart muscle C. Skin D. Liver E. Kidneys

ANSWER:

Tissue Plasminogen Activator : o Effective in activating plasminogen to plasmin. o “Dissolve some intravascular clots (if used within the  first hour or so after thrombotic occlusion) and heart is often spared serious damage.” Guyton.Textbook of Medical Physiology 11th ed. Chapter 36, page 466.



ANSWER: E On a per gram weight basis, the kidneys normally consume oxygen at twice the rate of the brain but have almost  seven times the blood flow of the brain . Thus, the oxygen delivered to the kidneys far exceeds their metabolic needs, and the arterialvenous extraction of oxygen is relatively low compared with that of most other tissues. •

Ganong. Review of Medical Physiology 23rd ed. Chapter 26, page 320. Guyton.Textbook of Medical Physiology 11th ed.Chapter 26, page 320.

5. Which of the following does not dilate arterioles in the skin? A. Increased body temperature B. Epinephrine C. Bradykinin D. Substance P E. Vasopressin

ANSWER: E

6. A baby boy is brought to the hospital because of convulsions. In the course of a workup, his body temperature and plasma glucose are found to be normal, but his cerebrospinal fluid glucose is 12 mg/dL (normal, 65 mg/dL). A possible explanation of his condition is: A. constitutive activation of GLUT3 in neurons. B. SGLT-1 deficiency in astrocytes. C. GLUT5 deficiency in cerebral capillaries. D. GLUT1 55K deficiency in cerebral capillaries. E. GLUT1 45K deficiency in microglia.

ANSWER: D



Vasopressin is even more powerful than angiotensin II as a vasoconstrictor , thus making it one of the body’s most potent vascular constrictor substances. Guyton.Textbook of Medical Physiology 11th ed.Chapter 17 , page 202.



Infants with congenital GLUT1 55K deficiency develop low CSF glucose concentrations in the presence of normal plasma glucose, and they have seizures and delayed development . Ganong. Review of Medical Physiology 23rd ed. Chapter 34, page 573..

TOPNOTCH MEDICAL BOARD PREP MED PHYSIOLOGY GANONG SUPPLEMENT HANDOUT BY THE TOPNOTCH TEAM Page 39 of 46 For inquiries visit www.topnotchboardprep.com.ph or https://web.facebook.com/topnotchmedicalboardprep/

TOPNOTCH MEDICAL BOARD PREP PHYSIOLOGY GANONG SUPPLEMENT HANDOUT BY THE TOPNOTCH TEAM For inquiries visit www.topnotchboardprep.com.ph or https://www.facebook.com/topnotchmedicalboardprep/

SECTION 6: RESPIRATORY PHYSIOLOGY (CHAPTER 34) INTRODUCTION TO PULMONARY STRUCTURE & MECHANICS 1. On the summit of Mt. Everest, where the barometric pressure is about 250 mm Hg, the partial pressure of O2 in mm Hg is about: A. 0.1 B. 0.5 C. 5 D. 50 E. 100

ANSWER: D

2. The forced vital capacity is A. the amount of air that normally moves into (or out of) the lung with each respiration. B. the amount of air that enters the lung but does not participate in gas exchange. C. the amount of air expired after maximal expiratory effort. D. the largest amount of gas that can be moved into and out of the lungs in 1 min.

ANSWER: C





The composition of dry air is 20.98% O2, 0.04% CO 2, 78.06% N2, and 0.92% other inert constituents such as argon and helium. The partial pressure of O2 in dry air is therefore 0.20 × 250, or 50 mmHg at the level of Mt. Everest. Ganong. Review of Medical Physiology 23rd ed. Chapter 35 , page 588.



The forced vital capacity (FVC), the largest amount of air that can be expired after a maximal inspiratory effort , is frequently measured clinically as an index of pulmonary function. Ganong. Review of Medical Physiology 23rd ed. Chapter 35 , page 593 .

ANSWER: A Tidal Volume: the amount of air that normally moves into (or out of) the lung with each respiration. Dead space: the amount of air that enters the lung but does not participate in gas exchange. Expiratory Reserve Volume: the amount of air expired after maximal expiratory effort. FEV1: fraction of the vital capacity expired during the first second of a forced expiration. •



3. The tidal volume is A. the amount of air that normally moves into (or out of) the lung with each respiration. B. the amount of air that enters the lung but does not participate in gas exchange. C. the amount of air expired after maximal expiratory effort. D. the amount of gas that can be moved into and out of the lungs in 1 min.





*Respiratory volume and capacities: see below

4. Which of the following is responsible for the movement of O2 from the alveoli into the blood in the pulmonary capillaries? A. Active transport B. Filtration C. Secondary active transport D. Facilitated diffusion E. Passive diffusion 5. Airway resistance A. is increased if the lungs are removed and inflated with saline. B. does not affect the work of breathing. C. is increased in paraplegic patients. D. is increased following bronchial smooth muscle contraction. E. makes up 80% of the work of breathing.

Ganong. Review of Medical Physiology 23rd ed. Chap35, page 593 ANSWER: E •

Gases moves thru  passive diffusion from the alveoli to the blood in the pulmonary capillaries or vice versa across the thin alveolocapillary membrane. Ganong. Review of Medical Physiology 23rd ed. Chapter 35 , page 601.

ANSWER: D Airway resistance: o is increased following bronchial smooth muscle contraction.  if the lungs are removed and inflated with o is decreased  saline. (saline decreases surface tension) o does affect   the work of breathing; it makes up 28% of the work of breathing. •

Ganong. Review of Medical Physiology 23rd ed. Chapter 35, pages 597,598 ..

6. Surfactant lining the alveoli A. helps prevent alveolar collapse. B. is produced in alveolar type I cells and secreted into the alveolus. C. is increased in the lungs of heavy smokers. D. is a glycolipid complex.

ANSWER:  Surfactant : o Prevents alveolar collapse. o produced in alveolar type II cells o mixture of dipalmitoylphosphatidylcholine, other lipids, and proteins.   in the lungs of chronic smokers. o Decreased  •

Ganong. Review of Medical Physiology 23rd ed. Chapter 35, page . 597

ANSWER: A 7. Which of the following causes relaxation of bronchial smooth muscle? A. Vasointestinal Peptide B. Leukotrienes C. Acetylcholine D. Cool Air E. Sulfur Dioxide Ganong. Review of Medical Physiology 23rd ed. Chapter 35, page . 594

TOPNOTCH MEDICAL BOARD PREP MED PHYSIOLOGY GANONG SUPPLEMENT HANDOUT BY THE TOPNOTCH TEAM Page 40 of 46 For inquiries visit www.topnotchboardprep.com.ph or https://web.facebook.com/topnotchmedicalboardprep/

TOPNOTCH MEDICAL BOARD PREP PHYSIOLOGY GANONG SUPPLEMENT HANDOUT BY THE TOPNOTCH TEAM For inquiries visit www.topnotchboardprep.com.ph or https://www.facebook.com/topnotchmedicalboardprep/

(CHAPTER 35) GAS TRANSPORT & PH ANSWER: E

1. Most of the CO2 transported in the blood is A. dissolved in plasma. B. in carbamino compounds formed from plasma proteins. C. in carbamino compounds formed from hemoglobin. D. bound to Cl−. E. in HCO3−.

Guyton.Textbook of Medical Physiology 11th ed.Chapter 40 , page 510.

2. Which of the following has the greatest effect on the ability of blood to transport oxygen? A. Capacity of the blood to dissolve oxygen B. Amount of hemoglobin in the blood C. pH of plasma D. CO2 content of red blood cells E. Temperature of the blood 3. Which of the following is true of the system?

ANSWER: B •

Ganong. Review of Medical Physiology 23rd ed. Chapter 32, page 425

ANSWER: D •

A. Reaction 2 is catalyzed by carbonic anhydrase. B. Because of reaction 2, the pH of blood declines during hyperventilation. C. Reaction 1 occurs in the red blood cell. D. Reaction 1 occurs primarily in plasma. E. The reactions move to the right when there is excess H+ in the tissues. 4. In comparing uncompensated respiratory acidosis & uncompensated metabolic acidosis which one of the following is true? A. Plasma pH change is always greater in uncompensated respiratory acidosis compared to uncompensated metabolic acidosis. B. There are no compensation mechanisms for respiratory acidosis, whereas there is respiratory compensation for metabolic acidosis. C. Uncompensated respiratory acidosis involves changes in plasma [HCO3−], whereas plasma [HCO3−] is unchanged in uncompensated metabolic acidosis. D. Uncompensated respiratory acidosis is associated with a change in Pco2, whereas in uncompensated metabolic acidosis Pco2 is constant. 5. O2 delivery to the tissues would be reduced to the greatest extent in A. a normal subject breathing 100% O2 on top of Mt. Everest B. a normal subject running a marathon at sea level C. a patient with carbon monoxide poisoning D. a patient who has ingested cyanide E. a patient with moderately severe metabolic acidosis

The primary function of hemoglobin in the body is to combine with oxygen in the lungs and then to release this oxygen readily in the peripheral tissue capillaries, where the gaseous tension of oxygen is much lower than in the lungs.



Reaction 1 o is catalyzed by carbonic anhydrase. o helps decrease the pH of blood during hyperventilation. o occurs primarily in plasma but can also occur in the red blood cell , at faster rate with the help of carbonic anhydrase The reactions move to the left  when there is excess H+ in the tissues. Guyton.Textbook of Medical Physiology 11th ed.Chapt 40, pgs 510-511.

ANSWER: D

Ganong. Review of Medical Physiology 23rd ed. Chapter 36, pages 615-616 ..

ANSWER: •

• • •

oxygen concentration: a normal subject breathing atmospheric O2 on top of Mt. Everest oxygen transport : a patient with carbon monoxide poisoning oxygen utilization: a patient who has ingested cyanide oxygen delivery : a patient with moderately severe metabolic acidosis. (high H+ shifts the O2-Hgb curve to the RIGHT) Ganong. Review of Medical Physiology 23rd ed. Chapter , page .

TOPNOTCH MEDICAL BOARD PREP MED PHYSIOLOGY GANONG SUPPLEMENT HANDOUT BY THE TOPNOTCH TEAM Page 41 of 46 For inquiries visit www.topnotchboardprep.com.ph or https://web.facebook.com/topnotchmedicalboardprep/

TOPNOTCH MEDICAL BOARD PREP PHYSIOLOGY GANONG SUPPLEMENT HANDOUT BY THE TOPNOTCH TEAM For inquiries visit www.topnotchboardprep.com.ph or https://www.facebook.com/topnotchmedicalboardprep/

(CHAPTER 36) REGULATION OF RESPIRATION ANSWER: D 1. The main respiratory control neurons A. send out regular bursts of impulses to expiratory muscles during quiet respiration. B. are unaffected by stimulation of pain receptors. C. are located in the pons. D. send out regular bursts of impulses to inspiratory muscles during quiet respiration. E. are unaffected by impulses from the cerebral cortex.



Main respiratory control neurons: o is not active during normal, quiet  breathing, when expiration is passive o are located in the medulla. o send out regular bursts of impulses to inspiratory muscles during quiet respiration. o are affected by impulses from the pain receptors and cerebral cortex. Guyton.Textbook of Medical Physiology 11th ed.Chapter 41 , pages 514-515

2. Intravenous lactic acid increases ventilation. The receptors responsible for this effect are located in the A. medulla oblongata. B. carotid bodies. C. lung parenchyma. D. aortic baroreceptors. E. trachea and large bronchi.

ANSWER: B Carotid bodies: o Peripheral chemoreceptors located at the bifurcation of the common carotid arteries. o Stimulated in cases of: ▪ Decreases in arterial PO2 (
View more...

Comments

Copyright ©2017 KUPDF Inc.
SUPPORT KUPDF